Jump to content

Wikipedia:Reference desk/Science: Difference between revisions

From Wikipedia, the free encyclopedia
Content deleted Content added
Line 1,256: Line 1,256:


:That's completely wrong. [[Entropy]] is not a measure of how many words it takes to give a macroscopic description of an object. It is measure of how many microscopic states are consistent with the macroscopic description. To labor on your example, if you were to melt the moon, mix if thoroughly and find a way to cool it fast enough to keep the mix uniform the final sphere would have a higher entropy. Read [[entropy of mixing]]. [[User:Dauto|Dauto]] ([[User talk:Dauto|talk]]) 15:42, 5 March 2010 (UTC)
:That's completely wrong. [[Entropy]] is not a measure of how many words it takes to give a macroscopic description of an object. It is measure of how many microscopic states are consistent with the macroscopic description. To labor on your example, if you were to melt the moon, mix if thoroughly and find a way to cool it fast enough to keep the mix uniform the final sphere would have a higher entropy. Read [[entropy of mixing]]. [[User:Dauto|Dauto]] ([[User talk:Dauto|talk]]) 15:42, 5 March 2010 (UTC)
:: I find this extremely hard to believe. You are telling me if we took the universe, melted it all together, and made a black hole out of it, with precisely 0 information in the black hole other than maybe it's total mass (a single real number, in grams) and MAYBE one or two more variables such as it's spin and charge (maybe) then there would be MORE entropy in Universe (even though you can just describe it as "1 black hole, in the "center" (ha ha) of nothing else, having mass x, charge y, and angular momentum z". Even if you give all of these to an obscenely unrealistic level of exactness, you still will use maybe a paragraph of digits. A paragraph, even using the best theoretically possible compression, is not enough to accurately describe (ie represent a compressed version of) even a single book (say, a collection of Shakespeare plays). So it seems to me that a SINGLE book would have more entropy than all of the universe, if you reduced the universe to a black hole. Likewise, it seems to me that a SINGLE city on Earth would have more entropy than the Moon, if the moon were a uniform substance you can perfectly describe in a few words. If I really am wrong, maybe it's because I'm conflating [[physical entropy]] with [[information entropy]]? For me, the fewer words you can use to give a second God in a different Universe enough information to fully reproduce an exact copy of something, the lower entropy it has. Our God would need to give a LOT of information to a second God in a different Universe to reproduce the Earth, but considerably less if the Earth were a uniform ball that is an exact geometrical sphere, of fixed temperature, density, etc. Don't you think? Can someone confirm whether I'm right, or whether Dauto above is right? Thank you. [[Special:Contributions/82.113.121.103|82.113.121.103]] ([[User talk:82.113.121.103|talk]]) 17:13, 5 March 2010 (UTC)


::These answers are just leading to more confusion. Dauto, is the Moon higher or lower entropy than the Earth? Why? [[User:TheFutureAwaits|TheFutureAwaits]] ([[User talk:TheFutureAwaits|talk]]) 15:56, 5 March 2010 (UTC)
::These answers are just leading to more confusion. Dauto, is the Moon higher or lower entropy than the Earth? Why? [[User:TheFutureAwaits|TheFutureAwaits]] ([[User talk:TheFutureAwaits|talk]]) 15:56, 5 March 2010 (UTC)

Revision as of 17:14, 5 March 2010

Welcome to the science section
of the Wikipedia reference desk.
Select a section:
Want a faster answer?

Main page: Help searching Wikipedia

   

How can I get my question answered?

  • Select the section of the desk that best fits the general topic of your question (see the navigation column to the right).
  • Post your question to only one section, providing a short header that gives the topic of your question.
  • Type '~~~~' (that is, four tilde characters) at the end – this signs and dates your contribution so we know who wrote what and when.
  • Don't post personal contact information – it will be removed. Any answers will be provided here.
  • Please be as specific as possible, and include all relevant context – the usefulness of answers may depend on the context.
  • Note:
    • We don't answer (and may remove) questions that require medical diagnosis or legal advice.
    • We don't answer requests for opinions, predictions or debate.
    • We don't do your homework for you, though we'll help you past the stuck point.
    • We don't conduct original research or provide a free source of ideas, but we'll help you find information you need.



How do I answer a question?

Main page: Wikipedia:Reference desk/Guidelines

  • The best answers address the question directly, and back up facts with wikilinks and links to sources. Do not edit others' comments and do not give any medical or legal advice.
See also:


March 1

Scientocracy

Scientocracy is listed on the "Wikipedia:Requested articles/Social sciences" page at http://en.wikipedia.org/wiki/Wikipedia:Requested_articles/Social_sciences

I have written a draft of an article about scientocracy. See http://en.wikipedia.org/wiki/User:JasonCupertino/Sandbox

Is that draft ready to be posted in Wikipedia's main space as a new article? JasonCupertino (talk) 00:17, 1 March 2010 (UTC)[reply]

It could use a section of the history of usage of the word. When did people first start using it? How many are using it now? etc. --Tango (talk) 00:23, 1 March 2010 (UTC)[reply]
The Ref Desk isn't really the place for this sort of question - on the other hand, I don't really know where it _should_ be posted. WP:DRAW, or WT:WikiProject Sociology perhaps? Anyway, the article looks quite reasonable to me - I would probably remove the first paragraph, especially the part about Ghits, and just start "Scientocracy describes the practice of basing public policies on science." A citation for the first paragraph of the "Scientocracy as a form of government" section would also help - which of the various authors you cite has this opinion? If it's derived from more than one author, add a reference to the appropriate author to the corresponding sentence - if it's just one of them, add the reference to the end of the paragraph. If it's your own opinion, however, it shouldn't be in the article, per WP:OR. However, these are just minor issues - I'm sure it would survive an AfD if it were to go live now. If you can't move it yourself, add the {{move draft}} to the top of the article. Tevildo (talk) 00:44, 1 March 2010 (UTC)[reply]
I strongly disagree with User:Tango. Your first line is "Scientocracy is a neologism" - which is an immediate red-flag. Have you read WP:NEO? If not - please do so immediately - especially the section "Articles on neologisms"! You need to be sure that this isn't an article about the word (Wikipedia isn't a dictionary) - so dump the entire first paragraph because that, right there, would be enough to cause the article to be deleted. It needs to be strictly about what scientocracy is - not about the word itself. Hence adding history of the usage of the word would be a very bad thing. The very last thing you want to emphasise is the newness of the word - you have to stick to the concept behind it - and kinda reluctantly use this admitted neologism to describe that concept. That said - without the first paragraph, it's not a bad article and I'd post it into main-space and see what happens when it's exposed to other editors. SteveBaker (talk) 01:14, 1 March 2010 (UTC)[reply]
Replace "word" in my comment with "concept" and the point stands. The difference between a word and the concept described by that word is largely meaningless. --Tango (talk) 01:19, 1 March 2010 (UTC)[reply]
For where it should be posted, I would say, besides WP:DRAW and any applicable WikiProjects like Tango said, WP:HD and WP:RfF are good for things like this. —Akrabbimtalk 01:21, 1 March 2010 (UTC)[reply]
That was Tevildo, not me. --Tango (talk) 02:32, 1 March 2010 (UTC)[reply]
For what it's worth, I'd vote against it on AfD as a neologism. Comet Tuttle (talk) 01:25, 1 March 2010 (UTC)[reply]
The execrable Pomosexual survived AfD ([1]) on a far flimsier footing; however, past history is no guide in that particular area of the project. Tevildo (talk) 01:45, 1 March 2010 (UTC)[reply]
You know, I'm convinced I !voted against that - why else would I know about it? - but my opinion isn't in the AfD. Was there a second one, or is this a glitch in the Matrix? Tevildo (talk) 02:23, 1 March 2010 (UTC)[reply]
The general requirement for inclusion in Wikipedia is the existence of good sources. This article seems to satisfy that. Being new doesn't count against it - plenty of things get articles as soon as they come into existence. --Tango (talk) 02:36, 1 March 2010 (UTC)[reply]
Are you saying that "Scientocracy" exists? It does not exist. Comet Tuttle (talk) 04:01, 1 March 2010 (UTC)[reply]
That depends on your definition of "exists". As an abstract concept, it definitely exists. --Tango (talk) 05:02, 1 March 2010 (UTC)[reply]
As an article about a word - it's newness definitely counts against it (per WP:NEO) - as a concept, newness is not necessarily an obstacle. As presented to us, this article would be about the word and that would likely get AfD'ed into oblivion in short order. However, without the first paragraph, this would be an article about a concept - with an unfortunate choice of a neologism for a name - which ought to pass muster. SteveBaker (talk) 13:03, 1 March 2010 (UTC)[reply]
The concept may sound appealing but it's so poorly defined, even with the references, that there's just no way. "Caninocracy" gets google hits and is appealing to some people and should not be a Wikipedia article about it. Comet Tuttle (talk) 15:26, 2 March 2010 (UTC)[reply]

Tango: I added a section that presents what I think may be the first use of "scientocracy". I also added a reference to an online Oxford English Dictionary search yielding "no results". But I should probably delete those things if I am going to eliminate the reference to scientocracy being a neologism.

Thank you one and all for taking the time to evaluate my draft of scientocracy. I will keep working on it with your comments in mind. JasonCupertino (talk) 06:03, 1 March 2010 (UTC)[reply]

The word scientocracy is not in Wikitionary. You may try to get it added there. Here is some etymology extracted from [2]:
science c.1300, "knowledge (of something) acquired by study," also "a particular branch of knowledge," from O.Fr. science, from L. scientia "knowledge," from sciens (gen. scientis), prp. of scire "to know," probably originally "to separate one thing from another, to distinguish," related to scindere "to cut, divide," from PIE base *skei- (cf. Gk. skhizein "to split, rend, cleave," Goth. skaidan, O.E. sceadan "to divide, separate;"
-cracy comb. element forming nouns meaning "rule or government by," from Fr. -cratie or M.L. -cratia, from Gk. -kratia "power, might; rule, sway; power over; a power, authority," from kratos "strength," from PIE *kratus "power, strength". The connective -o- has come to be viewed as part of it. Productive in English from c.1800. Cuddlyable3 (talk) 10:56, 1 March 2010 (UTC)[reply]
If it is a "new" neologism, then it probably should go in Wiktionary's list of "protologisms". It is unlikely to merit an entry in the dictionary unless you can cite permanently recorded media, conveying meaning, in at least three independent instances spanning at least a year (or use in a refereed academic journal). Just a mention or a definition of the word is not sufficient to merit inclusion. Dbfirs 12:32, 1 March 2010 (UTC)[reply]
... (later) ... actually, you do seem to have mentioned the required citations in your article, so perhaps the word is not so "new"? Dbfirs 12:38, 1 March 2010 (UTC)[reply]

nuclear versus fossil

France is the world leader in nuclear power. Cuddlyable3 (talk) 10:37, 1 March 2010 (UTC) ... but only if you are considering percentage of their total electricity generation, and not total number of plants or amount of electricity produced. If you consider the latter to be important, then the U.S. is the (reluctant) leader. --Mr.98 (talk) 12:43, 1 March 2010 (UTC)[reply]

How many nuclear reactors would be required to replace all of the fossil fuel now being used by the world? 71.100.5.197 (talk) 07:56, 1 March 2010 (UTC)[reply]

This says there are 436 reactors in the world, and the same website claims they provide 15% of the world's power. So, you'd need 6.66 times more reactors, or 2907 reactors in total. Of course, that assumes that the website is accurate, and that all reactors supply the same amount of energy (which they don't.) Vimescarrot (talk) 09:51, 1 March 2010 (UTC)[reply]
It wouldn't be 6.6 times even under those assumptions; fossil fuels do not account for the entire remainder of 85%. Someguy1221 (talk) 09:56, 1 March 2010 (UTC)[reply]
According to our nuclear power article, nuclear power supplies approximately 15% of the world's electrciity, but only about 2% of the world's energy. According to fossil fuel the breakdown of primary energy consumption (not sure how this is defined) is 86.4% fossil fuels, 8.5% nuclear, 6.3% hydroelectric, 0.9% other (hmmm ... those figures add up to 102.1% !). Gandalf61 (talk) 10:34, 1 March 2010 (UTC)[reply]
To eliminate all of the fossil fuels, you'd also have to find ways of not using coke in blast furnaces and in all sorts of other industrial settings, replacing every ship, diesel train, car, motorbike, bus and truck with either all-electric or hydrogen powered vehicles that could ultimately get their power from nuclear sources. Some forms of transport (such as aircraft) are not easily converted - I don't believe there is a single powered aircraft anywhere in the world that does not run on fossil fuels - and I don't see a whole lot of research being done to fix that. Some of those industrial processes would also be very difficult to replace with electrical processes. But if we believe Gandalf61's figures, we obviously need to increase our nuclear power base by a factor of 10. So we'd need about 4,400 reactors. I suspect that's an under-estimate because there are inefficiencies in using electricity to charge batteries and make hydrogen that are not accounted for in the energy extracted from fossil fuels directly. (If a car is currently rated at requiring X joules of energy per mile - then it will need X joules of hydrogen power to do the same thing - but it would take X+Y joules of nuclear-generated electricity to make that hydrogen - where Y is the inefficiency in the process.)
The obvious problem with doing this are that the capital investment to replace all of those users of fossil fuels with electric power would be spectacular - not least because nuclear power plants are 20 to 30% more expensive over their lifetime than coal plants...but WAY more expensive to construct in the first place.
Some critics of nuclear power would also claim that there is a problem with nuclear plants doing something called "load following" - when there is a major sporting event on TV, electricity consumption spikes when it's over as everyone goes off to cook food or whatever. The claim is that you can't ramp up power from your nuclear plants fast enough to meet that demand. However, France gets 80% of its power from nuclear and it manages this just fine - so there must be ways to fix that.
Another issue is that there are many places in the world with unstable governments and low technical expertise where nuclear power plants would be unsafe to run - and a dangerous source of fissionable products for nuclear weapons. That would be a serious consideration.
Finally, if electricity were to jump from being perhaps 20% of our energy supply to 90% or more, we'd need much more transmission line capability around the world. This kind of additional infrastructure doesn't come cheap.
Certainly we're going to need to dramatically increase our nuclear power base if we're going to duck this global warming catastrophy - but to make this work, we need a greater diversity of sources - and we need to cut consumption fairly dramatically too. It's much cheaper to use less than to generate more. Consider how many 20mpg cars there are on our roads and note that we have 40mpg cars (my car does 42) without resorting to hybrid technology or even suffering poor performance. If we pushed the technology, we could certainly get to 60mpg. If we did that then we'd drop the amount of energy consumed by cars by a factor of three - which would save about 16 billion gallons of gasoline a year...representing several hundred nuclear power plants. That kind of thing is a much better short-term fix than a massive nuclear build-out program.
SteveBaker (talk) 12:58, 1 March 2010 (UTC)[reply]
Comment - nuclear power does perform better at load following than coal, on account of nuclear control rod and associated throttling technology. Nimur (talk) 23:51, 1 March 2010 (UTC)[reply]
Indeed, however I believe there is a minimum energy production you need to maintain or it all goes horribly wrong. Shutting down a nuclear reactor is a lot of work, as is starting it back up again. Between that minimum and whatever the maximum is, you can vary things reasonably easily, yes. --Tango (talk) 01:35, 2 March 2010 (UTC)[reply]
I saw a talk by Richard Garwin a few years back that advocated thinking about these technologies as "wedges" in an overall pie of things to do, if you wanted to avoid global warming issues. One is increased nuclear. One is increased renewables. One is increased efficiency. One is increased capping and scrubbing. Maybe there were a few more, as well. But the emphasis was on the idea that you aren't going to have just one of these ideas actually do all of the work needed—and that focusing on trying to get one massive program adopted was surely going to fail, whereas doing a few things modestly, but simultaneously, would probably get a lot done. (Personally, I'm dubious that we can even get modest things done.) --Mr.98 (talk) 14:16, 1 March 2010 (UTC)[reply]
A side note... model airplane hobbyists have since the days of floppy, hard drive and CD/DVD motors experimented with "Y" and Delta windings to achieve higher RPM or higher torque. Several manufacturers have made still further improvements where shrouded props provide models with very high speed. Battery technology is a problem when scaled but promise lies in ultra-capacitors so like nuclear fusion capacitor powered airplanes might not be that far away. Next question I'll make reference to those technologies rather than just fission and lower energy density alcohol fuel as now. 71.100.5.197 (talk) 13:49, 1 March 2010 (UTC)[reply]
Even the most ambitious real-world (e.g. non-internet) boosters of nuclear fusion don't see it as contributing much if anything to the total electrical generation for another 40 years at a minimum. It is not really on the table in a real-world energy calculation. --Mr.98 (talk) 14:16, 1 March 2010 (UTC)[reply]
There is at least one manned solar plane: Solar Impulse[3]. Unmanned ones, too: QinetiQ Zephyr and NASA Pathfinder. Rmhermen (talk) 14:29, 1 March 2010 (UTC)[reply]
Well, there are some electrically powered gliders too - they pull the plane up to enough altitude to pick up thermals and such - but there isn't enough power to do useful work. SteveBaker (talk) 02:27, 2 March 2010 (UTC)[reply]
Yes, but there is no way those solar planes could replace current planes. They just don't have the lifting power to transport passengers or cargo. --Tango (talk) 20:50, 1 March 2010 (UTC)[reply]
Indeed, fusion is a long way off. Small scale fusion in individual devices is even further off. --Tango (talk) 20:50, 1 March 2010 (UTC)[reply]
The hope is that if we can build enough fission plants to keep us out of disaster - then maybe fusion can take over before we run out of dark places to hide the crap that the fission plants produce. But neither technology will fix the problem alone. We need boatloads of windmills and lots and lots of trimming of demand. SteveBaker (talk) 02:27, 2 March 2010 (UTC)[reply]
Or we can reprocess the crap and reuse it in breeder reactors, which will allow us to use fission for the next 3000 years without generating any significant nuclear waste. Clear skies 24.23.197.43 (talk) 06:42, 2 March 2010 (UTC)[reply]
That's a little naive. You should read Radioactive waste. Sure, you can reprocess spent fuel rods - but that's not the only radioactive waste these plants produce. Things like worn out equipment that's been exposed to radiation and end-of-life parts of the reactor itself produce low and medium level radioactive waste that cannot be re-used and yet is not safe to dispose of in ordinary ways. The casings of reprocessed fuel rods are highly radioactive and cannot be re-used when the rods are reprocessed. I read somewhere (sorry, I forget where) that just disposing of overalls that workers had worn inside the plant caused a significant waste storage/disposal problem at several US reactors (there is passing mention of this in the Radioactive waste article). SteveBaker (talk) 13:20, 2 March 2010 (UTC)[reply]
Not wanting to pick a technical fight, but I am pretty sure that reprocessing does significantly reduce the mass and volume of highly radioactive materials needed for long-term repositories. The problem with waste storage is mostly political, in any case—a problem of a lot of people against having waste sites anywhere, and a political system which requires waste storage sites to try and meet really fairly crazy and unscientific criteria (like being able to prevent basically any deaths over the course of thousands and thousands of years). If we were able (which in the US, we aren't) to politically treat nuclear waste the way we do with other, persistent and nasty industrial wastes (many of which are never going to decay into something less harmful), we'd be able to find an acceptable solution (one which would indeed probably involve a small number of people sacrificing something for the greater good, e.g. being relocated permanently). But we can't, and so we probably won't. But reprocessing would reduce a lot of the nasty stuff (and also make more use out of the existing uranium). The spent fuel rods are definitely the worst kind of radioactivity that the plants produce—fission products, transuranics, etc. Low-level nuclear waste (the overalls) requires its own procedures but it is orders of magnitude less dangerous than the high-level stuff. --Mr.98 (talk) 18:39, 3 March 2010 (UTC)[reply]
BTW, with enough hydroelectric dams (especially hydroelectric storage), we won't even need no stinkin' windmills, which aren't cost-effective anyway. 24.23.197.43 (talk) 06:51, 2 March 2010 (UTC)[reply]
Dams are being removed rather than created in the US because they aren't a good long term solution in most places. Aside from the damage they do to wildlife on these rivers, they also silt up over time and become gradually less effective. They are often horribly difficult and costly to remove - and when you figure that into the lifetime cost of them - they come out much less cost-effective than had previously been assumed. I really can't agree with you about the cost-effectiveness of wind power - what you are saying simply isn't backed up by science. There have been many studies on this - some find it to be marginally cheaper than coal and gas plants - others marginally higher (much depends on the projected lifespan of the equipment - which is something of an unknown right now). Probably the most comprehensive study said that cost per unit of energy for wind was estimated at $55.80 per MW·h, coal at $53.10/MW·h and natural gas at $52.50. (These numbers are quoted, with references in Wind_power#Economics_and_feasibility). If/when we get a proper accounting of the 'costs' of CO2 pollution and any kind of carbon tax or cap & trade system - then windmills will turn out to be highly economical - which is why so many of them are being built right now. However, they aren't good at load-following because you don't always have enough wind - and there aren't enough suitable places to build them in many parts of the world. Hence nuclear is likely to be a good part of the solution. SteveBaker (talk) 13:20, 2 March 2010 (UTC)[reply]
When I mentioned dams, what I meant was that they are uniquely useful for energy storage: you can just use excess electrical power at off-peak times to pump the water upstream, and then use the stored hydraulic power to generate electricity at peak times. You can't do that with any other kind of power plant, and definitely not with a windmill. And speaking of windmills, do the numbers you quoted include the capital cost of building them, or just the operating cost? 24.23.197.43 (talk) 05:31, 3 March 2010 (UTC)[reply]
I just looked at the references in Wind_power#Economics_and_feasibility; the main source for the figures you gave is a study done under the auspices of the BWEA, which may have a vested interest in promoting wind power. Do you know any independent studies that conclude wind power is economically competitive and sufficient to meet US electricity production needs? 24.23.197.43 (talk) 05:43, 3 March 2010 (UTC)[reply]
There are other references in that article that come up with somewhat more optimistic and somewhat less optimistic figures for the cost of wind energy. The BWEA study is actually right in the middle of the range - so it's probably the most reasonable guesstimate. Also, I doubt anyone believes that wind power is enough to meet US electricity production needs - not even close. We're going to need nuclear power and solar and some hydro (and yes, some coal and gas). We're also going to have to cut those demands back fairly sharply - because saving energy is much easier and cheaper than generating it sustainably. What I'm trying to point out here is that our OP's efforts to suggest an all-nuclear solution is not the best way to go. Besides that, having a diversity of methods gives us the most flexibility in the future - if we suddenly find some horrible and unforeseen problem with any one of the technologies, we can phase it out relatively quickly and do something different without having the entire country invested in one solution. SteveBaker (talk) 15:24, 3 March 2010 (UTC)[reply]

Methane from CO2 and H2?

Given, a priori, a limitless supply of cheap energy, is it possible to make methane from carbon dioxide and water, and then onto more complex hydrocarbons? Obviously this would be highly inefficient, but if workable, might be useful where a high energy/weight density power store is needed, such as aviation? CS Miller (talk) 19:21, 2 March 2010 (UTC)[reply]

It's certainly possible - but if we can get cars and trucks and trains and boats off of hydrocarbons, then we would be able to use biofuels like ethanol for aircraft. The point is not to get fixated on a single solution such as "produce 100% of mankinds energy needs from nuclear reactors" because doing that forces you into that kind of extreme inefficiencies. SteveBaker (talk) 20:09, 2 March 2010 (UTC)[reply]
But how in the world are you gonna "get cars and trucks and boats off of hydrocarbons" in the first place? Any ideas, Steve? 24.23.197.43 (talk) 05:02, 3 March 2010 (UTC)[reply]
Introduce a tax on carbon emissions that makes it cost effective to switch to electric or hydrogen cars and sit back and watch. Easy. (Technically easy, anyway. Politically it's a nightmare.) --Tango (talk) 05:50, 3 March 2010 (UTC)[reply]
I was asking specifically for technological ideas on how to do that, not the same ol' carbon-tax thing that would only reduce emissions by reducing economic activity. Specifically, I'd like to know how you plan to (1) improve the energy density and recharge rate of electric cars so it won't take all day to recharge their batteries, (2) address the serious safety hazards of hydrogen-powered cars, and (3) adapt either or both technologies to produce enough power for your typical eighteen-wheeler? I'm standing by for any ideas you might have. 24.23.197.43 (talk) 06:03, 3 March 2010 (UTC)[reply]
I plan to sit back and watch while businesses work out how to make profit. They are extremely good at that. The problems you mention are fairly simple engineering problems - throw enough money at them and a solution will turn up. I'm not remotely qualified to solve the problems myself, but I have great faith in the ability of the world's experts to do great stuff when given the necessary funding. --Tango (talk) 06:12, 3 March 2010 (UTC)[reply]
Are you saying that the hydrogen-safety issues and the battery charge limitations are "simple engineering problems"?! Obviously you had never worked with either of these things (and especially not hydrogen)! Anyone else have any ideas? 24.23.197.43 (talk) 06:42, 3 March 2010 (UTC)[reply]
The battery charge issue is trivial. Simply have a standard form factor and connector, and have a battery exchange system at the fuel station. Drop off the old one, put in a new one. That should take less than refueling takes now. You can also keep two at home, one for charging, one for using. Use a deposit system or something to handle battery ownership and possession. There is a company in Israel that is developing that system, but I forgot the name. --Stephan Schulz (talk) 13:55, 3 March 2010 (UTC)[reply]
It's not really that simple. A full set of batteries for (for example) a Prius costs several thousand dollars. They are a huge fraction of the cost of a car. So how many of these can a typical gas station afford to have "in stock"? 60? That's maybe $150,000...a big investment. Yet they might easily see 60 customers per hour (gas stations have maybe a dozen gas pumps - they see WAY more than 60 customers an hour). That means that they have to be able to recharge these things at a rate of one per minute - an hour to recharge a battery pack! That's not possible right now - so they'll need more like 600 battery packs in order to satisfy a 60 customers per hour rate - and now they have a 1.5 million dollar investment in batteries alone - that's not including the difficult robotic lifts that swap the batteries out and all of the charging stations. So now we have (probably) a multiple millions of dollars cost to opening a new battery-swap station. For those to be common enough for everyone to use, we need to have every gas station in the country spend several million dollars. There are close to 200,000 gas stations in the US alone - so if we're generous and put the cost at $2,000,000 to equip one with batteries, rechargers and battery-swapping robots - you're looking at 200 billion dollars of outlay. That's not outragous - but there is a chicken-and-egg problem. Until there are (let's say) 100 million cars out there that can use battery-swap technology - how is this $200 billion investment going to pay off? Nobody is going to buy a battery-swap car until there is enough infrastructure out there so that when they roll into a gas station with 10 miles worth of charge in their batteries they won't hear "Sorry - we just sold our last battery - it's going to be 5 hours until the next one is recharged!"
I think we really need a cultural change. We need to have people buy 100 mile range electric cars for daily driver usage (with charging sockets in company parking lots) - and be able (and willing) to rent long-range gasoline vehicles for road trips. It's not a terrible restriction - I'd go for it in a heartbeat because it makes sense. But for too many people, that's not an acceptable solution. SteveBaker (talk) 14:42, 3 March 2010 (UTC)[reply]
First, those are not technical problems, they are economic problems. Moreover, your calculation is off. Not everyone will have an electric car immediately, and not every gas station will have to provide a battery service immediately. If there is one per neighborhood that's enough - assume electricity is half the price of fuel per mile, people will gladly drive a bit further to the next station. I know some drive 20 km to fuel up for a few cents less per liter here in Germany. And those gas stations that offer the service will only need enough supply to deal with actual business (plus a few spares for reserve). Of your 60 cars/hour, in the beginning only one or two will be electric. As the business ramps up, the supply of batteries be ramped up. And, of course, if batteries are standardized and and used widely, their price will come down quite a bit. --Stephan Schulz (talk) 18:49, 3 March 2010 (UTC)[reply]
Thinking about this - what you'd really need is a system where the car is told "I plan to drive from here to there today" - the car then calls up gas stations along the route and books the batteries it's going to need to make the trip so you'll know they are going to be ready and charged at the gas station when you roll up there. With that degree of confidence about availability, the gas stations could plan ahead...maybe even move a bunch of pre-charged batteries in a truck from some central storage place to cover peak demands in unusual circumstances. If you changed your mind and take a different route - you'd have to be prepared for the car to tell you that you can't go that way because there aren't enough available batteries en-route to make it to your destination - maybe you'd even have to pay money to the gas stations whose batteries you'd reserved but didn't take. Combined with the ability to recharge at home and at work - this might produce a workable solution. But it would take some considerable consumer re-adjustment to make that work. SteveBaker (talk) 15:13, 3 March 2010 (UTC)[reply]
Another idea is to make the batteries smaller - say a Prius will normally get 3, a Lexus 5, and an E-Hummer 256. If a gas station runs low, they can ration batteries and, in the (hopefully rare) pinch situation, just provide enough to tide the driver over to the next station along his route. Or they can provide only partially charged cells (at a discount, of course), again to tide the driver over. Sure, it's an annoyance, but then so is gasoline, and fuel stations running out of gasoline is not unheard of even in the US. --Stephan Schulz (talk) 18:53, 3 March 2010 (UTC)[reply]
The thing about waiting for businesses to do this in order to make a profit is that their definition of "profit" is different from ours. If there is no cost whatever to dumping CO2 into the atmosphere (and there currently isn't) - then it is certainly vastly more profitable to burn coal and cause unending amounts of grief to future generations than it is to research cleaner technologies. If you want capitalism alone to cause free enterprise to want to do it - you have to adjust the rules such that producing CO2 comes up on the "costs" side of the balance sheet. If "producing a shit-load of CO2" is expensive then in order to make a profit, they'll have to clean up their act. The alternative is to write laws to simply make it illegal to produce more than some limit of CO2. I don't really care which we do - but the latter tends to upset the "free enterprise" thing.
In the automobile industry, (for example) we have the CAFE standards that do it the latter way - we require that the average fuel consumption of automobiles sold in the USA meet some specific goal. But we don't reduce that number fast enough - and we don't do similar things for things other than cars. One gasoline powered snow blower/leaf blower/chainsaw/lawn mower can produce truly insane amounts of pollution with (mostly) zero regulation. Big ships are often required to burn relatively clean kinds of oil when in US waters - but are allowed to switch to cheaper, exceedingly dirty stuff once they are out in international waters.
This kind of regulation is exceedingly patchy and because it's done with a complicated network of tiny rules and regulations, it's easily abused by lobby groups who can get a law changed for a single industry segment without causing national outrage. The good thing about "carbon taxation" is that it can apply uniformly across the whole of business with a uniform pricing scheme. No one industry would get their own laws - so the cost of CO2 emissions shows up on their balance sheet - and with luck, that will cause the profit mechanism to kick in and cause them to do research. It also has the nice property of creating the most incentive for the worst offenders. Coal power plants shouldn't be the cheapest form of energy - they should be by far the most expensive...a proper accounting for their real costs would make that clear and would result in more appropriate technologies becoming so blindingly obviously "profitable" that money would flock into building windmills or nuclear power plants or whatever it takes to solve this problem. SteveBaker (talk) 14:22, 3 March 2010 (UTC)[reply]
Agree absolutely. The best and simplest way to deal with the commons is to assess a fair price and set taxation so that that price will be internalized by businesses using the commons. Then let the market do its thing. To avoid disruption, taxes can be phased in in a predictable manner over 10 years or so. --Stephan Schulz (talk) 18:39, 3 March 2010 (UTC)[reply]

star composition

At the densities required to allow hydrogen to fuse into helium or any other element up to iron will the material with the higher atomic weight be the one to fuse,i.e. if the amount of carbon is the same, will carbon be the element to fuse if the amount of hydrogen is the same? 71.100.5.197 (talk) 14:00, 1 March 2010 (UTC)[reply]

If I understand your question correctly, some fusion reactions are more common on the aggregate than others. At different temperatures/densities, different reactions dominate. --Mr.98 (talk) 15:05, 1 March 2010 (UTC)[reply]
Does a star burn up all its hydrogen or does helium start fusing before all the hydrogen burns up. In other words what conditions cause helium or carbon to take over as the primary material of fusion? Is it the absence of less heavy elements or just that temperature and density favors one over another? In other words if you injected a star size amount of hydrogen into a helium burning star would it start burning hydrogen again or keep burning helium? 71.100.5.197 (talk) 17:17, 1 March 2010 (UTC)[reply]
It depends how much hydrogen you inject. The hydrogen doesn't run out entirely, it just gets too low to sustain sufficient fusion to stop the star collapsing under its own gravity. The star then starts to collapse until the heat and pressure is enough to fuse helium (it will still be fusing some of the left over hydrogen). If you injected a little more hydrogen it would just fuse that hydrogen while still fusing helium. If you injected a lot then it would fuse the hydrogen, generating lots of radiative pressure that causes the star to expand and cool, stopping the fusion of helium until the hydrogen runs out again and the star re-collapses. --Tango (talk) 17:50, 1 March 2010 (UTC)[reply]
It also depends on the star. The sun is fairly layered - it's slowly accumulating helium "ash" in its center. Eventually, the pressure will be high enough to start helium fusion. But some smaller stars (e.g. red dwarfs) are completely mixed by convection. They can keep fusing hydrogen much longer. --Stephan Schulz (talk) 22:08, 1 March 2010 (UTC)[reply]
The concept of a helium flash is also relevant. --Tango (talk) 01:30, 2 March 2010 (UTC)[reply]

Why not take from the sun?

Instead of expensive fusion initiation why not just go to the sun and scoop some already fusioning material and just keep it going with cheap gasses on Earth? 89.204.153.2 (talk) 14:49, 1 March 2010 (UTC)[reply]

First, getting to the sun at all is hard, because you need a very large delta-v. Second, the nuclear fusion in the sun takes place in the core, so it would be incredibly hard to create something that could survive the temperatures and pressures needed to reach the core, and get back out again. Third, the fusion reactions there are sustained by that pressure, which is a result of the gravity of the sun's enormous (compared to the earth) mass. If you remove the fusing material from that environment, the reaction will not be sustained. -- Coneslayer (talk) 14:53, 1 March 2010 (UTC)[reply]
I think "incredibly hard" is probably optimistic. I suspect there is no human-made object that could possibly make it to the core of the Sun and back in one piece. --Mr.98 (talk) 15:01, 1 March 2010 (UTC)[reply]
...and in any case we already have the technology to initiate a fusion reaction - that is relatively easy. What we lack is the technology to contain and control a fusion reaction without consuming more energy than it produces. Gandalf61 (talk) 15:04, 1 March 2010 (UTC)[reply]
what if we don't try to bring it back then but just park a power plant on the surface of the sun? Isn't cooling trivial when you had that much energy at your disposal? 89.204.153.101 (talk) 15:15, 1 March 2010 (UTC)[reply]
Good point, we could just have the power plant convert the energy into some form of radiation and 'beam' it back to the Earth. I think that the method has been tested before, it just might work! All we need is a power plant that can survive on the surface of the Sun. --144.191.148.3 (talk) 15:25, 1 March 2010 (UTC)[reply]
what if you use the energy right there? For example, A server farm doing massive massive number crunching a la NSA or complex rendering a la Pixar? Then would it make sense? 89.204.153.97 (talk) 15:45, 1 March 2010 (UTC)[reply]
It'll be tried soon and will succeed, but with unfavourable results. Bazza (talk) 15:55, 1 March 2010 (UTC)[reply]
The temperature on the surface of the Sun alone is some 5,778 K — nearly 10,000ºF. Even with a lot of energy I don't see that as being a "trivial" cooling problem, even you neglect the fact that the surface of the Sun is rather inhospitable for a number of other reasons. (And even more so, it's hard to imagine any on-the-sun/in-the-sun based situation that is going to be cheaper than a terrestrial option.) --98.217.71.237 (talk) 16:18, 1 March 2010 (UTC)[reply]
None of these answers makes any sense for me. You're talking about FREE energy, ALL around you. I just don't see why you can't use it to cool your power plant.  :( :( :( This thing doesn't make any sense to me. Let's get down to something more basic: how does a regrigerator work in space? Normally, a refrigerator uses energy to move heat from one place to another (same as an air conditioner). So, HOW does it do that on the surface of the sun? Thanks. 84.153.196.55 (talk) 16:29, 1 March 2010 (UTC)[reply]
So you want to use incredible heat to cool things down? Not so simple, methinks. LargeScaleForest (talk) 16:34, 1 March 2010 (UTC)[reply]
I for one hope no one has taken this thread seriously since about the second or third post... The first problem with gathering energy 'on the surface of the sun' is that all modern energy harnessing techniques aside from mechanical such as tidal energy, and photovoltaic with proton-electron conversion; require a *difference* in heat energy between one place and another in order to exploit for gainful purposes. The air inside of a combustion chamber vs. the outside air, for example. Since your whole power plant would quickly become the same temperature (unless you built it with a size proportional to the sun itself, say ten million miles across) and your ability to gather up energy would be rendered null. And, for what it's worth, refrigerators only work in space if you can count on infrared emission of the heat they relocate, since there is no air to blow across the condenser coil. --144.191.148.3 (talk) 16:48, 1 March 2010 (UTC)[reply]
That's exactly the problem. On a small scale, try using the heat of your oven to make ice cubes! We have a free fusion reactor provided for us just eight light-minutes away, so if we want an engineering challenge, why not just tap some of the energy as it arrives on earth (or we could tap the sunlight as it goes past the earth, but this would cost more, and would contribute to global warming.) Coincidentally, I've just been listening to Set the controls for the heart of the sun. Dbfirs 18:25, 1 March 2010 (UTC)[reply]
In order to use power to cool things - you have to take the heat out of the cool thing and somehow dispose of it - but the only way to do that is to funnel that heat to a radiator that is itself hotter than the surrounding environment. (Don't believe me? Check out how hot those coils at the back of your refrigerator get! They have to be hotter than the room the fridge is in.) So our solar refrigerator would need a radiator hotter than the sun itself. What could we possibly make that of? There are no materials known to man that wouldn't melt (or more likely boil) at those temperatures. SteveBaker (talk) 00:33, 2 March 2010 (UTC)[reply]
Certainly the idea of grabbing a bit of sun is both crazy-impossible - and completely unnecessary. As others have said - the problem isn't getting the fuel - and it isn't starting the reaction going. The problem is containing something that's as hot as the surface of the sun using 'normal' materials. There are no materials that we know of that can survive that much heat without melting. Hence we have to use the magnetic properties of the plasma and make massive magnetic fields to contain the stuff without it coming near the walls of the reactor. But those magnetic fields don't stay stable, the kink and bend and let the plasma loose. It's really tough - which is why we haven't solved it yet.
As for picking up energy at the sun and beaming it to earth - why bother? The sun is doing a great job of beaming energy at the earth without any help from us! The problem is collecting that energy when it gets here. There is plenty of sunlight lighting up boring bits of our planet that we could (in theory) gather up and use. But the cost of all of those solar panels is daunting - and all the time that there are cheaper alternatives, we're going to go on using them. The other problem with solar energy (that it shuts off at night) would apply to power collected near the sun and beamed here somehow. So there are no benefits to doing that...and the costs would be spectacular.
SteveBaker (talk) 00:28, 2 March 2010 (UTC)[reply]
Forgive my nitpicking, but the surface of the sun has no fusion and isn't nearly hot enough. Containing something at 6000 K is damn easy compared to the Tokamak operating temperature of 20-100 million K. Also, collecting energy near the sun and beaming it to Earth isn't entirely crazy. We get ~1370 W/m^2 at the top of the atmosphere, but if you place a satellite around the sun at say 5 solar radii, you could get 2.5 MW / m^2, so you'd only need 1/1000th the number of solar panels provided they could stand the heat. Put that energy into a collimated beam towards Earth, rather than letting it spread out in all directions, and there is a potential benefit, though in practice I can't imagine how you would build such a system without the sun pretty much instantly destroying it. Dragons flight (talk) 01:07, 2 March 2010 (UTC)[reply]
You also would need to be careful that the beam you would send from your theoretical power satellite goes exactly where you want it. Otherwise, we might see something like the superweapon in Golden Eye. Googlemeister (talk) 14:34, 2 March 2010 (UTC)[reply]


somebody above wrote "The problem is containing something that's as hot as the surface of the sun using 'normal' materials. There are no materials that we know of that can survive that much heat without melting. Hence we have to use the magnetic properties of the plasma and make massive magnetic fields to contain the stuff without it coming near the walls of the reactor. But those magnetic fields don't stay stable, the kink and bend and let the plasma loose." I understand why trying to contain the plasma magnetically is hard. Why don't we just use force fields then? A force field is, by definition, like a wall, but because it's not a normal material, it won't get hot. Why don't you just make a force field, and then a vacuum, like this:

crosssection:
[x|fusion|x]

wherein the fusion is the fusion reaction in progress, the | are the force field walls, and x is a vacuum? Then, you do not have to worry about direct energy transfer to your wall through convection. As for radiation, I actually don't know if heat radiates through a force field, I imagine it must as I have not heard of anyone saying that a force field is reflective of radiation. Is there any research being done along this avenue? 82.113.121.105 (talk) 00:44, 3 March 2010 (UTC)[reply]

And how in Heaven's name do you create a force field??? 24.23.197.43 (talk) 06:50, 3 March 2010 (UTC)[reply]
The short answer is "you don't" - but Force_field is worth a read. SteveBaker (talk) 13:43, 3 March 2010 (UTC)[reply]
sometimes science is able to predict the qualities of things that can't be produced yet. Insofar as that applies at all to force fields, does real science have anything to say about what thermal insulation and what thermal radiation properties it must have? 82.113.121.110 (talk) 14:55, 3 March 2010 (UTC)[reply]

Three (or more)-particle annihilation events

Is it possible for there to be a matter-antimatter annihilation event in which there are three (or more) particles involved. I surmised from a lecture today (possibly incorrectly) that there exist particles with charges of +- 2e, which made me wonder if you could collide two electrons and one anti-particle of charge 2e leading to mutual annihilation of all three and conversion of their rest-mass- and kinetic-energies into photons.

The question intrigued me becaus (again, if i recollect correctly) a classical triple collision is mathematically singular, that is, if three classical bodies were to collide at exactly the same time there is no way to determine the resulting trajectories.

Can anyone shed any light?

130.209.241.193 (talk) 16:31, 1 March 2010 (UTC)[reply]

No, that couldn't happen. A particle can only annihilate with its unique anti-particle. While charge does need to be conserved, it isn't the only conserved quantity. Things like lepton number would not be conserved in your example. (I don't think there are any elementary particles with charge +/- 2e, but something like a helium nucleus has charge +2e, or you may be thinking of an up quark with charge +2e/3.) --Tango (talk) 17:53, 1 March 2010 (UTC)[reply]
">2 particles" and "other than its own antiparticle" are separate issues. I don't know about the former, but for the latter, Zenone, Silvio Giovanni (1980), Antiproton Neutron Annihilation at Rest Into Kaon Antikaon-Pion Final States (Ph.D. thesis abstract), Syracuse University{{citation}}: CS1 maint: location missing publisher (link) is one of many such articles googling found me. DMacks (talk) 19:37, 1 March 2010 (UTC)[reply]
That sounds like an odd definition of "annihilation". If the final state includes fermions, it doesn't feel to me like the matter has been annihilated, just changed. --Tango (talk) 20:38, 1 March 2010 (UTC)[reply]
Yeah, this is beyond my field, I don't know how literal/technical the definition of "anhiliation" is in the field or anything about the (non)-presence of particles with mass in the products. DMacks (talk) 20:42, 1 March 2010 (UTC)[reply]
A Proton, an electron and a anti-neutron could have a triple anihilation into a neutrino plus photons. If fermions are not allowed in the final state than one of the three initial particles would have to be a boson. You could have for instance a neutron, a anti-proton and a W+ triple anihilate into photons.Dauto (talk) 01:28, 2 March 2010 (UTC)[reply]

Native American Height

My tribe is considered shorter than the average adult in the US (NW Canada). However I went to several pow wow's in the East Coast US and Eastern Canada and am still amazed how tall the native american people (men) were there. I'm talking over 6'3". Was what I saw unusual or is there an explanation for this? --Reticuli88 (talk) 16:41, 1 March 2010 (UTC)[reply]

Our human height has a link to details on the unusual height of Plain Indians [4] but not eastern ones. Rmhermen (talk) 20:25, 1 March 2010 (UTC)[reply]

Here are five possibilities: 1. The description of the tribe as short is old and obsolete because of improved health and nutrition. 2. The description of the tribe as short is old and obsolete because of continued intermarriage with people of European ancestry. 3. The description of the tribe as short was inaccurate even at the time. 4. Differential attendance at your gatherings-- tall people for some reason were more likely to be present. 5. Misperception: you noticed the tall people more, but they are atypical (or not tribe members) and the average is truly a bit short. Like any of these? By the way, the average adult male height in the US and Canada is close to 69.5 inches. alteripse (talk) 02:18, 2 March 2010 (UTC)[reply]

Re: The Seinfeld Junior Mints episode

What would be the likely effects (if any) of a Junior Mint getting left floating in between one's internal organs after an operation? Assume said mint is perfectly clean of any germs/bacteria.20.137.18.50 (talk) 17:46, 1 March 2010 (UTC)[reply]

Why would you assume that?? —Preceding unsigned comment added by 89.204.153.67 (talk) 19:00, 1 March 2010 (UTC)[reply]
To get past consequences I already know.20.137.18.50 (talk) 19:02, 1 March 2010 (UTC)[reply]
In that case, there would likely be a foreign body reaction within the connective tissue into which the mint settles. Macrophages would attack and fuse in an attempt to engulf it -- sort of like they did in Metchnikoff's seastar embryos. DRosenbach (Talk | Contribs) 20:04, 1 March 2010 (UTC)[reply]
Might it not dissolve before that could happen? It's a warm, wet environment - and those mints dissolve in your mouth in a short amount of time. Aside from the possibility of infection, I would kinda expect the sugars and starches to basically vanish. But if you just had an operation, surely the medical staff would be on the watch for possible infection and treat with antibiotics as soon as symptoms showed up. It's hard to be sure - but it seems possible that the answer might be "Not much". But it's tough to know for sure. SteveBaker (talk) 00:16, 2 March 2010 (UTC)[reply]
I stand corrected -- I did not know what a junior mint was, apparently. If you would drop some popcorn, let's say, what I said above would likely apply to that :) DRosenbach (Talk | Contribs) 00:39, 2 March 2010 (UTC)[reply]
But we have an entire article on the subject: Junior Mints - WHAAOE! - I'm horrified that you didn't know that! :-) SteveBaker (talk) 02:33, 2 March 2010 (UTC)[reply]

the article on Infiltration (medical) may be of some assistance. I know when donating blood, occasionally the needle with slip out of the vein, and blood will pool under the skin. (At least, for me - even then, it's totally safe.) This causes an ugly bruise, but my understanding is that this infiltration of blood is slowly absorbed back into the body. It sounds reasonable that the same thing would happen to a Junior Mint. (disclaimer:not a doctor} -Avicennasis @ 19:37, 3 March 2010 (UTC)[reply]

Citric acid

is it true that Citric acid is converted by the body into baking soda? —Preceding unsigned comment added by 67.246.254.35 (talk) 18:30, 1 March 2010 (UTC)[reply]

No. Citric acid is primarily relevant to the citric acid cycle, which has nothing to do with baking soda. Note also that citric acid and baking soda react energetically; an attempt to transmute one into the other would just cause a reaction into their byproducts. — Lomn 19:42, 1 March 2010 (UTC)[reply]
I'm wondering - did you confuse bicarbonate with baking soda? The acidosis article says that decreased bicarbonate production can lead to acidosis - it's used to balance the pH of the body. I won't pretend to understand it - I just found this by wiki-surfing. Vimescarrot (talk) 21:31, 1 March 2010 (UTC)[reply]

why is it okay for people with acidosis to have fresh lemons then? —Preceding unsigned comment added by 67.246.254.35 (talk) 21:19, 1 March 2010 (UTC)[reply]

Does anyone have a pointer to a scientific (i.e., not food quackery or Edgar Cayce hooey) discussion of the alleged alkalizing effect of lemons? --jpgordon::==( o ) 15:14, 2 March 2010 (UTC)[reply]
The low pH lemon is broken down in the stomach so it has no direct contact with the blood, so it wouldn't affect anyone with acidosis. In any case, if it magically did, the blood is buffered and can withstand pretty significant changes in pH. Regards, --—Cyclonenim | Chat  17:22, 2 March 2010 (UTC)[reply]

Semantic web

What, if anything, does the term "semantic web" even mean? —Preceding unsigned comment added by 89.204.153.98 (talk) 18:46, 1 March 2010 (UTC)[reply]

It's an idea that the meaning of the contents of a web page can be encoded in a way that specially written, hypothetical computer software is supposed to be able to understand. See the Semantic Web article, or wait for SteveBaker's long post which is sure to come below. Comet Tuttle (talk) 19:09, 1 March 2010 (UTC)[reply]
Jeez - talk about being taken for granted!
I happen to be a fan of this idea. Web pages are written for humans to read. That's OK as far as it goes - but it makes it exceedingly difficult for computers to understand them. The idea of the semantic web is to encode the information in a web page into a rigid, logic-based language that states things unambiguously for the sake of computers. (I rather like Cyc - but there are many others). If computers could read that data, they could fairly easily turn it into English for us (perhaps not very beautiful English...but definitely understandable). If we had that then search engines could be instructed to produce the exact right results - and it would be easily possible to have your computer read all of the pages that were found and return a summary of all of that information for you. The idea that any computer could surf the web and look for answers would enable all sorts of possibilities that would seem a lot like Artificial Intelligence. SteveBaker (talk) 00:11, 2 March 2010 (UTC)[reply]
On the other hand if we had real AI none of that would be necessary or desirable. Dauto (talk) 03:48, 2 March 2010 (UTC)[reply]
That's true - but we're really no closer to getting true AI than we were when research on this was started in the late 1960's. Besides, giving an AI system a huge stock of "common knowledge" is an important part of getting it to work - a very intelligent computer that didn't know that water is wet or any of a gazillion other common knowledge kinds of thing would appear very stupid to us. You can argue that an intelligent being could deduce this from reading the Wikipedia article on Water - but the act of reading really requires that a large pile of common sense be already in place. But no matter what, AI is hard - and writing software that traverses a semantic web is easy. SteveBaker (talk) 20:04, 2 March 2010 (UTC)[reply]
The first impression I got of the semantic web idea is that it's a system where humans work twice as hard in order to take a small burden off machines. That's not how we usually do things, we usually have the opposite aim. Perhaps it's good for some parts of the web, though. I guess it would be useful for sites which already tag things in ways which could be standardized. Seems less useful for, say, a personal homepage: oh joy, extra coding. 81.131.42.83 (talk) 11:10, 2 March 2010 (UTC)[reply]
What????? Humans put in immense effort to make the damned machines bend to their will. Ask any programmer. Comet Tuttle (talk) 17:33, 3 March 2010 (UTC)[reply]
The problem is that computers really can't do this for themselves...but if we do it for them, we'll get back many times more than we put in. (Or so the theory says!) SteveBaker (talk) 20:04, 2 March 2010 (UTC)[reply]
It's not really as dumb as all that. The idea is to save the data you already have so that the computer doesn't have to try to infer it later (which computers are too dumb to do (computer scientists, actually)).
Here's a simple example that affected me recently: I enjoy the local college radio station and sometimes in my car I would like to know what song is playing. I wanted to write a little app for my Google phone that would go out to the station's website, grab their playlist, and display it on my phone. Now whoever wrote the program that generates their playlist could easily have given it semantic qualities like this:
<span class="song_name">Small Change</span> by <span class="artist_name">Tom Waits</span>
They already had the name of the song and singer right there, but instead chose to do something very unsemantic like this:
<b>Small Change</b> by <i>Tom Waits</i>
So now instead of just looking for things tagged with "song_name", my program has to go through the page and look for "the first bold characters in each row of the third table but skip the first row". When they change the visual formatting of their playlist page in any way it is likely to break my program. It's stupid, and making the page more semantic would not have caused them any effort at all. Obviously the "semantic web" where everything is tagged is another step, but just making things a little smarter isn't hard. --Sean 20:48, 2 March 2010 (UTC)[reply]
Yes, because the inventors of HTML turned their back on one of the fundamental ideas of SGML that they based it on, and conflated the structural and presentation properties in a single system. --ColinFine (talk) 21:37, 2 March 2010 (UTC)[reply]
The Semantic Web idea envisions a Paradigm shift in how we humans get access via our machines to data sources on the web. It does not presume to create data of itself nor to indulge in spontaneous creative art. When the uncontained enthusiasm for the idea blows over, as it must, the view of a sceptic is that this immutable mantra shall prevail and that we can't expect more than this amount of advance towards Utopia. A sceptic can be wrong too. Cuddlyable3 (talk) 22:42, 2 March 2010 (UTC)[reply]
For the record, I wasn't taking SteveBaker for granted; I was forecasting a force of nature that was as inevitable as the Sun rising tomorrow. Oh, maybe that was taking it for granted. Comet Tuttle (talk) 01:11, 4 March 2010 (UTC)[reply]

LCD Clocks and Plasma Balls

About a decade ago I used to own a plasma ball. I had it next to my bed next to an old fashioned LCD alarm clock. One of the black and grey displays. When I turned on the plasma ball the LCD alarm clock would go wild. Segments of the figures would flash on and off in a seemingly random fashion. I assume it was due to some kind of electromagnetic interference. I was wondering what exactly. What happens in the plasma ball, and what process in the LCD alarm clock does it disturb, and why? •• Fly by Night (talk) 20:55, 1 March 2010 (UTC)[reply]

There are some really high voltages and high frequencies present in those balls (35 kHz, 2–5 kV) - and the arcing inside is producing frequencies that are all over the dial. It doesn't surprise me that it would upset your clock. Two things might help - one is obvious - which is to move the clock as far away as possible from the ball - whatever electromagnetic junk the ball puts out will decrease as the square of the distance - so twice as far away means a quarter the amount of interference. If they are an inch apart now then putting them a foot apart reduces the noise by a factor of 144:1. Secondly, it's possible that the interference is travelling through the wall socket from one to the other. If you could plug one of them into a different outlet, then that might fix the problem.
The reason for the problem in this case is almost certainly that your alarm clock also has a radio in it - that means it has an antenna that's tuned to pick up radio frequencies. Once those signals get inside the box, they can cause 'crosstalk' in nearby wires. Those random spikes can look like digital data to a computer or other digital electronics could certainly crash the little computer inside by messing up the instructions it reads out of it's memory. But there are any number of other detailed mechanisms that could cause this kind of thing and it's really impossible to figure it out.
SteveBaker (talk) 00:03, 2 March 2010 (UTC)[reply]
It was a battery operated alarm clock, and it didn't have a radio. It was a little one that just beeped when it was time to get up. The globe's electromagnetic field was getting inside the LCD and causing segments to misfire. I guess that the segments of the figures are activated by an electrical current. So somehow the electromagnetic field was turning into electricity inside the LCD and making it misfire. •• Fly by Night (talk) 12:54, 2 March 2010 (UTC)[reply]
Yes, the segments are controlled by voltage (rather than by current), and the circuit is matrixed, so you will get apparently random effects from varying electromagnetically induced low voltages on just a few elements of the matrix. It would be interesting to see whether you could reproduce the effect with static electricity - amber (or just a plastic comb) rubbed on dry wool. Dbfirs 19:12, 2 March 2010 (UTC)[reply]
As SteveBaker said, plasma balls generate high voltage, low current fields. This extends into any nearby wiring. LCD devices (especially 7-segment displays), use high voltages (around 100V AC, IIRC) at low currents to affect the twist of the crystals in the display. The plasma ball might have been inducing a high enough voltage in the display to cause it to twist. CS Miller (talk) 19:33, 2 March 2010 (UTC)[reply]

Line between species

How hard set are the lines between species? I imagine at some point maybe 1 out of 10 matings between the 2 branching "species" would produce fertile offspring and then 1 out of 100 and so on. But is there really a hard limit? So for example if a woman and a horse (gross I know but I'm not sure how else to phrase this)...is there zero possibility of producing offspring (let alone fertile) or is it more like a 1 in 10,000 chance? TheFutureAwaits (talk) 21:09, 1 March 2010 (UTC)[reply]

It is very poorly defined. The existence of ring species really complicates matters. The fact that there are even interfamilial hybrids shows that our taxonomic system doesn't really work. It's a decent approximation for most purposes, though. --Tango (talk) 21:18, 1 March 2010 (UTC)[reply]
A human and a horse, zero probability -- the genomes are way too different. A human and a chimpanzee might be possible, just as it is possible for a horse and a donkey to produce offspring, although not fertile. Looie496 (talk) 21:35, 1 March 2010 (UTC)[reply]
You are not going to get cross-breeding between different species that are radically genetically different. Horses and humans belong to entirely different orders—they have little in common other than the fact that they are mammals. Even amongst animals much more closely related to humans (e.g. orangutans, gorillas, chimps), the possibility of hybridization is thought to be basically nonexistent. For some long-standing speculation on chimp-human hybrids, see humanzee.
All of this is a little tangential to asking about the species concept. The species concept is necessarily fuzzy and not set in stone. But that fact does not mean that the genomes of radically diverged lineage are compatible. --Mr.98 (talk) 22:30, 1 March 2010 (UTC)[reply]
Wikipedia has articles on Liger and Tiglon that both have parents of the same genus but different species. Cuddlyable3 (talk) 19:50, 2 March 2010 (UTC)[reply]
There are lots of examples of hybrids of different species in the same genus. It is those in different genera that are particularly weird. It shows our definitions of genera are very arbitrary and don't mean the same thing in different parts of the animal kingdom. --Tango (talk) 20:14, 2 March 2010 (UTC)[reply]
Why do we restrict ourselves to the animal kingdom? Hybrids are frequently observed in the plant kingdom. --Kvasir (talk) 23:56, 3 March 2010 (UTC)[reply]

water

which way does water polarize the e-field of light, paralell or perpendicular to the plane of the surface? —Preceding unsigned comment added by 129.67.118.36 (talk) 21:16, 1 March 2010 (UTC)[reply]

Parallel. See Brewster's angle, although you may need to read it several times to decide which particular "planes" and "normals" the article is discussing. :) The diagram is reasonably clear, though. Tevildo (talk) 22:03, 1 March 2010 (UTC)[reply]

blatter infections and pregnancy

would blatter infections cause miscarriages in women by chance? ive been wondering bout that for some time... —Preceding unsigned comment added by Jwking (talkcontribs) 21:53, 1 March 2010 (UTC)[reply]

Do you mean "bladder"? As in a urinary tract infection? I think any sufficiently serious illness in the mother can cause a miscarriage (basically, her body decides it needs to worry about itself rather than a baby) and UTIs can get very serious, so I'd say "yes". --Tango (talk) 22:15, 1 March 2010 (UTC)[reply]
Yes yes yes. Bladder infections in pregnancy are serious and require immediate medical attention because left untreated they can result in premature delivery of your baby or miscarriage. If this applies to you, seek medical advice. Regards, --—Cyclonenim | Chat  17:25, 2 March 2010 (UTC)[reply]

Colder after chewing gum

Why is it whenever a person is chewing gum, mint flavored, then drinks a cold drink or cold water, is the cold sensation in your mouth so extreme? Sometimes you get an instant brain freeze effect. Can anyone shed some scientific light on this? —Preceding unsigned comment added by Zerojjc (talkcontribs) 21:57, 1 March 2010 (UTC)[reply]

See Menthol#Biological properties. --Tango (talk) 22:07, 1 March 2010 (UTC)[reply]
For the same reason why hot peppers taste hot, menthol tastes cold. They react with the same receptors, respectively. DRosenbach (Talk | Contribs) 00:41, 2 March 2010 (UTC)[reply]
And if one eats menthol with peppes it will feel both hot and cold at the same time!! Dauto (talk) 03:17, 2 March 2010 (UTC)[reply]

Malitol

What is Maltitol's history? Is it naturally occurring, like sorbitol?174.3.99.176 (talk) 22:15, 1 March 2010 (UTC)[reply]

Reading the first few lines of the article told me that it comes from starch. Beach drifter (talk) 22:17, 1 March 2010 (UTC)[reply]
Sorbitol certainly isn't "naturally occurring" - our article says: "It is obtained by reduction of glucose changing the aldehyde group to an additional hydroxyl group." SteveBaker (talk) 23:36, 1 March 2010 (UTC)[reply]
To be picky, that doesn't say that sorbitol is not naturally occurring, only that it is produced synthetically as well (or, could be, exclusively, but it doesn't say that). --Trovatore (talk) 23:43, 1 March 2010 (UTC)[reply]

Exorcists after "the Exorcist"

It seems to me that there is a regurgitation of exorcism in the Cristianity; particularly, in the Catholic Church. I had a glance to some film reports of exorcisms on youtube, both home-made and from TV programs, and I've got the impression that today's exorcism rite owe a lot to the movie the Exorcist. It seems to me that today priests and possessed people tend to behave and speak like the characters of the movie (as far as they can, of course). Is there an evidence of the influence of the movie on the behaviour of these people? --pma 22:28, 1 March 2010 (UTC)[reply]

It may be the other way around. I have heard it said that The Exorcist was a very accurate depiction of how exorcisms are handled. I couldn't find any reference to that in the article on the film, though. --Trovatore (talk) 01:49, 2 March 2010 (UTC)[reply]
Wikipedia has an article on Exorcism. The movie The Exorcist was the 2nd most popular film in 1974, has been the source of some urban legends, see the linked article that significantly notes that its critical reception has grown over the years. That is evidence of its increasing influence. The Catholic Church does not endorse its representation of exorcism any more than it endorses the conclusions of a more recent movie The Da Vinci Code. Cuddlyable3 (talk) 19:43, 2 March 2010 (UTC)[reply]
Thank you both for your answers. Well, it seems that the Church attitude towards The Exorcist is quite another story from The Da Vinci Code, whose content is evidently heretical. I see now we have an article on Gabriele Amorth, which is a kind of president of the official association of exorcists of the Catholic Church; it reports he appreciated the movie as accurate, although a bit exagerated (I'm glad to know it is, of course), but substantially confirming what Trovatore sais. But also, this suggests that there could be a feedback: that is, that the movie then influenced the scenography of the rite and increased its popularity. After all, if something inside you tells you that you should feel possessed (a mental disorder, a big social embarass, or the diable himself, or whatever) you may feel encouraged to let yourself go, if you have already learnt how to behave properly. --pma 23:07, 2 March 2010 (UTC)[reply]

TV color

In the USA colors seem to be a combination of red, blue and green at varying intensities to derive the combined color. Are there other countries that use a different scheme that if recorded and played back on TV's in the USA will show a blue exchanged with red? 71.100.5.197 (talk) 22:29, 1 March 2010 (UTC)[reply]

(Removed long discussion about the OP's questioning habits to the discussion page). SteveBaker (talk) 12:52, 2 March 2010 (UTC)[reply]
Well, no, not exactly. NTSC (what the USA uses for TV) does do color a bit differently than, say, PAL, does, and sometimes you do need to fiddle with the tint on NTSC. One presumes that playing a PAL on an NTSC or vice-versa could result in some color shift. But it should not result in colors simply exchanging, I don't thnk. --Mr.98 (talk) 22:33, 1 March 2010 (UTC)[reply]
All TV's, computer monitors, and color displays of all kinds use Red, Green and Blue. The main reason is that this is how our eyes perceive color - we have 'sensors' in our eyes that only pick up those three colors. Hence, this is the smallest number of colors (and the exact perfect ones) to trick us into thinking that we're seeing all colors possible. However, there are other animals (such as the humble goldfish) that can see in more than three colors. This would presumably mean that a goldfish would think that watching color TV would be somewhat like watching just a red/green display would be for us. Other animals (such as bees) see in parts of the spectrum where we can't see at all. Bees do a good job of seeing in the ultra-violet waveband. There are actually a very few humans who are tetrachromats who see in four colors. One of them who has been studied by scientists is an elderly lady in the UK - and she has reported that she finds color TV pictures somewhat lacking in full color...which is what we'd expect.
The reason there is a need to 'fiddle' with the tint on NTSC TV's is that NTSC misses a rather clever trick that PAL TV's use to keep the color looking good. With NTSC, the color is decoded and displayed very directly - but if the decoding circuit gets a bit out of sync, it has to be manually adjusted to get the color right again. PAL rather cleverly encodes the colors with the opposite signal on alternate lines of the image (PAL stands for "Phase Alternate Line encoding" - or something like that). That means that if one line of the image 'drifts' (say) towards green then the alternate lines either side of it must drift away from green by the exact same amount. Because our eyes don't perceive color at high detail, this 'alternation' blurs together to produce the exact right color in our brains - provided it hasn't drifted too far from the right values. That's pretty clever - and it's why a lot of Brit's call NTSC "Never Twice Same Color".
SteveBaker (talk) 23:49, 1 March 2010 (UTC)[reply]
Thanks for that explanation for the difference in NTSC and PAL. However, the problem I have encountered is a converter from VGA that switches red and blue regardless of setting to NTSC or PAL for output to a standard NTSC TV and to a S-Video/RCA composite input video card on a personal computer. The seller says it converts fine on his TV but for sure it does not convert fine on the buyers TV. What could be the explanation other than defective converter? 71.100.5.197 (talk) 00:00, 2 March 2010 (UTC)[reply]
What are you using when you connect to the TV? Also when you are testing with the video in card, When you say s-video/composite, do you mean you've tried to connect both S-video and composite? Are there any controls other then switching output from NTSC & PAL? Nil Einne (talk) 00:17, 2 March 2010 (UTC)[reply]
The converter has 2 binary switches, a VGA input/output/bypass connector, and S-Video output connector and an RCA output connector. With both switches set to off you are supposed to get PAL-B/B/G/H/I. With switch #1 on and switch #2 off you are suppose to get NTSC and with switch #2 on (regardless of switch #1 setting) you are suppose to get RGB out. I assume this applies equally to both the S-Video and the RCA output connectors. Regardless of connector setup to TV or to S-Video or to RCA on computer input card testing each unit and each possible configuration and connection in turn you still get blue and red colors reversed. 71.100.5.197 (talk) 00:46, 2 March 2010 (UTC)[reply]
I assume from this that the connections (wiring) are intact but that the switches might be inoperative. In any case my conclusion is that the unit(s) are defective. 71.100.5.197 (talk) 01:48, 2 March 2010 (UTC)[reply]
Do you have anything to try the RGB with? Also if there are multiple VGA connectors, have you tried plugging it in to one of the other ones in case you have the wrong connector? And the final obvious thought, have you tried fooling around with switch 2 and in particular, using it on the other setting in case you mixed it up? Nil Einne (talk) 02:18, 2 March 2010 (UTC)[reply]
RGB monitors such as the Amigo are hard to find now-a-days. I do have an Atari monitor but it would take some sync circuit interfacing to tap its RGB lines. Actually RGB is a technology that would have to use the S-Video output connector since it has horizontal and vertical sync lines in addition to the RGB analog lines. The original Radio Shack Color Computer may have used an RGB monitor but the RS Color Computer II I still have has only an RCA composite output connector for use with an antenna box to hook its up to the VHF or UHF TV antenna connector for use with channel 3 or 4. This box is what I'm now using to go from the RCA connector output to the TV. The connection to the computer RCA input connector, however, does not require this antenna box.Regardless of switch settings of which there are only three the red and blue revered color does not change. 71.100.5.197 (talk) 03:27, 2 March 2010 (UTC)[reply]
Do you actually know what sort of RGB connection the device outputs? Are you sure it isn't RGsB? Incidentally I believe projectors often use RGBHV. Also are you saying that whatever of the 4 possible settings (switch 1 on and off, switch 2 on and off) you set it, the output works but red and blue is reverse? Below however you appear to say it doesn't work however when set to NTSC and connected to a computer Nil Einne (talk) 03:56, 2 March 2010 (UTC)[reply]
The human eye is sensitive to all wavelengths from red through violet, not just to three specific wavelengths. Because there are three cone types, three primaries suffice to reproduce any hue. Many sets of three primaries will work. The choice of primaries is driven by practical considerations like the availability of cheap phosphors/filters/LEDs and compatibility with established standards based on certain primaries. There are (or at least were) some displays that used red-green-violet, orange-green-violet, and the like, and there's a lot of variation in the "red", "green" and "blue" used in RGB displays.
The NTSC signal uses a color space called YIQ. Y (brightness) is encoded independently, while I and Q (color) are encoded together as a signal whose phase is the hue (angle in the I-Q plane) and whose amplitude is the saturation (distance from the origin). The famous tint problem comes from this signal being out of phase. It's the TV's job to convert this into phosphor intensities, and in principle the TV can use whatever phosphors it likes as long as it does the conversion correctly. Though I guess that none of this ends up being relevant to the OP's question. -- BenRG (talk) 02:22, 2 March 2010 (UTC)[reply]
Its probably relevant in that the NTSC setting does not work either regardless of connection or switch setting at least with the converter unit whereas it works fine with all other devices I have connected to the computer that use NTSC correctly. 71.100.5.197 (talk) 03:33, 2 March 2010 (UTC)[reply]
So wait, if I understand you correctly, when you turn it to NTSC it doesn't work with your computer at all? Does it work with your TV? When you turn it to PAL it works? Does your computer identify the device as PAL? Nil Einne (talk) 03:56, 2 March 2010 (UTC)[reply]
On any setting the picture is still displayed but the red and blue colors are reversed. On the computer I adjusted the resolution from lowest and highest the video card and monitor would support. My question above was to determine if the converter might require a codec that switched blue and red color back to normal or if there was some other difference in signal say like what might be used in Russia which was incompatible with Standard US TV. Now I just think the converter is defective. 71.100.5.197 (talk) 04:50, 2 March 2010 (UTC)[reply]
If the switches don't do anything, then yes, it sounds rather dubious. I still don't understand what you mean by 'NTSC setting does not work either regardless of connection or switch setting at least with the converter unit whereas it works fine with all other devices I have connected to the computer that use NTSC correctly' since the switches don't do anything. In any case, it doesn't matter now I guess given your upcoming major life change. Do remember to disclose that this device is probably defective when you sell it, as you've said it's the moral thing to do. Nil Einne (talk) 10:11, 2 March 2010 (UTC)[reply]
I remember now DECO isn't it? Anyway I thought we were friends there for the longest time and now I realize it was you who went through some kind of life change... Anyway... to clarify. No matter whether the switches are set to produce NTSC, RGB or PAL the blue and red colors are reversed. i.e., the sky is red instead of blue. The sunset is blue instead of red. A field with a morning haze of light blue is a morning field of light red. Got it? Anyway... it also does not matter what resolution the computer viewing monitor is set to or which TV set is used so the only explanation I can come up with is that the device is defective accidentally such as the switches not working or temporarily defective by intent on the part of the seller to allow the seller to engage in the Defective Item Return Scam. I can't sell the device because I have already returned it to the seller who declared that it was not defective but working and together with the excess postage fee he did not refund and the 15% restocking fee he not only got a "working" item back to sell once again but a 30% profit. 71.100.5.197 (talk) 20:49, 2 March 2010 (UTC)[reply]
For clarification, as I said in my earlier response I've never gone by the name DECO, or any other name. Maybe you're confusing me with User:Jiuguang Wang, User:Dcoetzee (who remains an active wikipedian) or someone else you've had a run in with one of you numerous previous accounts. Anyway back to the original question, if the switches didn't do anything at all even change the output to PAL instead of NTSC as they are supposed to, this is something I would have emphasises to the sellet and it's clearly a flaw whatever they use in Russia, but I guess that's too late now. And you still haven't explained what 'NTSC setting does not work either regardless of connection or switch setting at least with the converter unit whereas it works fine with all other devices I have connected to the computer that use NTSC correctly' means since according to you now the switches don't do anything so there's no such thing as a NTSC setting but only one setting (since the buttons don't do anything) so the device either works all the time when outputting to a device like a computer with video in, or never works at all. And it's not clear to me from your responses whether the device is really outputting NTSC. It may be that the device is permanently on the RGB setting and however the device works when it does that, it reverses the colour when outputting to composite and s-video on that setting. Either way as I've already said, if the switches aren't working at all, that would be a clear cut sign the device is broken, and compatibility issues aren't relevant. Nil Einne (talk) 08:54, 3 March 2010 (UTC)[reply]


March 2

Solving a simplified 1-D time-independent Schrodinger-esque equation

I've been trying to solve the following equation numerically using a Python program that I've written. It works reasonably well, but not perfectly. I'm under the impression that it can be solved exactly using analytical methods and I was wondering if anybody would be able to point me in the right direction. Unfortunately, my maths and physics knowledge is very limited, so I can't guarantee I'll have a clue what you are talking about.
, where m = mass of electron, h = Planck's constant, E = energy of first level in hydrogen atom, e = charge on an electron, = permittivity of free space, R = distance between nucleus and electron, A = amplitude of resulting wavefunction. Thanks --80.229.152.246 (talk) 00:04, 2 March 2010 (UTC)[reply]

See: Hydrogen_atom#Wavefunction, specifically the n = 1, j = 0, m = 0 case. Dragons flight (talk) 00:48, 2 March 2010 (UTC)[reply]
Yeah, I've looked at stuff like that before and I'm afraid it's slightly above my head. Any suggestions as to where I should start to try and learn some of the stuff? --80.229.152.246 (talk) 15:32, 3 March 2010 (UTC)[reply]

Long legged wasps

Over the last 15 years I've noticed the normal wasp being replaced by a longer legged more aggressive variety in the Pyrénées-Orientales in France. I can't seem to find it in the list of common wasp species on the wasp article, does anyone know what this wasp is? Is it another species or just a variant, it seems pretty different. As far as I can tell it's a paper wasp but the article says that they're less aggressive, not more, and also doesn't mention the fact that the yellow jackets are becoming less common compared to the paper wasps. 82.132.136.206 (talk) 00:31, 2 March 2010 (UTC)[reply]

Coreinae - Insect families/tribes

This page lists 10 tribes, Wikispecies lists a lot more than 10 tribes on it's Coreinae page ... which is correct? Should the extras from wikispecies be in the wikipedia article aswell? I'm asking because I was about to make a stub for Amorbini... Thanks. Benjamint 01:36, 2 March 2010 (UTC)[reply]

Coreinae page gives the tribes list from ITIS 2006 record, here. You can use it. However, those things tend to change every once in a while, as more studies are carried out; so don't be surprised if the tribe list changes in a couple of years. --Dr Dima (talk) 06:41, 2 March 2010 (UTC)[reply]
OK, so wikispecies is more up to date? Should I amend the wp page with the wikispecies list? I think I'm out of my depth so maybe I should just leave it. Benjamint 08:15, 2 March 2010 (UTC)[reply]
If the Wikispecies list is reliably sourced, by all means bring the WP page up to it; but you can't use Wikispecies as a reference in WP. --ColinFine (talk) 21:42, 2 March 2010 (UTC)[reply]

Speckled Bear vs. Spectacled Bear

Are the Speckled Bear and Spectacled Bear different species?

Searches on Google yield conflicting results; Wikipedia doesn't, have an article, or any reference to, a "Speckled Bear." Each name is used individually on multiple websites; but I haven't any one source that acknowledges the use of two similar, yet distinct names... let alone the resulting confusion/misinformation. Results from Google image searches are different enough that they could be of separate species, but similar enough that they could simply represent variation within one species. Aside from Wikipedia's "Spectacled Bear" article, none of the sources I've encountered offer much in the way of verification/credibility, so I figured I'd ask for feedback here.

Ajburket (talk) 02:21, 2 March 2010 (UTC)[reply]

There is no speckled bear. See our Bear article which lists all eight surviving species. Rmhermen (talk) 02:46, 2 March 2010 (UTC)[reply]
There are several videos[5] [6] [7] at YouTube with "speckled bear" in their titles. This seems to be just a misnaming of Spectacled bears by zoo visitors. Cuddlyable3 (talk) 19:11, 2 March 2010 (UTC)[reply]

magnetic field

Would a magnetic field generated around a space ship or space station protect it from radiation, particles or anything else or just make radio transmission more difficult. 71.100.5.197 (talk) 02:49, 2 March 2010 (UTC)[reply]

Yes, it is one option that has been considered. Here is one reference that discusses it: [8]. --Tango (talk) 03:57, 2 March 2010 (UTC)[reply]
The Maxwell equations are linear, therefore a static magnetic field does not make radio transmission more difficult. —Preceding unsigned comment added by 83.134.160.138 (talk) 06:41, 2 March 2010 (UTC)[reply]
It would protect the ship from alpha particles, beta particles and proton radiation, but not from neutrons or gamma rays. Clear skies to you 24.23.197.43 (talk) 07:04, 2 March 2010 (UTC)[reply]
Cosmic rays are almost all charged particles, so we're ok. --Tango (talk) 07:06, 2 March 2010 (UTC)[reply]
A magnetic field wouldn't have much affect on large particles unless thay were ferrous would it? And then wouldn't it tend to attract them? (I'm not thinking of sub-atomic here as above, tell me if I'm wrong!) These may be of interest Scientists Designing "Ion Shield" To Protect Astronauts From Solar Windsourced from Sun shield to let space crews boldly go to Mars and Spacecraft Shielding 220.101.28.25 (talk) 17:06, 2 March 2010 (UTC)[reply]
Ferrousness (is that a word?) applies only to lattices of atoms, we're talking about individual particles. The key concept here is whether they are charged or not. A charged particle (electrons, protons and alpha particles, primarily) travelling through a magnetic field will be deflected. --Tango (talk) 18:22, 2 March 2010 (UTC)[reply]
Ferocity ? :-) StuRat (talk) 18:42, 3 March 2010 (UTC)[reply]
I wonder how powerful a magnetic field should be to deflect protons coming at relativistic speeds. The LHC article says dipole magnets at the field strength of 8.3 teslas keep 7 TeV proton beams in their orbit. As some cosmic particles are much more energetic than man-made ones, is it feasible to maintain such a field for long durations, in a small spaceship? 88.242.228.248 (talk) 17:56, 2 March 2010 (UTC)[reply]

I have a simulink model consisting of feedbacks. After a few iterations, simulink shows up a "cannot solve algebric loop" error for a block. I noticed that for that time instant (0.24), it had already found out converging values of the block (within my accuracy requirments). How do I modify its algebric loop solver configuration ? —Preceding unsigned comment added by 218.248.80.62 (talk) 08:01, 2 March 2010 (UTC)[reply]

I'm not familiar with that software, but I suspect it must have some setting for how many iterations to try before giving up. If not, you could manually copy the results from one run as the starting conditions for another run, and thus get a better result (assuming it is a converging case). Also, you might try the Math Desk. StuRat (talk) 18:39, 3 March 2010 (UTC)[reply]

plasma

does plasma take an indefinite article, (such as 'a lake') or not (such 'as water'). The article was inconclusive. —Preceding unsigned comment added by 129.67.116.170 (talk) 18:05, 2 March 2010 (UTC)[reply]

It depends on context. Is there a particular sentence you are wondering about? --Tango (talk) 18:25, 2 March 2010 (UTC)[reply]
... like soup! Dbfirs 19:02, 2 March 2010 (UTC)[reply]
Or, indeed, like "solid", "liquid", and "gas". ("The bottle contains 200 cc of liquid"; "The bottle contains a liquid") Probably a question that's better for the language desk; see mass noun. Tevildo (talk) 19:27, 2 March 2010 (UTC)[reply]
No it doesn't -- you can't say "a plasma", it would be wrong. The only exception is if you specify a particular type of plasma (e.g. you could say "a xenon plasma"). 24.23.197.43 (talk) 05:06, 3 March 2010 (UTC)[reply]
You (or at least I) might say "when heated, the material formed a plasma", or some other construction. I agree that it depends on the context, just as other states of matter do. Buddy431 (talk) 06:31, 3 March 2010 (UTC)[reply]
Yeah, 24.23.297.43 is wrong. There's nothing about plasma that forbids taking an indefinite article. In fact it's (much) easier to find correct, natural sentences that include a plasma than it is to find ones that include a water. But even a water is possible in the right context. --Trovatore (talk) 19:27, 3 March 2010 (UTC)[reply]

Male and female human voices

It's an easily observable, almost universal, fact that almost all female humans have higher-pitched voices than male humans. This is already noticeable with children, but even more so with adults. Most people accept this as natural and don't think twice about it. But what exactly causes it? JIP | Talk 20:04, 2 March 2010 (UTC)[reply]

Puberty#Voice change has some information about the mechanism of it. I don't know what, if any, evolutionary benefit it has. --Tango (talk) 20:16, 2 March 2010 (UTC)[reply]
Human voice may also be of interest to you. Vimescarrot (talk) 20:17, 2 March 2010 (UTC)[reply]
I imagine that sexual selection is at play here. Different pitches are a sexual characteristic magnified by puberty, I think. Imagine Reason (talk) 15:41, 3 March 2010 (UTC)[reply]

Super-fast spaceship

Wouldn't it be in practice impossible to have extremely fast space ships such as you'd need to get humans to other stars, because they would go so fast that the tiniest specks of space-dust would go through them like bullets and cause lots of damage? —Preceding unsigned comment added by 84.13.29.241 (talk) 21:41, 2 March 2010 (UTC)[reply]

Yes, relativistic effects can make the interstellar medium deadly. You'd need some sort of shielding mechanism to counteract it. — Lomn 21:50, 2 March 2010 (UTC)[reply]
Then there is also the practical problem of being able to accelerate your spacecraft at about 1g for a full year, and having a full year of fuel to decelerate once you arrive (well a year before). Googlemeister (talk) 21:55, 2 March 2010 (UTC)[reply]
There are possible methods. The acceleration can be done by pushing the ship along with a laser beam from Earth if you can't take enough fuel with you. There are ways of slowing a ship down the same way, but you would be so much further away that it would be hard - a laser could be sent in advance as part of an unmanned probe that can take decades to arrive without it being a problem. --Tango (talk) 00:09, 3 March 2010 (UTC)[reply]
The Bussard ramjet is an idea for using the interstellar medium as fusion fuel on your spaceship. In practice, "go slow" is our only recourse. --Sean 22:01, 2 March 2010 (UTC)[reply]
It is unclear if a Bussard ramjet could actually generate more thrust than drag. --Tango (talk) 00:09, 3 March 2010 (UTC)[reply]

Only as impossible as a deflector dish! TastyCakes (talk) 22:59, 2 March 2010 (UTC)[reply]

The shielding could be done, the real question is whether it can be done without adding so much weight that the ship is impossible to accelerate. --Tango (talk) 00:09, 3 March 2010 (UTC)[reply]
Here's my approach:
1) Start with a spaceship which is a massive linear particle accelerator, using matter/anti-matter engines with tanks full of anti-matter in a magnetic containment "bottle".
2) Go to the asteroid belt and clamp onto a huge asteroid.
3) The ship will then slowly mine the asteroid, which is positioned in front of the ship to act as a shield.
4) A particle stream (consisting of particles mined from the asteroid) zips out of the linear accelerator at the speed of light, driving the ship ("be careful where you point that thing !", the Sun might be a good choice).
5) When the ship nears it's destination, a hole is drilled through the asteroid, and the particle accelerator is reversed to send particles through the hole at the speed of light. Alternatively, the ship can be turned 180 degrees. The particle stream will act as a shield to prevent dust particles from getting to the ship during decel. There will be a short period of vulnerability while the reversal occurs. It may be possible to scan for approaching dust particles and time the reversal for a clear period, or they may just have to take their chances.
6) When the ship stops, discard whatever remains of the pitted and hollowed-out asteroid and pick up a new asteroid for the trip home (you might want to make sure the target system has asteroids before you set out). You should have a good 5 minute period before the kids return to asking "Are we there, yet ?".
Such a design might be able to do maybe 10% of the speed of light, so it might take some 45 years to reach the nearest stars. Unless you freeze the people, they likely would die during the return trip. So, I'd either have them stay there or do the whole thing robotically, and don't worry about a return trip. StuRat (talk) 15:49, 3 March 2010 (UTC)[reply]
That's all great except for the "matter/anti-matter engines" bit. We have absolutely no idea how to generate antimatter in such a way as to get positive net energy. Oh, and the dust is pretty constant (although very low density), so you aren't likely to find any gaps. If you put the sensitive stuff in the middle of the asteroid, it won't matter which way it is facing. I can't see how a particle stream would act as a shield, though, the density of the stream wouldn't be high enough to stand a good chance of intersecting much of the dust. --Tango (talk) 04:44, 4 March 2010 (UTC)[reply]
The antimatter wouldn't be generated on board, but would be created and placed on the ship prior to launch, similar to how rocket fuel is loaded now. The energy source used to produce the antimatter at the launch base might be a series of nuclear reactors (hopefully fusion). I like the idea of putting the ship completely inside the asteroid. I'm hopeful that the matter stream could be kept narrow and at a high density, at least a small distance ahead of the ship. Remember, it only has to protect the hole from which the matter stream flows, while the asteroid will protect the rest of the ship. StuRat (talk) 15:35, 4 March 2010 (UTC)[reply]

Are chickens intelligent enough to suffer?

This afternoon I was faced with the dilemma of buying six free-range eggs for £1 or ten battery-hen eggs for 78p. Are tiny-brained chickens intelligent enough to suffer in a battery cage, or realise that they are missing out on going free-range? Are they smart enough to have any conciousness at all, or have any sense of self? I did buy the free-range eggs. Thanks. 84.13.29.241 (talk) 22:02, 2 March 2010 (UTC)[reply]

There is, in principle, no way to know anything for sure about the qualia of chickens (or, for that matter, any being other than yourself). But to me it seems very likely that they are capable of suffering. I also prefer to buy cage-free eggs. --Trovatore (talk) 22:10, 2 March 2010 (UTC)[reply]
Unfortunately I think "cage-free" is likely to mean hens that spend all their lives in pernament darkness or almost-darkness in a large modern barn while packed in shoulder-to-shoulder with a sea of other chickens. 84.13.29.241 (talk) 22:18, 2 March 2010 (UTC)[reply]
Having raised chickens in my youth, being a free range chicken would not be all fun and games either. Chickens can be quite brutal to each other. Googlemeister (talk) 22:15, 2 March 2010 (UTC)[reply]
Though this may not answer your direct question, I'm sure you'd be interested to know that there is literally no (US) federally mandated standards with which a chicken/egg/ect. producer needs to meet in order to call their product "free range". In every meaningful sense it means nothing besides it being a fantastically effective marketing scheme. Chris M. (talk) 22:20, 2 March 2010 (UTC)[reply]
This is a very complex issue and my personal jury is still out on the subject. Unfortunately one of the main factors to consider is that "free range" is not very well regulated in the US or Australia, I'm not sure about the UK, but I'd be surprised if it was much different. This means that there is actually no "standard" that a producer has to follow to label their product "free range" and "some" free range labelled eggs come from very similar conditions to battery eggs, except maybe they have a "window" or something equally as superfluous. In Australia we have brands which are RSPCA certified free range eggs which I believe is well monitored and enforced but there are other eggs labelled "free range" which are not as enforced. That doesn't actually answer your question. Yes chickens are stupid but I just can't see how that gives us a right to exploit and abuse them to the fullest possible extent for our benefit. Chickens are easily the worst abused animals we farm, just google clips of battery hen condition, and don't do it with kids around, it's like a horror movie. Or maybe don't if you don't have a strong resolve and don't want to be a vegetarian. ;) Vespine (talk) 22:24, 2 March 2010 (UTC)[reply]
There are regulations for free-range in the EU, see Free-range eggs. Four square meters per hen for "free-range". ten square meters per hen for "organic free range". However the current photo in that article does not look like free-range to me, but barn-hens. The free-range commerical hens I've seen from the roadside here in the UK were similar to this http://www.youtube.com/watch?v=gS78WhX2ubY and could have been organic-free-range. 84.13.29.241 (talk) 22:37, 2 March 2010 (UTC)[reply]
Part of this requires a definition of suffering. They certainly feel pain. They certainly feel stress. They certainly have some type of consciousness, though they don't appear to be sentient. (I say "certainly" in these instances to mean, "within a reasonable physiological definition," not to indicate I know what chicken qualia is like.) They probably don't know they are "missing out" (but then again, a human raised exclusively in such conditions would probably not know they were "missing out" on a wide world of options, either). Is a sense of self required for suffering? --Mr.98 (talk) 22:29, 2 March 2010 (UTC)[reply]
I belive the term you're looking for is sapience (when you say a chicken doesn't have it). the sentience/sapience confusion is remarkably common. Chris M. (talk) 15:03, 3 March 2010 (UTC)[reply]
I would note here that sentient means "capable of feeling", not "capable of thinking". I don't know what you mean by saying they have consciousness but are not sentient. --Trovatore (talk) 00:52, 3 March 2010 (UTC)[reply]
Hmm, I meant in the sense used at Sentience: "the ability to feel or perceive subjectively". Apparently the term has a number of meanings depending on the field it is being used, though. The article itself brings up some useful points for thinking about it. --Mr.98 (talk) 00:57, 3 March 2010 (UTC)[reply]
If you don't have subjective experience, in what sense can you be said to be conscious? --Trovatore (talk) 01:11, 3 March 2010 (UTC)[reply]


I think you probably need a concept of emotion - do chickens feel happiness and sadness? I'm not sure we'll ever really know... --Tango (talk) 00:13, 3 March 2010 (UTC)[reply]
Here's an interesting quote from Cecil Adam's treatment[9] of the subject : "UK researchers studying commercial poultry farms say only 15 percent of chickens who have the opportunity ever leave the henhouse." APL (talk) 00:37, 3 March 2010 (UTC)[reply]
This video seems somewhat informative. Bus stop (talk) 00:54, 3 March 2010 (UTC)[reply]
Please understand that regulations and conventions are different in different parts of the world even in english speaking countries. The regulations for "free range" seem more hen-friendly in the EU than they are in the US - see above. 78.151.146.204 (talk) 01:09, 3 March 2010 (UTC)[reply]
Is opposition to suffering, as such, morally correct, and if so why? ...Is a question you might ask next. 213.122.45.10 (talk) 02:05, 3 March 2010 (UTC)[reply]

A chicken will undoubtably experience everything that you yourself would experience if you were put in its position. the only real difference between animals and humans (cognitively speaking) is that humans are capable of a high level of abstraction: You can sit in a cage and imagine all sorts of alternate situations (how you might escape, what you'll do if you get out, whether or not being stuck in a cage is justified, how you might pass the time each day). Chickens can't - they are simple stuck experiencing whatever it is they experience while stuck in the cage. I don't know if that's more misery or less misery, but I think you can judge by the fact that most chickens would run away from a cage if they could that it's probably not a lot of fun. --Ludwigs2 02:17, 3 March 2010 (UTC)[reply]

I've thought a lot about these sorts of issues, and in my opinion that only sensible approach is to look for signs of distress. Do the chickens fail to care for themselves, or peck at themselves, or make attempts at escaping, or squawk, or otherwise fail to thrive? I don't know the answer, but in my opinion any approach that isn't based on looking at behavior leads to answers that are purely based on preconceived beliefs. Looie496 (talk) 02:29, 3 March 2010 (UTC)[reply]
The assumption that brain size has anything to do with depth of feeling is a specious one. After all, Neanderthals had a much larger cranial capacity than extant humans. Did they have a richer experience of the world as a result? Who knows -- but I for one very much doubt it. SortedButter (talk) 03:09, 3 March 2010 (UTC)[reply]
Not sure what you mean by "depth" of feeling but, given that these feelings originate in the brain, I would expect that brain size is relevant. If a hypothetical animal had a tiny brain composed of 5 neurons, would you say that it is capable of suffering or feeling distressed at being locked up? Zain Ebrahim (talk) 08:10, 3 March 2010 (UTC)[reply]
Yes, but you cannot honestly say that a fly has a weak or negligible response to finding something sweet to eat, because it has only a tiny fraction of the 'brain cells' that a human has. Who knows how anything feels that is not you. SortedButter (talk) 13:34, 3 March 2010 (UTC)[reply]
  • The field of animal cognition is a very interesting one and still a matter of considerable debate. It used to be that animals were considered automatons with no emotions, which reacted to stimuli. Some older scientists still hold onto this, but the depth of animal behaviour is slowly being explored. Alex (parrot) is a good place to start (although the article doesn't really examine the impact he had on the science in general... something I may have to rectify). While crows, parrots and tits are reckoned to be the brightest of birds, that doesn't mean that dumber birds are less capable of complexity and feeling. The extent to which humans can relate to the moods and lives of chickens is evident in how many chicken behaviours have influenced our own langauge. Sabine's Sunbird talk 03:43, 3 March 2010 (UTC)[reply]
You might also ask yourself whether you actually want to eat an egg that came from a chicken covered in feces, which breathes fecal dust its whole life, which must be fed antibiotics due to all the feces in its life, which in many cases is sharing a cage with a partially decayed hen that is left to rot through the wire floor, and on and on. I personally don't buy eggs unless they came from chickens I can actually see (friends with hens, etc.). The alternative is too repugnant, and not just morally. --Sean 16:38, 3 March 2010 (UTC)[reply]
In my youth we had hens who slept in a large henhouse, but ran around in a large fenced enclosure during the day, free to scratch for bugs, and free to flap their wings and fly a bit. Some of the birds could fly out and explore the woods, but came back for dinner. A pigeon also moved in and stayed for a couple of years, so that "free-range" setup was clearly preferable to some birds than "unlimited freedom." I cannot honestly say the eggs tasted any different from those I buy at the store which are "cage free" or eggs produced in typical abusive commercial operations. I just prefer to pay several times as much for the cage free ones. I certainly believe that birds can be happy or unhappy, calm or distressed, and that they can suffer. Edison (talk) 19:47, 3 March 2010 (UTC)[reply]
I have no direct experience, but from what I've been reading it seems that what is called "cage free" in the US is not the same as what we'd call "free range" in the UK. I fear that "cage free" in the US may simply mean that the chickens are kept all their lives in darkness in an enclosed barn packed with a huge crowd of other chickens, possibly de-beaked too. Nearly as bad as being in a cage. 89.243.73.49 (talk) 22:50, 3 March 2010 (UTC)[reply]

thermoelectric generators

It would seem like the relatively extreme difference in temperature of an object's surface that faces the sun and the surface that faces the stellar background would be ideal for generating electricity both from thermoelectric generators and closed system liquid/gas phase generators. Are they ever used in space or are solar panels that conver solar radiation the only choice? 71.100.5.197 (talk) 23:41, 2 March 2010 (UTC)[reply]

Solar heat is used on Earth (a rather extreme example). In space it may be used as well, but it isn't used because we don't have anyone that far out in space. I do not believe it is possible, at this time, to make an educated claim about what exactly will be used when manned spacecraft go well beyond the Moon and need plenty of methods of generating electricity and heat. -- kainaw 23:46, 2 March 2010 (UTC)[reply]
Actually I was thinking on a much smaller scale that the terrestrial based systems your link points to. I was thinking on the order of solar panels used to power the International Space Station for instance. I'm asking because such terrestrial systems used to heat and cool homes are far more limited in terms of efficiency due to the slight temperature differences between hot and cold whereas in space the very opposite is true. 71.100.5.197 (talk) 01:05, 3 March 2010 (UTC)[reply]
I would think that the issue is one of relative efficiency in terms of power generated per unit area and, more importantly, power generated per unit mass. I suspect that photovoltaics (given how thin they can be) come out on top over solar thermoelectric generation, but I don't know the numbers. 124.157.249.27 (talk) 01:03, 3 March 2010 (UTC)[reply]
For craft in a low orbit around a major body like a planet or a moon, the assertion that the plate would see the heat of the sun on one side and the cold of space on the other only applies for a small fraction of their orbit. When they are behind the body - then they are seeing only the temperature of the dark side of the planet on one side of the plate - and the temperature of space on the other. When they are between the planet and the sun, they'll see the temperature of the sun on one side and the temperature of the sun multiplied by the albedo of the planet on the other. That's much less effective than the ideal case. So it seems to me that this idea might be more useful for craft that either have very high orbits or are out in deeper space - than for the ISS - which orbits at only 300km or so above the earth and is only rarely in that ideal situation. Deep space missions tend to use radioactive heat sources to generate power if they are going far from the sun - and I presume the thermoelectric effect would be vastly less efficient out at the orbit of Jupiter and beyond - where the sun is a tiny dot in the sky and solar panels aren't a great deal of use. SteveBaker (talk) 13:38, 3 March 2010 (UTC)[reply]

March 3

salt peter

does eating potassium nitrate reduce testosterone? ( pilgrims did it to lower the sex drive ) how does it work —Preceding unsigned comment added by 67.246.254.35 (talk) 00:14, 3 March 2010 (UTC)[reply]

According to our article Potassium nitrate, "saltpetre is thought to decrease sex drive, but there is no scientific evidence to support that the substance causes such an effect". And the reference for that is Jones, Richard E.; Kristin H. López (2006). Human Reproductive Biology, Third Edition. Elsevier/Academic Press. p. 226. ISBN 0120884658, 9780120884650. --Tagishsimon (talk) 00:29, 3 March 2010 (UTC)[reply]
So, the myth that it causes impotence is just a big boner (def. 3) ? StuRat (talk) 18:30, 3 March 2010 (UTC)[reply]
Batman agrees. 86.177.121.239 (talk) 20:37, 3 March 2010 (UTC)[reply]
I had a teacher in school who hopefully was unaware of the more recent definition, and returned from the copy room with both sides of the test copied over the top of each other on the same side, only to say "I just pulled a giant boner". We never quite thought of that teacher the same way again. StuRat (talk) 19:18, 4 March 2010 (UTC) [reply]

brine shrimps

How long do brine shrimps live in enclosed containers? can they live longer if they break open the container and put them in a fish tank?? —Preceding unsigned comment added by Delvenore (talkcontribs) 02:10, 3 March 2010 (UTC)[reply]

Do you mean: “how long will their dormant cysts remain viable in a closed container?” The answer to this is easy: nobody is sure! anhydrobiotic animals are extreamy tollerant to harsh conditions when in this state. See: NASA article A Pothole in the Road of Life By: Leslie Mullen. This means that they will survive many years longer in this state than in a fish tank (year at most)-especially if the fish are hungry. --Aspro (talk) 14:56, 3 March 2010 (UTC)[reply]

Why would anyone use a pH meter instead of an indicator for acid-base titrations?

I've only used an indicator for acid-base titrations but I heard that some people use a pH meter instead. I don't really see much difference other than that indicator only works in certain pH range and pH indicator works in all pH. —Preceding unsigned comment added by 70.68.120.162 (talk) 03:57, 3 March 2010 (UTC)[reply]

A pH meter gives you a more precise value for your solution's acidity, which is why we use it whenever we have to make an acid-base buffer or otherwise adjust a solution's pH within a narrow range. Clear skies to you 24.23.197.43 (talk) 05:16, 3 March 2010 (UTC)[reply]

Rate of freezing vs temperature

Which would you say would freeze the fastest: Boiling hot water, room-temperature water, cold water? Explain. 198.188.150.134 (talk) 08:19, 3 March 2010 (UTC)[reply]

Welcome to the Wikipedia Reference Desk. Your question appears to be a homework question. I apologize if this is a misevaluation, but it is our policy here to not do people's homework for them, but to merely aid them in doing it themselves. Letting someone else do your homework does not help you learn how to solve such problems. Please attempt to solve the problem yourself first. If you need help with a specific part of your homework, feel free to tell us where you are stuck and ask for help. If you need help grasping the concept of a problem, by all means let us know. A hint, though: Mpemba effect -- Aeluwas (talk) 08:53, 3 March 2010 (UTC)[reply]
Nope, this is not a homework question! Just because I put the word "Explain" doesn't imply that at all. I was merely curious ever since I've read about the mpemba effect. I know hot water freezes quicker than cold water but what about room-temperature water vs cold water? Which freezes faster then? 198.188.150.134 (talk) 08:57, 3 March 2010 (UTC)[reply]
Why not try it and see for yourself? I'm sure it will be an interesting experiment. (A word of caution: Putting boiling water in the freezer may cause the container to shatter due to thermal shock.) 24.23.197.43 (talk) 09:28, 3 March 2010 (UTC)[reply]
There has to be some clarification and some care taken in defining our terms here. The Mpemba effect isn't magic. Boiling water has to become cold before it can freeze - so what this experiment says is that there are two kinds of "cold water" - the kind that has just come out of the cold tap of your sink - and the kind that was boiled and then cooled down to that same temperature. But we know that one container of chemically pure water at a specific, uniform temperature is no different than any other identical container of pure water at the same uniform temperature then the two must freeze at the same rate. Since the boiling water first has to reduce in temperature and then freeze - it can't possibly freeze faster than cold water if we control all of the bizarre side-effects relating to the nature of the container, the stuff that's dissolved in the water, evaporation during the experiment and inhomgeneities in the temperature.
But the Mpemba effect does work in the kinds of uncontrolled situations that come up in simple experiments. It clearly shows that in an uncontrolled situation, the boiling water freezes faster (at least for some definitions of "freezes"). So this can't strictly be about the temperature of pure water in an idealized container - it's about some other aspect of the situation. Given that, there will undoubtedly be some variations on the experiment that take more care over the purity of the water, the container it sits in, etc that will make the results come out the way common sense would make you expect them to.
Our article on the effect lists several causes that have been considered but all of them are about the nature of the container or some other aspect of the way in which the experiment is conducted. For example, it is far from clear whether boiling water will freeze sooner than cold water if both are stirred continually - or if the boiled water is kept in a gas-tight container that would prevent water from escaping - thereby reducing the overall volume that remains to be frozen. It's not clear whether the boiled water does indeed completely freeze before the cold water - it's suspected that a thin crust of ice may form on the top of the boiled water but the liquid beneath take much longer to freeze than the initially cold water. The Mpemba effect is therefore not so much to do with the properties of water - but more to do with the definition of the results and the way in which the experiment is conducted.
We know that when careful experiments are done, the anomalous results vanish and the system behaves as common sense says it should. However, having said that, we don't have a single, proven answer for why this happens in the uncontrolled case - and it would be nice if more studies could be done so that we can put this one to bed with a clear, logical explanation.
The bottom line here is that if you want to make some ice-cubes fast, you might well get them to freeze faster by boiling the water first...but on the other hand, maybe you won't - a lot will depend on the water quality and the shape and material of your ice-cube tray.
But you should not take away from that the idea that the laws of thermodynamics need to be repealed or that water has some kind of magical memory effect (such as, for example, the homeopathy nut-jobs would have us believe). It's simply that the physics and chemistry of the general situation are a lot more complex than the simple statement "hot water freezes faster than cold" implies. The Mpemba effect should probably state something like "Some mixture of hot water with other dissolved ingredients in an open container where convection currents are not inhibited will first produce some freezing before a similar mixture that starts off cold completely freezes." - which is a much less strong statement (and frankly, one that would not have surprised anyone). SteveBaker (talk) 13:14, 3 March 2010 (UTC)[reply]
Aristotle would have explained the Mpemba effect as a clear case of Antiperistasis and left it at that. This video demonstration was put on YouTube less than a month ago. Cuddlyable3 (talk) 16:49, 3 March 2010 (UTC)[reply]

Night Purge Ventilation

I was wondering if this was an effective method of cooling a building in a relatively mild climate. I can't find much reliable information. I think it relies on venting a building during the night when the air is naturally cooler and having the air cross a thermal mass, which will store the 'coolness' (release heat into air) and then cool the building during the day when it it occupied and warmer. Thanks.---- —Preceding unsigned comment added by 69.181.156.221 (talk) 08:22, 3 March 2010 (UTC)[reply]

From what I understand of your description of this method, it may amount to something as simple as leaving the windows open at night. Yeah, I normally do that (I live down south), and I tell you, it really works, 'specially on cool autumn nights (not so much in midsummer, unfortunately...) 24.23.197.43 (talk) 09:39, 3 March 2010 (UTC)[reply]
See passive cooling. 75.41.110.200 (talk) 14:20, 3 March 2010 (UTC)[reply]
I don't think simply leaving windows open is all that effective, for the following reasons:
1) The thermal capacity of air is quite low. This means that most of the heat stored in a building is in the floors, walls, furniture, etc., not just in the air. Therefore, you don't just need to exchange the air once, but several times, allowing some of the heat from the floors, walls, and furniture to bleed off into the air each time.
2) Strong winds are required to force the air out of the building. Unless you have consistently strong winds in the proper orientation and a clear path through the building that exchanges all the air, you need to use intake and exhaust fans. Fans can still use far less electricity than A/C, however.
3) Humidity and rain can be an issue. Rain is easier to deal with, as an overhang can prevent rain from spraying in. Humidity, on the other hand, can be a real problem. Even if you lower the temp of the building, raising the humidity can make it even more uncomfortable, and more expensive to remedy with A/C. Therefore, exchanging air only makes sense where and when there's dry air outside. A desert would be a good choice, for example, with cool, dry air at night.
4) Papers can be blown around. This can be fixed, of course, if people know to secure all papers before they leave at night.
5) Windows don't open in many modern office buildings. This is a serious limitation when it comes to being environmentally friendly, leading to using A/C inside even when it's nice and cool outside, due to solar heating of the building and heat generated from the people and machinery inside. Screens are also required to keep insects, bats and birds out, and most office buildings don't have those, either.
6) Overcompensation can be an issue. To keep it cool all day long, you need to make it quite cold in the morning, perhaps so cold people would need to wear jackets. StuRat (talk) 14:50, 3 March 2010 (UTC)[reply]
Original research here: it works just fine in a small building in a semi-desert climate. Rain is rare, and the roof overhang common in residental structures takes care of that when it does happen. Strong winds are not required: you can exchange ventilation area for air speed, so (for example) a single large sliding door, a few windows, and a wind speed of one mile per hour can exchange the air in my apartment once every three minutes. --Carnildo (talk) 01:48, 4 March 2010 (UTC)[reply]
I'm rather skeptical of the 3 minute claim. How have you measured it ? Here's a test you can do: Let an egg rot in a sealed container (like a mason jar), then expose it to the air in the apartment. Open the windows and see if the stink completely disappears in 3 minutes. StuRat (talk) 15:26, 4 March 2010 (UTC)[reply]
I've done something similar: cook something with a strong odor, then see how fast the smell goes away after I put away the leftovers. The kitchen circulation isn't as good as the rest of the apartment, but it's still gone within ten minutes. --Carnildo (talk) 01:23, 5 March 2010 (UTC)[reply]

general principles of pharmacy technician practice

What are the relationship between the pharmacy technician and other health workers in an hospital setting. —Preceding unsigned comment added by Enipez (talkcontribs) 08:57, 3 March 2010 (UTC)[reply]

Um, they ask him for drugs, and he gives them what they asked for ? I have a feeling this a homework Q, so look through the book and regurgitate whatever they say about it. StuRat (talk) 22:21, 3 March 2010 (UTC)[reply]

Why bring eggs to room temperature?

Cookbooks always tell you to bring eggs to room temperature before using them in a recipe, such as for cake or cookies, but they never tell you why. (I know that people in many countries don't refrigerate eggs in the first place, so this question really only applies to those who do usually refrigerate them.) What difference does it make to my cake or cookie recipe whether the eggs are cold or not when I beat the eggs in? The batter/dough stabilizes to approximately room temperature by the time it's all mixed ready for baking anyway, doesn't it? +Angr 10:46, 3 March 2010 (UTC)[reply]

I'm not really sure, but it could have something to do with them being more viscous below room temperature (and therefore more resistant to being mixed into the dough). Clear skies to you 24.23.197.43 (talk) 11:02, 3 March 2010 (UTC)[reply]
Yes, that's the idea. Colder eggs will tend to clump together and form lumps in the batter. It's not a huge difference, though, and you could probably just beat the batter a bit more to get the lumps out, if you don't have time to wait for eggs to warm to room temp. StuRat (talk) 14:31, 3 March 2010 (UTC)[reply]
(ec)There are a couple of different reasons, depending on the application. For example, in an omelet the eggs should be warm because the short(er) thermal trip from raw to cooked gives you a shorter cook time and therefore a more tender end product. In other cases, such as where the egg (specifically the yolk) is being added to create an emulsion the warmer temperature helps reduce viscosity and allow for more thorough mixing (and therefore a more even texture), as 24.23 says above. Matt Deres (talk) 14:40, 3 March 2010 (UTC)[reply]
Perhaps also for the same reason recipes often call for unsalted butter only to later add salt: so that everyone's starting on the same page, whether they refrigerate their eggs or not. --Sean 16:43, 3 March 2010 (UTC)[reply]
Page 208 of Shirley Corriher's Bakewise says that room-temperature eggs beat faster and separate more easily. Similarly, page 122 cites a source who says that cold eggs give less volume. -- Coneslayer (talk) 16:56, 3 March 2010 (UTC)[reply]

Sexuality of Hedgehogs

I have three questions:

  1. Are there any recorded cases of gay / lesbian hedgehogs?
  2. Can and do hedgehogs masturbate?
  3. Do hedgehogs experience orgasms? —Preceding unsigned comment added by Delvenore (talkcontribs) 11:03, 3 March 2010 (UTC)[reply]
I refer you to "The Hedgehog Song".http://www.youtube.com/watch?v=wmPeX6syPPY (Warning: lyrics NSFW) --TammyMoet (talk) 13:00, 3 March 2010 (UTC)[reply]
The Wikipedia article Hedgehog has a section about the animal's reproduction that mentions that their physical difficulty of mating has a counterpart in human psychology. Cuddlyable3 (talk) 17:22, 3 March 2010 (UTC)[reply]

inhaling

I learned that when smoking marijuana it is more effective to "hold it in" for as long as possible after inhaling to maximize the amount of THC taken in. Is this tactic effective? —Preceding unsigned comment added by ShadowFire101001 (talkcontribs) 11:17, 3 March 2010 (UTC)[reply]

Probably, but it will also maximize the amount of tar that ends up in your lungs. Eating "special brownies" or taking THC capsules would be far more effective at getting the good stuff without the bad. StuRat (talk) 14:19, 3 March 2010 (UTC)[reply]
In USA Marihuana Tax Act of 1937 criminalised sale and use of Marihuana which is a preparation of the Cannabis plant. It is a Class B controlled drug in the UK. Smoking of cannabis is the most harmful method of consumption and brings the user into close or actual involovement with Drug abuse, illegality and drug peddlers.THC is only one of the active chemicals that a marihuana smoker ingests. Some research suggests THC may have some medicinal value besides its toxicity and negative mental effects. It is irresponsible to encourage anyone to self-medicate with THC derived by smoking, as might be inferred from StuRat's expression "the good stuff". Cuddlyable3 (talk) 14:52, 3 March 2010 (UTC)[reply]
Who is asking anyone to self-medicate? Is it more irresponsible than telling someone how to mix a good drink? What if the original poster is in a jurisdiction where marijuana is legal? Just pretend the poster asked: "Is the transference of THC to the bloodstream maximized by holding in the inhalation of marijuana smoke, or is that a myth?" A perfectly good science question. --Mr.98 (talk) 15:28, 3 March 2010 (UTC)[reply]
That's 3 supplementary questions. Answers in order: 1) Nobody asked. StuRat encouraged, see above. 2) Yes. 3) Legality does not make an action harmless. No, we don't rewrite an OP's question. Cuddlyable3 (talk) 17:04, 3 March 2010 (UTC)[reply]
1) No, I didn't.
2) Wrong. Alcohol is far more harmful, as numerous studies and statistics have shown.
3) It's absolutely true that being legal doesn't make something harmless, as in the cases of alcohol and tobacco. Conversely, being illegal, ostensibly for health reasons, in no way means that the substance is actually harmful. Unfortunately, lobbying by powerful interests has far more impact of what is legal or criminalized than science ever will. StuRat (talk) 17:44, 3 March 2010 (UTC)[reply]
So... let me get the right: we shouldn't tell someone about marijuana, because they might continue doing exactly what they already said they are doing. StuRat (and my) answer is actually one that encourages one to do the less harmful practice here—emphasizing the lung damage that comes from such activities (and such myths). Whether you think people should abstain from consuming THC in any form or not seems irrelevant here. Anyway, no one is encouraging anyone to do anything (other than not believe harmful myths!). In any case, this is a topic that has been researched by real-deal scientists, so I think that by itself should give us some clue that it's not an illegitimate question for the Science ref desk. (In any case, the post doesn't say that the poster actually smokes—just that they "learned" this myth from someone.) --Mr.98 (talk) 17:51, 3 March 2010 (UTC)[reply]
1) StuRat identified THC as "good stuff" and thereby endorses ingesting THC with no warning or qualification. That is not NPOV and a drug peddler's pitch would sound the same.
2) Good drinks are not necessarily alcohol drinks. StuRat's argument is to introduce a strawman because mixing alcohol drinks has nothing to do with self-medication with THC.
3) I answered the question "What if....legal" with a truth that StuRat agrees.
Cannabis has been judged by many to be so harmful that it must be controlled by laws. It is okay to debate the medical evidence for those judgements. But StuRat cannot dismiss such a weight of evidence and legal opinion merely by claiming that it "in no way means" anything. The rant about alcohol, tobacco and alleged "powerful interests" is OT. StuRat claims science will always be impotent in guiding legislation but that prediction is crystal ball gazing. Cuddlyable3 (talk) 03:04, 4 March 2010 (UTC)[reply]
1) There's plenty of medical evidence or the benefits of THC: [10], but that really isn't relevant the the Q, which was how to increase absorption of THC. Whether they should is not the Q (and would be a request for medical advice).
2) You know very well that the phrase "drinks" was used in this context to mean alcoholic drinks. Pretending otherwise is disingenuous.
Science should never be based on useless "legal opinion". When we do so we get really bad science, like Galileo being forced to say that the Sun revolves around the Earth, in order to comply with the law. I don't know what "OT" means, did you mean OR (Original Research) or OTT (Over The Top) ? The La Guardia Committee was an early scientific inquiry which determined that marijuana was not as harmful as politicians, specifically Harry J. Anslinger, claimed in their justification for criminalization. StuRat (talk) 15:05, 4 March 2010 (UTC)[reply]
1) StuRat you use the qualifier plenty to tell us that your opinion is that there is enough evidence to justify what you endorse. I say your endorsement of THC as "good stuff" is reckless and opinionated. Your alleged plentiful evidence hasn't overwhelmed the medical world yet.
2) Please use any English dictionary to help you comprehend the word "drinks". It is a word not a phrase. If you mean "alcoholic drinks", "carbonated drinks", "hot drinks", "cold drinks" or any other category of drinks then make the effort to write what you mean. Contrary to your false claim, I did not know that you meant "alcoholic" drinks because I don't read your mind.
OT stands for Off Topic. If you now imagine yourself as a champion of good science like Galileo Galilei and living in a world where legalities are mere useless opinions then you have detached yourself from reality as well as social responsibility. Cuddlyable3 (talk) 21:54, 4 March 2010 (UTC)[reply]
1) Since you are the one claiming THC does more harm than good, it's your responsibility to back that claim up with evidence, not mine.
2) You are either unaware that the meaning of words is dependent upon the context in which they are used, which would mean you suffer from a learning disability, or are just pretending to not know what the word meant in this context, to be difficult. Clearly (to everyone but you), the word "drinks", in the context of a discussion of another drug, THC, refers to drinks containing a drug, namely, alcohol. And that would be ethyl alcohol, just in case you are unable to determine this from the context, due to your learning disability. If you are really that incompetent to not understand what words mean in context, then you shouldn't be (trying to) contribute here. And you also seem to think that I used the word initially. I didn't, Mr. 98 did.
If you imagine that legal opinions carry any weight in the scientific world, it's you which have detached yourself from reality. More recent cases involve the teaching of evolution. But, I guarantee that scientists won't change their stance on evolution based of whether the law decides it can be taught or not. StuRat (talk) 23:24, 4 March 2010 (UTC)[reply]
1) False premise. I have made no such claim.
2) StuRat's response violates WP:NPA. Otherwise StuRat's retort is a collection of fallacies, still defending the notion that the qualifier "alcohol" must be assumed where it does not exist because that would be part of Mr. 98's pro-marihuana smoking rhetoric.
StuRat must be informed that:
  • The Ref Desk is not the place for pursuing an opportunistic debate that is OT to the OP.
  • StuRat may argue about marihuana in an apropriate forum. StuRat would be wise to avoid hand-waving claims of having plenty of evidence, denial of the function of Law in civilised society, personal insults, and ridiculous parallels to Galileo and Evolution.
I have identified in red the abuse by StuRat to help intervention stop this. Cuddlyable3 (talk) 13:51, 5 March 2010 (UTC)[reply]
As for answering the question... what you're asking about, in medical terms, is whether "breathhold duration" affects the absorption of THC or not. As far as I can tell from a few articles on Google Scholar, the answer seems to be, "probably not." On the contrast, they do seem to be associated (as StuRat said) with higher levels of tar, which is bad. My skimming of the literature seems to imply that volume of smoke matters (which makes sense), but breathhold duration does not. (This page has a lot of references to studies on it, along with summaries of their conclusions.) So, without wanting to give any kind of medical advice, I would tend to think that a marijuana smoker would be better off not holding their breath for a long time, as it does not seem to increase THC absorption, and does more lung harm that way. --Mr.98 (talk) 15:37, 3 March 2010 (UTC)[reply]
Bill didn't inhale. (video) Cuddlyable3 (talk) 17:08, 3 March 2010 (UTC)[reply]
And, based on some of W's statements, I'd have to say he's "still waiting to exhale". StuRat (talk) 22:12, 3 March 2010 (UTC)[reply]
No, you don't have to say anything. But anything you do say will be used to judge you. Cuddlyable3 (talk) 03:04, 4 March 2010 (UTC)[reply]

How many degrees of freedom does a string have ie the largest possible set of commuting observables? What are these degrees of freedom? 174.112.66.226 (talk) 13:13, 3 March 2010 (UTC)[reply]

I seem to recall that there are many variants on string theory, some of which have closed strings (loops) and others of which have open strings. I suspect that this would result in a variable number of degrees of freedom. StuRat (talk) 16:42, 3 March 2010 (UTC)[reply]
Yes, I was aware of this. In fact, many string theories have both closed and open strings. 174.112.66.226 (talk) 17:32, 3 March 2010 (UTC)[reply]
Would the number of degrees of freedom correspond with the number of dimensions, in each version of string theory ? StuRat (talk) 18:21, 3 March 2010 (UTC)[reply]
I doubt it; there should be more degrees of freedom than dimensions, since a string is not a point particle; it has substructure. I want to asking what degrees of freedom this substructure has, since I was already aware of the position/momentum/angular momentum degrees. 74.14.111.8 (talk) 21:52, 3 March 2010 (UTC)[reply]
But aren't there extra hidden dimensions associated with that internal structure ? I don't believe that string theory limits itself to the familiar 3 linear and one temporal dimension. StuRat (talk) 22:07, 3 March 2010 (UTC)[reply]
There are extra spatial degrees of freedom, but, even if all dimensions are treated as spatial rather than internal, there are still more degrees of freedom than a point particle in 10/11/26 dimensions would have. 70.27.196.12 (talk) 01:30, 4 March 2010 (UTC)[reply]

"Dimensions" in string theory is really code for the number of independent degrees of freedom, so there are 10 or 11 or 26 depending on the assumptions of the theory. The classical picture of a physical string with substructure that might be arbitrarily positioned really isn't helpful since the eigenstates of a quantum string are only allowed to be pure vibrations with an integer number of waves. Dragons flight (talk) 02:08, 4 March 2010 (UTC)[reply]

Thank you for your answer. Just to make sure I fully understand you, does that mean that a single string without interactions would behave identically to a point particle in however many dimensions with an extra quantum number representing the number of waves? Wouldn't a string's vibrations require a plane of polarization? 70.27.196.12 (talk) 03:24, 4 March 2010 (UTC)[reply]
Your question doesn't really make sense. If you imagine a physical string tied to a post, then you might shake it right-left and up-down. Those would be two dimensions. In string theory, the number of dimensions specifies the number of "ways" in which one can "shake" the string. For each dimension there would exist an integer specifying the number of waves in that direction (subject to various technical constraints). Once you specify all 10 (or 11 or 26) integers you have completely specified the properties of some possible particle. How those integers map onto conventional quantum numbers is not necessarily trivial however. Dragons flight (talk) 04:28, 4 March 2010 (UTC)[reply]
Oh, I get it now. 99.237.180.215 (talk) 17:07, 4 March 2010 (UTC)[reply]

mass/weight question

Does a Pokemon retain the same mass/weight when inside its Pokeball —Preceding unsigned comment added by Teltala (talkcontribs) 13:30, 3 March 2010 (UTC)[reply]

According to Gameplay of Pokémon#Poké Ball, the Pokémon is converted into energy when put into the Poké Ball. You may like to read mass–energy equivalence to see what happens when mass is "converted" to energy. +Angr 13:53, 3 March 2010 (UTC)[reply]
Also, I'd ask a fictional Q like this at the Entertainment Desk, not here. StuRat (talk) 14:16, 3 March 2010 (UTC)[reply]

Science for Dummies website

Any suggestions for websites for the scientifically challenged? What I would like to do is read about a certain topic then do my own research on Wiki. BTW, I am a fan of Neil deGrasse Tyson's approach to teaching astronomy, for instance. --Reticuli88 (talk) 13:47, 3 March 2010 (UTC)[reply]

Simple Wikipedia. Cuddlyable3 (talk) 14:21, 3 March 2010 (UTC)[reply]
Well, there's the PBS website. You might like the Discovery Channel, too. Also, you might want to get the book Cosmos by Carl Sagan or rent the series on DVD. StuRat (talk) 14:23, 3 March 2010 (UTC)[reply]

Thank you,thank you Cuddlyable! Did not know this existed! Thank you!--Reticuli88 (talk) 14:25, 3 March 2010 (UTC)[reply]

Simple Wikipedia is written in a simplified form of English. It uses a restricted set of words in its articles, but that doesn't mean it makes science topics easier to understand for non-scientists. I still find simple:Quantum mechanics very difficult to understand, for example. +Angr 16:13, 3 March 2010 (UTC)[reply]
It may be an unpleasant truth that some topics will be dificult to wrap your mind around regardless of how well it is explained.
There are two types of simplification, those which still present the full material, but in an easier to understand fashion, and those which actually present a simplified (and thus less technically accurate) model. For example, subatomic physics gets quite messy when electrons are thought of as a quantum probability function, but is far easier to picture when they are thought of as little balls orbiting the atomic nucleus in circular rings. Depending on the audience, this simplification may be entirely sufficient, as it is for most chemists, for example. StuRat (talk) 17:31, 3 March 2010 (UTC)[reply]
See lie-to-children for more on this kind of simplification. -- Coneslayer (talk) 19:06, 3 March 2010 (UTC)[reply]
There are many adults, even scientists, which use simplified models for the majority of their work, such as the atomic model used by most people in the chemistry industry. For another example, Newtonian physics is used for almost all real-world calculations, as time-dilation and other relativistic effects simply aren't significant when, say, designing a car bumper. StuRat (talk) 19:31, 3 March 2010 (UTC)[reply]
The article I linked to says The word "children" should not be taken literally and Such statements are not usually intended as deceptions, and may, in fact, be true to a first approximation or within certain contexts. (However, I would also point out that there's a difference between using a simplification in your work, and teaching that simplification as "the truth." Lies-to-children is specifically about pedagogy, not simplification/approximation as strategy in problem solving.) -- Coneslayer (talk) 19:51, 3 March 2010 (UTC)[reply]
Yes, and that's why it's not particularly relevant to what I was talking about. StuRat (talk) 20:02, 3 March 2010 (UTC)[reply]
Depending on the type of science question How Stuff Works might be OK. APL (talk) 16:23, 3 March 2010 (UTC)[reply]

Turbulent gas

I have a gas stove. When I turn it on low, it makes a steady hissing sound and each of the flame tips stays blue and at the same height. When I turn it up high, it makes a sound more like a windy day, and the flame tips flicker in and out and up and down with flecks of orange mixed in with the blue. I assume that the first case is laminar flow and the second case is turbulent flow. Is this correct ? Also, does turbulent flow lead to less complete combustion, and thus more unburnt gas and carbon monoxide released into the air ? StuRat (talk) 15:27, 3 March 2010 (UTC)[reply]

you should probably clean the burners - you shouldn't get turbulent flow like that, even under high pressure. It's difficult to say whether you are getting less complete combustion or more diffuse combustion - both would produce orange flame because the combusting elements are less concentrated. --Ludwigs2 16:08, 3 March 2010 (UTC)[reply]
Have a look at the illustration of flames at the article Bunsen burner. Cuddlyable3 (talk) 20:58, 3 March 2010 (UTC)[reply]
That seems to imply that the orange flame is a sign of insufficient air for the gas volume, and thus incomplete combustion. StuRat (talk) 21:19, 3 March 2010 (UTC)[reply]

Traffic

Is it legal to travel down a road in reverse, as long as your following the direction of the traffic? —Preceding unsigned comment added by Dendalonger (talkcontribs) 16:03, 3 March 2010 (UTC)[reply]

no. --Ludwigs2 16:09, 3 March 2010 (UTC)[reply]
Specific laws will apply in different jurisdictions. In the UK, if I recall correctly, the Highway Code says that one may reverse only as long as is necessary for the manoeuvre one is doing (turning, parking, backing out of a space or away from an obstacle); I don't think it specifies an actual distance. -- Finlay McWalterTalk 16:11, 3 March 2010 (UTC)[reply]
Or there's the cover-all driving without due care and attention, a bucket designed to scoop up the many stupid things one can do with a car. FMcW is right: section 203 of the Highway Code says "You MUST NOT reverse your vehicle further than necessary."--Tagishsimon (talk) 16:12, 3 March 2010 (UTC)[reply]
every jurisdiction I know of has generalized laws against driving in a fashion that endangers other people. Trying to drive a car in reverse under normal traffic conditions will inevitably constitute endangerment - difficult body posture, poor visibility, poor steering control, and etc all add up to lessened control over the vehicle. ** --Ludwigs2 16:19, 3 March 2010 (UTC)[reply]
** = deleted part of post. Cuddlyable3 (talk) 16:53, 3 March 2010 (UTC)[reply]
On single-track roads it is legal and often necessary to reverse for a hundred yards or more to find a place wide enough when meeting a vehicle travelling in the opposite direction. Those who regularly drive on such roads become adept at reversing at speed. Such roads are perhaps rare in the USA. Dbfirs 02:07, 4 March 2010 (UTC)[reply]
Most such roads (in the UK, at least) have regular passing points, so 100 yards would be unusual. If one (or both) of the vehicles is particularly wide (I was once on a bus that met a horse-box on a narrow road) then much more reversing can be required. This is all consistent with what has already been said - you can reverse for as far as it necessary. Sometimes it is necessary to reverse 100 yards, so you are allowed to do so. --Tango (talk) 04:53, 4 March 2010 (UTC)[reply]
They aren't rare in the US, just rarely traveled. Most of them are logging roads or other limited-use roads. --Carnildo (talk) 01:32, 5 March 2010 (UTC)[reply]

Nothing but the tooth

Resolved

I hear a lot about how acidic drinks, like soda pop, dissolve the teeth, leading to cavities. How about alkaline drinks ? Are there any, and are they bad for the teeth ? StuRat (talk) 16:45, 3 March 2010 (UTC)[reply]

Unless you happen to drink them both at the same time (and in the same proportion, such that they balance each other out), conventional wisdom tells us that alkaline (caustic) liquids can be just as damaging, in similar ways, as acidic liquids. As far as their existence, there certainly isn't anything that rivals the prevalence of carbonation/phosphate additive in modern drinks. Probably thanks to the pretty bad reaction they have with the taste buds.--144.191.148.3 (talk) 17:00, 3 March 2010 (UTC)[reply]
There just aren't many alkali foods at all. Alkali foods have a very bitter taste; there are a few foods which are processed with alkali (like Dutch process chocolate and Hominy). However, in these foods, the alkali is often washed away, and the food itself is used in applications that returns them to a more neutral or slightly acidic state. The only food I can think of which is eaten in a strongly alkali state is lutefisk, the taste of which can most charitably be described as "soapy". Generally speaking, however, nearly all foods lie on the lower side of 7 on the pH scale. --Jayron32 17:07, 3 March 2010 (UTC)[reply]

So, basically, there are no alkaline foods or drinks. I don't think that's a lye, but what about sour balls ? StuRat (talk) 17:22, 3 March 2010 (UTC)[reply]

Sour tastes are generally associated with acid not alkali. Googlemeister (talk) 17:35, 3 March 2010 (UTC)[reply]
<Facepalm>... Sour flavor, when added artificially, is often from citric acid or ascorbic acid. The kind of sour that you experience when tasting alkalinity is a less desirable one, to say the least. --144.191.148.3 (talk) 17:46, 3 March 2010 (UTC)[reply]

Baking soda is alkaline, right, and people use it to clean their teeth, right? Looie496 (talk) 21:40, 3 March 2010 (UTC)[reply]

Indeed, but you don't generally ingest a dentifrice; you spit it out. Likewise, while the alkali baking soda is often used as part of cooking, the end product which you actually eat, like a bread or a cake, is not actually alkali, it is usually very close to neutral or slightly acidic. --Jayron32 21:50, 3 March 2010 (UTC)[reply]
Tooth structure demineralizes at an average pH of 5.5 (I say average because enamel, dentin and cementum are of different compositions and relative mineralization and will dissolve at different pHs). The general formula for hydroxyapatite (tooth mineral) is [Ca5(OH)(PO3)]2 -- two molecules in a united pair. An acidic environment will draw the hydroxyl group out of the hydroxyapatite, causing it to dissociate. That's why F- is provided to protect teeth from demineralization -- despite what our article on Hydrofluoric acid states, dental text books promote the idea that HF moves towards ionization with such power that a proton will virtually never be able to pull the F- out of fluoroapatite as it would the OH from hydroxyapatite, and there you have acid-proof tooth structure. As a dentist, I never really thought about the effect of alkaline on teeth, but seeing how the theory promoted by the texts and professors is that the OH is pulled from the tooth by protons, alkaline would be able to accomplish no such similar thing. DRosenbach (Talk | Contribs) 02:58, 4 March 2010 (UTC)[reply]
Thanks. With that I'm marking this Q as resolved, as "alkaline foods and drinks are rare, and, even then, they don't cause tooth decay". StuRat (talk) 16:13, 5 March 2010 (UTC)[reply]

Fire drills

I've often thought that fire drills might actually harm more people than they help, when people, especially kids, are forced to stand outside in the cold, without coats, for hours at a time. I'd also include over-reactions, like when a waste-paper basket catches fire and is immediately extinguished, but the entire building is evacuated anyway until the fire department comes out and gives the "all clear". So, my questions are as follows:

1) Is the potential harm done to people by having them stand out in the cold without coats considered when scheduling drills, or reacting to minor fires ?

2) Does any jurisdiction prescribe methods for handling this, such as having blankets available in a shed in the evacuation area or a plan to move evacuees to a warm building nearby ? StuRat (talk) 17:15, 3 March 2010 (UTC)[reply]

Can you cite a source for the claim that fire drills cause people to stand outside in the cold without coats "for hours at a time"? Can you cite a source that standing in the cold causes harm to people? I mean, it's uncomfortable and annoying, but people aren't dropping dead from hypothermia during a fire drill, and the common cold is not contracted via exposure to colder weather. Comet Tuttle (talk) 17:36, 3 March 2010 (UTC)[reply]
I just saw a case on TV where kids were outside for 2 hours, and shivering severely, until a bus came to take them someplace warmer. In this case it was a bomb threat, but I've personally seen fire drills and reactions to minor fire events that have involved similar-length evacuations. StuRat (talk) 18:00, 3 March 2010 (UTC)[reply]
Given that the fire, the smoke from fire, or the panic from being unprepared for said fire all account for many many tragic deaths each year, I am going to venture the argument that spending 5 to 10 minutes outside in 'uncomfortable' weather to instill good fire preparedness is well worth it. If you are really standing outside for hours on end for a drill, or are doing it during truly inclement weather, your safety coordinator probably needs to find a new outlet for his masochism. Where I am in the US, we do fire drills every so often, and it's not on a bad weather day (since there is discretion during planning for a drill) and it never takes more than about 5 minutes of my time. Were this a high-rise building of some sort, I could see it taking maybe 15 minutes to get everyone out, accounted for, and back in but if the process to conduct the drill takes hours on end, imagine the hassle of fleeing an *actual* fire! --144.191.148.3 (talk) 17:51, 3 March 2010 (UTC)[reply]
But keep in mind that people probably evacuate 1000 times for every actual life-threatening fire. So, if only a few kids get sick each time, it may end up worse than the many injuries and deaths in actual severe fires. StuRat (talk) 18:00, 3 March 2010 (UTC)[reply]
It's one thing to say a few kids get sick, and a whole 'nother thing to say they get sick enough to perish. --Sean 19:59, 3 March 2010 (UTC)[reply]
Not really. Anything that makes thousands of kids sick (like millions of kids forced outside in the cold without coats) will probably kill a few of them, either directly or indirectly, as when their immunity is still low when they get some other fatal condition, or when the doctors kill them via medical error. The problem, though, is that the death certificate won't say "killed by a fire drill", so no statistics will be kept or action taken to prevent further such deaths. StuRat (talk) 20:11, 3 March 2010 (UTC)[reply]
As for evacuating a building for minor fires...Personal experience: two peoples' work went up in flames when someone (I think) knocked over a Bunsen burner in a chemistry lesson. The teacher came long, tipped the flaming work into the sink, then continued with the lesson. (The sink was wet; the fire went out almost immediately). No alarms, no evacuations, no-one had to leave the room. But, I live in Britain; where are you writing from? And to add to what the others have said - our drills were 10-15 minutes, and our accidental alarms were about 20. Vimescarrot (talk) 17:57, 3 March 2010 (UTC)[reply]
Well, if it never triggers the alarm, that might happen here in the US, too. But once an alarm goes off, I think schools are obligated to do a full evacuation. There are places in the US where the temp goes down to -40 in winter, and even a few minutes at those temps without coats could cause medical problems, even death. StuRat (talk) 18:03, 3 March 2010 (UTC)[reply]
The problem with fire is how rapidly something that "looks" small can become a huge and threatening danger. The general public is terrible at recognizing that fact, and even in cases where someone recognizes the real extent of a developing disaster, how do you convince others that they need to react a certain way, now, without panicing? And children certainly don't have the the experience to recognize and react properly to emergencies or the urgency of "get out now" vs "if you're a few minutes late wandering out for recess, that's okay". So instead, we have a plan that works in all normal situations rather than relying on those who are maybe not capable of making the correct judgement (and finding that person, and having him/her examine/decide, which takes time, etc.). And drills are a rehearsal so that when the actual performance comes, you don't have to figure out and read the instructions for the very first time in a situation where there isn't time. It's easy: "fire? pull alarm." "fire alarm? get out."--the potential for harm (and harm to others not just self) if you make either of those optional/judgement is huge. Modern buildings often have fire-alarm zones, so pulling in one area doesn't dump other areas, but that requires more complicated wiring of the alarm system and (more importantly) that the areas are strongly firewalled from each other. Different wings of a sprawling complex are like this. Also, tall buildings are sometimes zoned "alarm floor and the one immediately above it". That latter is interesting...it recognizes that fire tends to go up and that stairwells are a often bottle-neck so best to get the most-threatened people out asap rather than clogging the exit with downstairs people. Getting back to the risk/trade-off, somehow we ("public policy in the US") lean heavily towards "better safe than sorry", even if there are health risks and real costs of that safety. We usually go to an extreme to prevent disasters or react properly to them, even if they are likely to be rare, because disasters are so disasterous and unpredictable.
One of the funniest (in a scary way) things I've seen is a lab-safety training session on using fire extenguishers. Now this is a place where fires are a real possibility, they are detected immediately, can be extinguished rapidly, and if so there is little risk of spreading (but if not, can easily take out the whole building). And everyone knows "read the instructions on the extinguisher" (and even most knew "PASS-- pull, aim, squeeze, sweep"). So everyone's given an extinguisher and told "now use it". Usually takes several minutes (!) before the first person actually gets it discharged, and this was not in a panic with real risks as time runs and where the situation itself isn't predictable. DMacks (talk) 19:14, 3 March 2010 (UTC)[reply]
It actually might go better in a real fire, as nobody would try reading the instructions, which are probably full of time-wasting advice like "never insert the hose into your anus". :-) I hope I didn't put beans up anybody's nose :-) StuRat (talk) 19:24, 3 March 2010 (UTC)[reply]
Schools in the U.S. always did the fire drill, but after the Our Lady of the Angels School fire in 1958 killed 92 children and three nuns, when a fire started in the old firetrap school, there was a delay sounding the alarm, the wrong address was given to the fire department, and there was a lack of fire escapes, school fire safety became a much greater concern, and the drills were taken seriously. Did all parts of the building hear the promptly sounded alarm, and did they line up and leave in an orderly fashion in an acceptably short time? In a 1954 fire referenced in the first article,860 students in "Gadsden" safely exited in 2 minutes. Strangely, in one 1956 school fire, the Principal told the students it was just a fire drill, "to prevent panic." At Our Lady, a teacher escorted her pupils out, then went to notify someone in authority, rather than someone seeing smoke minutes earlier and sounding a central alarm. That fire also resulted in physical safety improvements, like better fire alarms and the installation of lockers in hallways to prevent clothing on pegs feeding the fire. Schools in the U.S. and elsewhere were inspected and many old schools were promptly closed. The frequency of fire drill was increased. Millions were spent on alarms, sprinklers and improved fire escapes, including security doors which opened automatically in a fire, and automatic alarms to fire stations. [11]. Fireproofing was added to over 16000 schools in the U.S. as a result of the fire, and 68 % of U.S. communities adopted stricter fire codes for schools.[12] Edison (talk) 19:25, 3 March 2010 (UTC)[reply]
And I'd be in favor of most of those things, but that doesn't mean nothing should be done to protect kids from cold during drills and actual fires. As for modern risks, I bet many inner-city schools chain the doors shut during the day, to prevent unauthorized access to the school by drug pushers and such. Unfortunately, I doubt if a fire drill would address such an issue, as the staff would just unchain the doors before the drill, then chain them back up again after recording a successful evacuation. A kid pulling a fire alarm would actually be a better test of whether the staff can get the doors open in time. StuRat (talk) 19:56, 3 March 2010 (UTC)[reply]
I know of no school which does that (and I have friends who teach high school in the Bronx). Unless you have a source, that sounds incredibly unlikely, due to the safety implications you mention. It's much simpler to simply make doors that can only be opened from the inside (without a key), and doors like those are used on many, many schools, not just those in the "inner city". —ShadowRanger (talk|stalk) 20:07, 3 March 2010 (UTC)[reply]
Here's a source: [13]. Doors that open from the inside only can still be used for criminal acivity with an accomplice on the inside, such as the student who wants to buy the drugs. You probably need a guard (possibly armed) at each door to both keep criminals out and allow a quick exit in the case of a fire, and that would be expensive. StuRat (talk) 20:42, 3 March 2010 (UTC)[reply]
Or you just alarm the door so the fire alarm goes off if opened, that way someone can't use it to sneak drugs into the school and it still works as a fire door. 82.132.139.83 (talk) 22:48, 4 March 2010 (UTC)[reply]
That was mentioned elsewhere. Kid opens door and gets drugs, alarm goes off, kid runs like hell and gets away, guards get pissed and chain door shut. StuRat (talk) 01:17, 5 March 2010 (UTC)[reply]

See the article Fire drill. The regularity of fire drills may be set by local authority rules. Examples in US. The purpose of the drill is to assess how long it takes to confirm that everyone has evacuated which imposes a certain waiting time that may be uncomfortable, though real harm to anyone is unexpected. Fire protection equipment such as blankets, hoses, extinguishers and alarms in a building is for immediate emergency response and dealing with unharmed evacuees is not a priority. Cuddlyable3 (talk) 20:45, 3 March 2010 (UTC)[reply]

They should do a proper cost/benefit analysis of fire drills, which includes a proper evaluation of the down side of such drills. I would expect them still to be worthwhile, but some measures should probably be taken to limit risks to kids during drills. There's also the risk that fire alarms can be used by terrorists to get kids outside where they can be attacked and killed, as they were here: [14]. StuRat (talk) 21:38, 3 March 2010 (UTC)[reply]
As opposed to the cost of paying out multi-million dollar wrongful death settlements after an actual fire kills some of your students? Or any number of other issues that chaining the doors shut would cause? They hire the security anyway, because the students themselves can be drug dealers or become violent. The easy solution to the case you describe is to have most of the doors alarmed. Then you only need one or two guards to cover the doors without alarms that are used regularly. Since the schools already have metal detectors installed, they need one or two guards to man them in the morning. You're being intentionally obtuse; try and think about what you're saying for a second or two before posting drivel. —ShadowRanger (talk|stalk) 20:51, 3 March 2010 (UTC)[reply]
"The door is alarmed ? Why, did you threaten to chop it up with an axe ?" :-) StuRat (talk) 21:10, 3 March 2010 (UTC) [reply]
Please don't get abusive. A door with an alarm on it isn't necessarily effective, as the kid who opens the door can probably run and hide before anyone responds to the alarm and catches him. Adding a video camera would help, but the kid could always cover his face as he opens the door, gets some drugs from someone outside, then run and hide. After this happens several times, the guards might stop responding altogether. All those doors with alarms and video cameras would also start to get expensive. One or two guards won't stop this, you'd need one for every door. As for the cost of a multi-million dollar lawsuit, the administrator who orders the door locked won't pay any portion of that, and, if there isn't a fire, they aren't likely to be punished at all. StuRat (talk) 21:10, 3 March 2010 (UTC)[reply]
By law, there needs to be a certain number of exits available, well-signed and accessible, that can be nearly instantly operated by nearly anyone. Your local fire marshal would be happy to give the building manager the option of either unblocking them or else having their occupancy permit revoked for violation of building/life-safety code, and your local news media would love to do an investigative report of a school (especially one in New York) that didn't learn from the Triangle Shirtwaist Factory fire. One popular design is doors with have electronic locks that unlock automatically by fire-alarm system. DMacks (talk) 20:55, 3 March 2010 (UTC)[reply]
I don't trust "automatic" locks not to lock people in during a fire. Some fairly complex electronics needs to work properly for the fire to be detected and for the doors to open. StuRat (talk) 21:30, 3 March 2010 (UTC)[reply]
Trust (or not) is certainly your prerogative, but "cut power to electromagnet" doesn't seem more complex than "send power to bell-ringer". DMacks (talk) 21:33, 3 March 2010 (UTC)[reply]
It's how to determine when to cut that power that's the issue. Do you do it when a fire alarm is pulled ? Then the kid who wants his drugs will pull the alarm, open the door, and get his drugs. Do you open the doors when a smoke detector is triggered ? Then the kids smoking in the bathroom will trigger it. How many times does this happen before somebody disables the door release circuit ? And, unlike a chained door, a disabled automatic lock isn't apparent until the actual fire. Also, if the alarm bell fails, there are backups, like the PA system or even somebody running down the halls yelling "Fire !", but what backup is there for an automatic door that won't unlock ? StuRat (talk) 21:47, 3 March 2010 (UTC)[reply]
I'm sure it's illegal to chain the doors shut, but that won't stop people from doing so. There would need to be wide-scale random inspections and long prison sentences whenever a violator was found, to have any hope of actually stopping this. StuRat (talk) 21:01, 3 March 2010 (UTC)[reply]
Why precisely would they do it? You're assuming people are doing it without giving a legitimate reason for them to do so, particularly given the consequences of violation. You can't assume something bad is happening and then demand something be done. Prove your case. Give one example of a school currently using this practice. Myths like this about inner city schools are based in ignorance and racism. —ShadowRanger (talk|stalk) 21:05, 3 March 2010 (UTC)[reply]
I already gave a source, here: [15]. The consequences to the person who orders the door chained are probably nothing more than a slap on the wrist (unless they get caught in a fire, that is). They would do it to prevent criminal activity (with no money budgeted to hire guards), which is far more likely than a fire. It's hardly an ignorant, racist myth that criminal activity is a problem in inner city schools, do you really need me to find a source to support this, too ? Please stop being abusive. StuRat (talk) 21:15, 3 March 2010 (UTC)[reply]
The source you gave answers your question: there are random inspections, there are directives issued, there is public outcry for change, there are fines for non-compliance. I have to agree with others, you seem to have a preconceived idea of a problem and are asking us to either prove you wrong or somehow else argue about it and criticize public policy you disagree with. To answer your original question (again as others have) yes, fire-drills are scheduled to be somewhat convenient/minimize disruption/etc and yes, some places do not just dump the whole building outside for extended periods. Do you have any further science question about this topic? DMacks (talk) 04:35, 4 March 2010 (UTC)[reply]
Obviously, once it reaches the newspapers the problem will be addressed, at least for a while. The problem is all those cases that have so far avoided public scrutiny. The percentage of schools with illegally locked doors will reach an equilibrium in each area, with factors like widespread, truly random inspections and severe punishment for those who locked the doors (not a fine for the school) tending to move the percentage down. Also, kids who tell their parents and the press about it might help, too, but that only works if the locked doors are visually apparent. Automatic unlocking doors that would fail to open in a fire aren't visually apparent and thus such a method wouldn't work on them. Also, some time after a public outcry over locked doors, I'd expect the inspections to go down, and possibly be scheduled for specific dates, resulting in administrators illegally locking the doors again and only unlocking them on inspection days. Also, low safety and security budgets will lead to this cheap "quick fix". And yes, I do want more scientific info. What specific measures are taken to keep kids from the cold ? Where are these taken and at what times of the year ? I'd also like sources. StuRat (talk) 14:32, 4 March 2010 (UTC)[reply]
If kids were really standing for hours outside during a drill, then the folks running it are idiots incompetent. The purpose of a drill is not to make you stand outside in the cold, it's to get you familiar with the evacuation process. And if it's actually a fire, you're better off outside the building than inside it. ←Baseball Bugs What's up, Doc? carrots04:49, 4 March 2010 (UTC)[reply]
That would depend on the type of fire. In the example I gave earlier, of a waste-paper basket that caught fire when a ciggy was tossed in, was thoroughly extinguished, and is now sitting outside, filled with water; an evacuation could do more harm than good. Another example would be a pot of food that went dry and caught fire in the kitchen area, was immediately covered and extinguished, but set off the smoke detector anyway. I'd say that most fires are like this, with only a few escalating to life-threatening status. StuRat (talk) 14:39, 4 March 2010 (UTC)[reply]
The problem you seem to be missing is that by the time you've identified the cause, it's a bit late to evacuate if it's a serious fire. So that's why you don't do it. If the alarm goes off, you evacuate. Once you've identified the cause and determined it's safe, then you go back in. You don't wait until you know whether it's a serious fire before evacuating. Nil Einne (talk) 14:25, 5 March 2010 (UTC)[reply]
That's sometimes true, but a person is often present at the site of ignition, such as a cook at a kitchen fire. I was personally present at a "fire" where a waste-paper basket with a ciggy in it lit up, and saw the whole building being evacuated as a result (I was in the building, but somebody else was the witness to the start of the fire). StuRat (talk) 17:12, 5 March 2010 (UTC)[reply]
I don't know how things work in the US, but here in NZ if the fire alarm goes off (usually due to a smoke alarm) in any commercial building and I presume schools as well, you always evacuate, I presume it's mandatory. The fire service will come (an automatoic signal being sent went the alarm goes off) and checks out the alarm, if it's a false alarm they'll disable it. Most/all? buildings get one free call out per year, any further callouts will cost usually I believe NZ$1000 (although it may vary depending one the circumstances and I'm not sure if this applies to schools). This may not seem like much, but I'm pretty sure it's enough to ensure most buildings avoid false fire alarms. Of course I'm not saying it doesn't happen, you do see it resonably often (fire trucks and people standing outside) most commonly I assume it's mostly a false alarm (e.g. someone in the eatery burnt fish). Nil Einne (talk) 08:09, 4 March 2010 (UTC)[reply]
Fines for false alarms would tend to lower fire safety, by causing the building managers to disable the alarms to avoid further fines. StuRat (talk) 14:35, 4 March 2010 (UTC)[reply]
I don't think disabling the alarms is easy (remember these things automatically warn the fire station, if you try to fool around with them, there's probably a fair chance you'll end up sending off a warning) and it would likely carry a severe penalty (when caught by spot checks etc) and I've seen no evidence it's a regular occurance or a real problem. There's also the issue of mentality I guess. Perhaps in the US, the mentality is such that many managers would do such a thing and it wouldn't surprise me if you saw similar behaviour in Malaysia for example, but I suspect the mentality here in NZ is such that it doesn't happen that often. Also, I suspect the person who owns the building doesn't pay the cost in many cases anyway. If a restaurant sets off the smoke alarm due to burning fish, the restaurant pays. The owner isn't going to turn off the alarm, which other then the penalties involved, would likely affect their insurance because some tenant keeps setting off the alarm. Yes this may not apply to schools, but they aren't really a typical business in particular for any public school, there's likely a risk to the principals job if they pull off crap like that and in any case, there are likely plenty of people in the school who would be aware if this went on and they'll generally have the ability to get something done about it if they find out. Of course, this probably occurs with many commercial properties anyway in many cases the owner may be at arm's length from the management and if there are problems, with frequent false alarms, they'll expect the management to deal with it but if they deal with it by disabling the alarm, there's a far chance the owner isn't going to be pleased. Also it's not primairly intended to be a fine but a cost recovery exercise. Nil Einne (talk) 14:14, 5 March 2010 (UTC)[reply]
I think, rather than just giving them a fine, the fire department should work with the building owners, managers, and occupants to come up with a plan to keep the alarms functional, but also prevent further false alarms. I have an aunt who had an alarm go off every time she cooked, and I just moved the smoke detector further away from the kitchen, and the problem was solved. Of course, one could argue that this won't provide as quick of a warning of a fire that starts in the kitchen, but she was at the point of pulling the batteries out to disable it altogether, so my solution is certainly preferable to that. In a commercial setting, another option might be upgrading the ventilation system in a commercial kitchen to get smoke out of there before it builds to a point that triggers an alarm. StuRat (talk) 16:09, 5 March 2010 (UTC)[reply]
You've still not cited a source that standing outside in the cold during a fire drill has made anyone sick. Comet Tuttle (talk) 17:29, 4 March 2010 (UTC)[reply]
Here's a chart showing that frostbite can occur in 15 minutes or less at -10F with only a 5 mph wind: [16]. I'd expect that even a fast fire drill takes about that long. Hopefully they avoid fire drills when it's that cold (although I'd still like a source with a policy on that), but that still leaves open the other part of my Q, how they deal with false alarms (and real fires) in cold temps to reduce this possibility. StuRat (talk) 18:05, 4 March 2010 (UTC)[reply]
Nice chart, but, again, you have not cited a source that standing outside in the cold during a fire drill has made anyone sick. I think it's laudable that you think that cost-benefit analyses are worthwhile, but I don't see a lot of evidence here demanding a reaction. Comet Tuttle (talk) 18:40, 4 March 2010 (UTC)[reply]
Well, it's circular logic to say that we shouldn't study hypothermia from fire drills unless we have proof that it exists from existing studies on hypothermia from fire drills. But what part of the logic chain do you doubt, exactly ? That fire evacuations happen when it's that cold or windy ? That they can last for 15 minutes ? Or that, as the chart states, this can cause frostbite ? Lacking any actual studies on hypothermia from fire drills/evacuations, the best we can do is to look at the components that would lead to hypothermia. StuRat (talk) 19:13, 4 March 2010 (UTC)[reply]
When I was in high school, we did fire drills (and one actual fire) under cold-weather conditions: instead of evacuating outside, everyone evacuated to the semi-detached cafeteria. The building had been designed for exactly that purpose, with two firewalls rated for half an hour each between the cafeteria and the main school building. I expect that most schools in cold-weather areas have similar plans. --Carnildo (talk) 01:40, 5 March 2010 (UTC)[reply]
Thanks for that. You know, you're the first person to actually provide an answer to part of my question, rather than ask me to prove the assumptions in it ? Well done. StuRat (talk) 12:56, 5 March 2010 (UTC)[reply]
Of course it's not that hard to find actual references. For example a very quick search finds [17] says that a fire drill was delayed because of cold weather and [18] says students are allowed to pick up coats etc provided it doesn't take extra time. Of course there's also [19] and [20] which are complaints about fire drills in cold weather Nil Einne (talk) 14:33, 5 March 2010 (UTC)[reply]
Thanks for those. I still get the impression that they look at it solely from the POV of making students uncomfortable, though, and not an actual risk to their health. That part about only being able to get coats "if it doesn't take any time" seems a bit silly, as other fire regulations require that coats be stowed in lockers, which, of course, require time to open. The last two sources are also from students and a parent, but I'm still looking for some official policy saying "fire drills will not be conducted when the wind chill factor is below X". StuRat (talk) 15:56, 5 March 2010 (UTC)[reply]

Is there a type of Bee/Wasp/Hornet that doesn't have a stinger?

A friend of mine mentioned this. He said it doesn't have a stinger, but it drops something on you that burns. And if you swat it on your skin, it leaves behind a residue that also burns. Does such a thing exist? ScienceApe (talk) 17:19, 3 March 2010 (UTC)[reply]

I've never heard of the burning substance part, but there are flies (no stinger) that mimic bees as a defensive measure. StuRat (talk) 17:25, 3 March 2010 (UTC)[reply]
The drones of many bee species lack a stinger, and other bee species (e.g. the mason bee) possess stingers but generally don't use them except when directly attacked (squeezed, stepped on, etc.), as they lack the territorial instincts of the honeybee. I don't know of any species with a contact venom. —ShadowRanger (talk|stalk) 19:07, 3 March 2010 (UTC)[reply]
Not a hornet, but see bombardier beetle (which is probably what your friend is thinking of) and perhaps assassin bug. Matt Deres (talk) 21:25, 3 March 2010 (UTC)[reply]
Perhaps your friend lived in ancient Canaan -- see here. DRosenbach (Talk | Contribs) 03:01, 4 March 2010 (UTC)[reply]
See stingless bee for stingless bees, but as MD says, your friend is likely talking about a non-bee. --Sean 20:58, 4 March 2010 (UTC)[reply]

anti-gravaton beam

In a surprisingly large number of Star Trek episodes the use of an "anti-gravaton beam" is employed to solve various problems; collapsing wormholes, disabling shields and deactivating mines to name but a few. What exactly is an "anti-gravaton beam"? Does such technology currently exist? —Preceding unsigned comment added by ShadowFire101001 (talkcontribs) 17:57, 3 March 2010 (UTC)[reply]

What is it? Nothing, it's completely fictional. The idea would be that a graviton would be a gravity-producing particle, so an anti-graviton would produce anti-gravity (a general repulsive force). You could use it to push things. Vimescarrot (talk) 18:00, 3 March 2010 (UTC)[reply]
Which in fact would clash with general relativity. The whole theory is hypothetical and has not even reached any mathematical formulation yet. noisy jinx huh? 18:07, 3 March 2010 (UTC)[reply]
From Memory Alpha [21]
Anti-gravity refers to the state in which an object defies the laws of gravity.
I take this to mean that an anti-graviton beam is something of a MacGuffin. Subryonic compound (talk) 18:16, 3 March 2010 (UTC)[reply]
I personally like the idea of anti-gravity, emanating from galactic voids, pushing the galaxies apart from each other and also compacting each one together. This would explain both the observations that the universe is expanding at an ever-increasing rate and that galaxies seem to have insufficient mass to hold themselves together by gravity alone. This also relates to the topics of dark matter and/or dark energy. StuRat (talk) 18:16, 3 March 2010 (UTC)[reply]
I believe that in current theory, the graviton's antiparticle is also the graviton. So an anti-graviton would be sort of like "unobtainium". It's completely fictional, and so transparently so that you can ascribe any properties you like to it. APL (talk) 20:19, 3 March 2010 (UTC)[reply]
As transparent as transparent aluminum ? :-) StuRat (talk) 20:36, 3 March 2010 (UTC)[reply]
It can not be as useful as ejecting the warp core Googlemeister (talk) 21:50, 3 March 2010 (UTC) [reply]
When Dr. McCoy used one of his instruments to scan a person's body, the instrument he was using was actually a salt shaker. A subliminal message to take any preposterous-sounding technology with a few grains of salt. ←Baseball Bugs What's up, Doc? carrots04:53, 4 March 2010 (UTC)[reply]

theory of relativity

Apparently Albert Einstine didn't invent the theory of realitivy he just modified an early theory. Is this true? Who was the original author? —Preceding unsigned comment added by Delvenore (talkcontribs) 18:09, 3 March 2010 (UTC)[reply]

Since you are the one claiming that such a theory existed, may be you should be telling us who did it. As far as I know there was not such a theory. Dauto (talk) 18:16, 3 March 2010 (UTC)[reply]
Perhaps you are thinking of the work of Lorentz? -- Coneslayer (talk) 18:17, 3 March 2010 (UTC)[reply]
Maybe these articles will help. History of special relativity / History of general relativity. --Mark PEA (talk) 18:19, 3 March 2010 (UTC)[reply]
I think Minkowski did some work on this stuff. —Preceding unsigned comment added by RubberBeaver (talkcontribs) 20:59, 3 March 2010 (UTC)[reply]
Einstein's work was indeed based (like all scientific theories) on the work of others. The general ideas that are contained in the theory special relativity are very similar to lines of thought being pursued by Hendrik Lorentz, Henri Poincaré, and others at the time. However, Einstein's work was in many ways quite different than both Lorentz's and Poincaré's. In retrospect, people go, "oh, it's the same thing!" mostly because they don't know how to distinguish them (e.g. why the Lorentz contraction is doing something different in special relativity than it is in Lorentz's ether theory), and because they have been sold a myth about how Einstein and science works, whereby a lone genius magically came up with a totally new way of seeing the world. Einstein had some really key insights, which he both derived from and merged with a lot of previous work. --Mr.98 (talk) 00:01, 4 March 2010 (UTC)[reply]
You might be talking about Galilean relativity. --Tango (talk) 04:57, 4 March 2010 (UTC)[reply]
The quote, which is attributed sometimes to Isaac Newton (its likely an old aphorism) applicable here is "If we have seen farther, it is because we have stood on the shoulders of giants." In other words, no scientist works in a vacuum. Every scientist, even the REALLY REALLY IMPORTANT ones, like Einstein and Linus Pauling and the like, makes incrimental progress towards increasing human knowledge. It may be helpful to think of each scientist adding some small % towards aligning our working models of the universe with the actual way the universe works. If the average physicist contributed like 0.01% towards improving the model, and Einstein was say 1000x more important towards that model than the average phycisist, he'd still only be contributing 10% of the total picture. So, he can still both one of the most important physicists in history, and STILL working extensively with the works of others. --Jayron32 16:37, 4 March 2010 (UTC)[reply]
"If I have seen a little further it is by standing on the shoulders of Giants." Buddy431 (talk) 06:50, 5 March 2010 (UTC)[reply]

Antimatter

Since regular matter emits regular gravity (in the form of gravitons), does antimatter, the opposite of regular matter, emit antigravity/negative gravity? --J4\/4 <talk> 18:18, 3 March 2010 (UTC)[reply]

The graviton is still a hypothetical theory, nothing more. noisy jinx huh? 18:21, 3 March 2010 (UTC)[reply]
(ec) This might be better joined with the Q 2 questions back. Also note that the graviton is only a theoretical particle. StuRat (talk) 18:24, 3 March 2010 (UTC)[reply]
(ec)Antimatter has mass equivalent to the mass of its matter counterpart; an anti-proton weighs the same as a proton. As such, it has the same gravitational attraction as matter; it doesn't cause a repulsive effect. This has not been directly observed to date (we don't have enough stable anti-matter to make any such observations), but it is the overwhelming consensus of physicists. Handily enough, we have an article on it: Gravitational interaction of antimatterShadowRanger (talk|stalk) 18:28, 3 March 2010 (UTC)[reply]

Sunlight

Why does the sun make your skin dark but your hair light? —Preceding unsigned comment added by Transfigurations (talkcontribs) 18:45, 3 March 2010 (UTC)[reply]

The reaction with skin is with living cells whereas the reaction with hair is with non-living cells. 71.100.11.118 (talk) 18:56, 3 March 2010 (UTC)[reply]
Sunlight makes photopaper dark (even without chemical development). By your reasoning, does that imply it is living? Edison (talk) 18:59, 3 March 2010 (UTC)[reply]
Edson, I think if you look carefully you might spot the fallacy of your statement above. Dauto (talk) 22:12, 3 March 2010 (UTC)[reply]
The fallacy is in the explanation by 71.100.11.118, which I pointed out. Edison (talk) 19:57, 4 March 2010 (UTC)[reply]
This is not relevant to the OP but, since the question has been answered below, I'll post. 71.100 said that skin cells are living and hair cells are not therefore sunlight darkens skin while it lightens hair. Your post above implies that 71.100 said that sunlight darkens skin therefore skin cells are living. Note: 71's post was incomplete and unhelpful and inappropriate, but not fallacious. Zain Ebrahim (talk) 22:08, 4 March 2010 (UTC)[reply]
Sunbleaching (what, no article ?) has an effect on many objects with pigment, living and inorganic, by breaking down the pigmentation molecules, where the light stops, doing damage in the process. A suntan is a reaction to sunlight's UV damage to the skin, which causes the release of melanin, to prevent further damage. This is a protective measure more common in people with light skin, which allows them to get the sunlight they need to produce vitamin D when sunlight exposure is low (as in winter), and yet not develop burns and skin cancer when sunlight levels are high (as in summer). Those whose ancestors lived closer to the equator tend to have more melanin, and hence darker skin, since sunlight exposure is high there year-round. Why doesn't hair have such a protective measure ? It's dead (except for the follicle), so isn't susceptible to cancer or sunburn. StuRat (talk) 18:58, 3 March 2010 (UTC)[reply]

cause of exponential expansion of the universe

My studies have come to the conclusion that the Big Bang is the result of the polar force of gravity I call anti-gravity. This force is also the cause of the expansion of the universe and explanation why the universe is expanding exponentially in the presence of less gravity to slow it. However, I think this is original research and can not add it to an article. Could the reference desk please inform me if this is original research of if others have reached the same conclusion? 71.100.11.118 (talk) 18:48, 3 March 2010 (UTC)[reply]

If you can link us to your studies published in a peer-reviewed journal, and preferably to some secondary sources discussing the importance of your findings and how meaningful they are considered, then we can add it to an article. If you have none of these things, it is original research and cannot be added. If you have those things, I'd be surprised that you are trying to get it published in an encyclopedia: I'd have thought you have more research to do, papers to write, conferences to speak at, etc. 86.177.121.239 (talk) 18:57, 3 March 2010 (UTC)[reply]
Note that Wikiversity lacks Wikipedia's ban on OR, so your thoughts (which happen to be quite similar to mine) would be quite welcome there. StuRat (talk) 19:11, 3 March 2010 (UTC)[reply]
Done, thanks. Exponential expansion of the Universe 71.100.11.118 (talk) 21:05, 3 March 2010 (UTC)[reply]
Please don't add your ideas to Wikiversity. One can't usefully teach a subject that one doesn't understand, and you don't know the first thing about cosmology. I mean no offense. If you'd like to learn cosmology, I recommend Introduction to Cosmology by Barbara Ryden (ISBN 0805389121). -- BenRG (talk) 23:27, 3 March 2010 (UTC)[reply]

artificial intelligence

Have artificial intelligence reached the point where they are self aware yet? I've tried the current AIs and they always just seem to regurgitate random parts of conversations you had previously with them, they don't seem to think —Preceding unsigned comment added by Aject8886 (talkcontribs) 19:31, 3 March 2010 (UTC)[reply]

Definitely not. We don't even know how to do that in theory, as simply adding more processing power just leads to a more powerful calculator. And, if we could make a self-aware computer; should we ? There are many moral concerns, leading all the way up to the world of the Terminator movies.
Also note that we may someday be able to create a computer which passes the Turing test, meaning we can't tell if it's following a program or actually thinking. But, to me, this just means it's able to simulate intelligence, not that it's actually intelligent. Is it just a new and improved Magic 8-ball ? The answer is unclear at this time. StuRat (talk) 19:40, 3 March 2010 (UTC)[reply]
"Sentience" is an abstract like "Art". I don't see that there's a difference between a perfect simulation and the genuine article. APL (talk) 19:51, 3 March 2010 (UTC)[reply]
Or maybe it's more like stage magic. It's only real if you don't know how it works. APL (talk) 19:53, 3 March 2010 (UTC)[reply]
That seems to be a popular opinion. Personally I think it's nonsense. I know very well that I am sentient, though I can't prove it to you. Showing that a simulacrum had the same electrical patterns would not address the question at all. --Trovatore (talk) 22:30, 3 March 2010 (UTC)[reply]
I didn't address electrical patterns at all.
I'm saying that as an abstract concept I'm not convinced that there's a difference between "real" and "exactly like real, but not".
If Leonardo De Vinci jumped out from behind a curtain and said "Haha! Fooled you! All of my paintings were based on mathematical equations and tedious computations and no creativity at all!" would that immediately move them from the realm of "Art" to some new realm of "Fake Art"? Of course not. There's no such thing as "Fake Art", it's a meaningless concept. I'm no philosopher, but I honestly don't understand the difference between "Sentience" and "Fake Sentience".
(Note : I do, of course, understand the concept of a "soul" as some immaterial, but necessary, prerequisite for intelligence. But I do not believe this theory. ) APL (talk) 23:13, 3 March 2010 (UTC)[reply]

(After E/C with StuRat)

You're probably thinking of things like Megahal, and yes. Those are specifically designed to regurgitate past conversations in a vaguely believable manner. Done properly they can be very convincing for short conversations. To my knowledge there is no "Full AI" that even comes close to fictional AI's like HAL 9000. I don't think there's even a good chatter-bot that can keep up a long conversation in a logical way.
Most of the research into AI nowadays goes into either AIs designed to complete some specific task, or to autonomous AIs with much 'smaller' minds than you'd get in Science Fiction. The idea being to duplicate, say, an ant's mind before trying to simulator a human.
"Self aware" and "Sentient" are difficult terms to quantify. A more objective standard is the Turing Test. Many people argue that a computer that consistently passes the Turing test would only be "pretending", and using a complex set of rules to create its intelligence. (Personally, I'm not at all convinced that those of us with squishy brains aren't doing the same thing. The laws of chemistry are just as immutable as a computer's instruction set.) A more serious argument is that a computer that is smarter than a human would also fail the Turing test.
The Artificial Intelligence article is a bit of a mess, but there's still good information there, worth reading. APL (talk) 19:51, 3 March 2010 (UTC)[reply]
(ec)The "AI"s you've communicated with are likely variants on the ELIZA bot, which was never even intended as an AI, but rather as a limited natural language processor. Natural language processing would be required to create an AI humans could interact with a la Star Trek, and true natural language processing might require AI, but they aren't the same field. Mimicking the processing power of even a fraction of the human brain would require substantially more processing power than we are likely to be able to produce in the next few decades (at least without introducing communications delays between the components). We have no other model for AI besides the human brain, so unless there is a major leap in AI theory, we're not going to see a self-aware machine for a long time. —ShadowRanger (talk|stalk) 19:53, 3 March 2010 (UTC)[reply]

Computers have been self-aware for ages. Every time you ask how much disk space is available, the computer is telling you about its internal state. Self-awareness is by no means the hardest part of artificial intelligence, in fact it's one of the easiest. And it definitely isn't the same thing as "thinking". Looie496 (talk) 21:32, 3 March 2010 (UTC)[reply]

I don't think that counts. Even the simplest animals can respond to internal state. When you're hungry - you go and eat. Knowing whether you are hungry or not is a very basic thing. We could probably show that plants are self-aware in that sense. Self-aware means being able to perform introspection on one's own thought processes. The tangled mess that is the first paragraph of Self-awareness attempts to say something like that! SteveBaker (talk) 02:05, 4 March 2010 (UTC)[reply]
The problem here is that there isn't a test for "self-awareness" - how can you tell whether any person you bump into in the street is self aware? You can ask them "Are you self aware?" - but then the following one-line C program would pass that test:
   int main () { printf ( "Yes!  I am self-aware.\n" ) ; return 0 ; }

...so that's not going to work. We have no idea whether small children, dolphins, dogs, or ants are self-aware. It's possible that I'm the only entity in the universe that is self aware...and everyone else has a mindless ability to claim to be self-aware when I ask them...but you may have a different outlook than me!

As for AI, well, the business of building a truly intelligent system has really gotten nowhere since the 1960's. The AI community are attacking more tractable problems - things like linguistics, speech, visual recognition, knowledge representation - things that hover around the edge of actual intelligence. The Turing test will undoubtedly be passed - and probably not long from now - but serious AI researchers don't regard that test as anything particularly meaningful these days and the thing that passes it would seem remotely intelligent to careful investigators.
SteveBaker (talk) 21:50, 3 March 2010 (UTC)[reply]
Steve, may I lend you an "n", an aposthrophe, and a "t", so you can change your "would" to a "wouldn't" ? StuRat (talk) 21:58, 3 March 2010 (UTC)[reply]
As long as I'm picking Steve's nits -- that one-line C program won't compile. (Don't believe me? Try it.) --Trovatore (talk) 22:22, 3 March 2010 (UTC)[reply]
I did and it does. There's a warning about printf being implicitly redefined, but it works fine. Gcc version is i686-apple-darwin8-gcc-4.0.1. --Stephan Schulz (talk) 23:21, 3 March 2010 (UTC)[reply]
Whoops, you caught me speeding — I didn't try it. I assumed you had to #include "stdio.h" in order to use printf(). --Trovatore (talk) 23:46, 3 March 2010 (UTC)[reply]
GAh! I just ran this code (It compiled without warning here.) and now my machine claims to be self-aware! SteveBaker, WHAT HAVE YOU DONE?!? APL (talk) 00:06, 4 March 2010 (UTC)[reply]
But despite all this nit-picking, we don't think Steve's post was lousy, right ? StuRat (talk) 22:33, 3 March 2010 (UTC) [reply]
Interesting choice of words. Would you expect your letters and punctuation to be returned to you at some point in the future?
Yes, and I'll provide a stamped, self-addressed envelope for just that purpose. :-) StuRat (talk) 22:33, 3 March 2010 (UTC) [reply]
Testing a computer for self awareness is easy! It involves two colors of paint and a mirror. APL (talk) 22:28, 3 March 2010 (UTC) [reply]
The mirror test is an interesting experiment - but it's really meaningless when it comes to AI. I can't write a computer program that will pass the test in one line of code - but given a camera and a robot arm, I could probably do it in a few thousands of lines of code. Definitely not something that requires intelligence or sentience. SteveBaker (talk) 02:00, 4 March 2010 (UTC)[reply]
Yea, I know, I was joking. Similarly, animals that can be trained to pass the mirror test are not generally thought of as having passed it. APL (talk) 02:39, 4 March 2010 (UTC)[reply]
Actually, that'd be an interesting program. The "Mirror test" has many variants, a general solution that could not be fooled by non-mirror images wouldn't be entirely trivial. Adding in all the usual computer-vision headaches like lighting issues and shadows, could make it a real challenge. The end result still wouldn't be "intelligent" in any general sense, of course. APL (talk) 07:11, 4 March 2010 (UTC)[reply]
My own opinion, the problem of artificial intelligence is constructing a computer that has intent (or even the minor form of intent called desire). It's very natural for a living creature to form an intent - even the lowest life forms seek out food, light, and other necessities of life, and humans can form complicated intentions without batting an eye, but so far the best we can accomplish with computers is programming them to make choices based on human intentions we can't yet get a computer to have an intent of its own. it will be a bit scary when we can, I think: with other humans, and even with animals, we can assume that their intents are reasonably similar to our own, but god knows what a computer would want if it had the capacity to want something. --Ludwigs2 02:55, 4 March 2010 (UTC)[reply]
A Roomba has the intelligence of a grazing animal (without its sex drive) and makes less mess. Cuddlyable3 (talk) 03:25, 4 March 2010 (UTC)[reply]
not in the least. a roomba doesn't differentiate between what it does and doesn't suck up, it doesn't make decisions about where to go next, it doesn't choose when to vacuum and when to rest (except as it has been programmed to have logic forks, or as it is turned on or off). your average Amoeba has greater intelligence than a roomba. --Ludwigs2 03:34, 4 March 2010 (UTC)[reply]
What makes you think I posess an amoeba? A Roomba and its cousin the Trilobite navigate around obstacles in a previously unknown area. As the article notes, a Roomba decides when to seek out its energy supply. It can wail for your help if it gets stuck. Is your amoeba intelligent enough to help with the housework? Cuddlyable3 (talk) 04:57, 4 March 2010 (UTC)[reply]
I have to say, when I think of "Grazing animal" I think of a relatively intelligent mammal. I suppose you're thinking of some tiny and stupid invertebrate?
Even so, From the Roomba article "Roombas do not map out the rooms they are cleaning. Instead, they rely on a few simple algorithms such as spiral cleaning, wall-following and random walk angle-changing after bumping into an object or wall. " APL (talk) 07:11, 4 March 2010 (UTC)[reply]
The kind of "AI" that computer games give to life-forms in the virtual world is very crude compared to "real" AI - and I wish we could give it another name - but it can pass a kind of 'reduced' Turing test. It's perfectly possible in some of the better networked games to be confused by who is a human-controlled player and who is a computer controlled drone. Obviously, that fails miserably when you can talk to them - but for non-verbal stuff in a game world, it's sometimes hard to tell.
Anyway - one strategy for implementing this kind of thing is called "Goal Oriented Behavior" (GOB for short) - and this approach uses a hierarchy of goals and means to achieve those goals - with weightings that force some goals to be more important than others. For a typical first person shooter, the primary goal might be to obey some game-designer-issued "order", the secondary goal might be self-preservation and the tertiary goal might be to kill human players - balanced by another goal to keep the ammunition level in his gun above zero by reloading. So it's never going to leave the precarious catwalk you placed him on - even if he's gonna get shot if he doesn't - but if he's getting shot at then he'll prefer to go and hide behind the inevitable nearby crate than to stand there like some damned idiot shooting back at you with his super-inaccurate gun at a guy with 5 lives remaining and a couple of health-packs nearby! When he has ammo left, shooting you is more important than reloading - but when he's fired his last shot, he'll reload.
Seeking weighted goals is a good way to do "intelligence" on the cheap - and it's not surprising that Computer games, Amoeba and Roomba's all use it. However, it says nothing at all about sentience or self-awareness. SteveBaker (talk) 15:59, 4 March 2010 (UTC)[reply]
The flaw with the "mirror test" is that it sets up a test that's based on human concepts of what self-awareness is, and then concludes that anything that fails the test lacks self-awareness. That's a little like setting up an IQ test and if someone scores poorly they're assumed to be of low intelligence. All such a test really demonstrates is the ability of the subject to take that particular test. And individuals can react differently. Consider television. Some dogs and cats ignore it. Some react to the sounds of animals (their ears twitch or whatever). Others will watch the images. It occurs to me that the ones that watch the images would be the most likely to pass the mirror test, as they recognize there's something there. Animals that fail the mirror test might just not comprehend what they're seeing, or as someone pointed out earlier, since the "other" creature has no scent, the dog or cat doesn't concern itself with it. ←Baseball Bugs What's up, Doc? carrots12:52, 4 March 2010 (UTC)[reply]
I suspect that most cats and dogs see a series of flickering still images on the TV, similar to what I see when I turn the refresh rate down on my computer screen. Why would they be able to resolve faster refresh rates than us ? It was probably more important to them, in their evolutionary past, since they tended to hunt smaller animals that can change direction rapidly, and even a fraction of a second delay in detecting that change could cause them to lose their lunch and maybe their life. So, those cats and dogs that do see a moving image instead of a series of still images probably just have a slower visual processing speed, not more intelligence. Interlaced images might look even worse to pets, looking like a blurry double image. StuRat (talk) 14:10, 4 March 2010 (UTC)?[reply]
Dogs rarely get interested in TV's because they live in a world of smells. It doesn't matter that what they see looks like a dog and sounds like a dog - if it doesn't smell like one, it's not a dog. Period. SteveBaker (talk) 15:59, 4 March 2010 (UTC)[reply]
I disagree. They most definitely use sights and sounds, as they work on a longer range. They tend to use smells to "verify identity" at close range. Some dogs will react to a barking dog on the TV, by going up to smell it. Then, once they smell it, they determine that it's not really a dog after all, and ignore it. StuRat (talk) 16:13, 4 March 2010 (UTC)[reply]
The editor who talked about "intent" is onto it. A machine does what it's programmed to do, and nothing more, regardless of how sophisticated it is. We have only a smidgen of understanding of how the brain works. If or when we figure that out, then we'll have a clue of how to build a truly intelligent creature - a cyborg, or whatever you want to call it. But beware of the "intent" factor. If you leave out morals and conscience, you could end up with a creature that has ambitions to rule the world, or at least to kill its creator. This is a familiar theme, somehow. ←Baseball Bugs What's up, Doc? carrots12:59, 4 March 2010 (UTC)[reply]
So would a self aware AI attempt to rewrite its own program? Googlemeister (talk) 14:15, 4 March 2010 (UTC)[reply]
Not necessarily. Do we try to rewrite our own ? I suppose learning might qualify as rewriting your own program. If so, how about an O/S that downloads new drivers to "learn" how to use new devices ? StuRat (talk) 16:01, 4 March 2010 (UTC)[reply]
You can train a Neural net program, which is kind of like that.20.137.18.50 (talk) 19:59, 4 March 2010 (UTC)[reply]
Some people do. See drug abuse, Gene therapy, immunization, Plastic surgery etc... Googlemeister (talk) 16:17, 4 March 2010 (UTC)[reply]
It might want to (just as some humans would wish to) - but that's not to say that it would be able to without doing something suicidally dangerous. The authors of a 'real' AI program would certainly want to defend against that possibility (unless they were doing some kind of strange experiment or other).
However, at some level, "rewriting" is the same thing as "learning" - and any AI would certainly need to be able to learn. Humans can rewire their neurons just by memorizing a new telephone number - but that doesn't give us the conscious ability to demand that neuron number 123,456 connect to neuron number 987,654 on demand. Similarly, the AI computer software would be able to add lines of Cyc code to it's database by memorizing a phone number - but would be unable to specifically compose a rule "(#$genls #$1-800-1234 #$PhoneNumber)". That's rather important because you wouldn't want some stray thought to cause it to write something like "((#$capitalCity #$France #$WibbleWibbleI'mATeapot)" and thereby firmly believe this to be a true fact from that point onwards. There is good reason why conscious thought can't do things like that. SteveBaker (talk) 17:00, 4 March 2010 (UTC)[reply]

anti-gravity and any violent explosion

Might not any violent explosion be a form of anti-gravity incantation or creation; a violent dismissal of gravity if you will? 71.100.11.118 (talk) 22:44, 3 March 2010 (UTC)[reply]

No. Explosions create force that can counteract gravity, but gravity is still in effect. Gravity is a relatively weak force though, so it may not assert itself noticeably (at a human scale of perception) at the moment of the explosion. —ShadowRanger (talk|stalk) 22:52, 3 March 2010 (UTC)[reply]
To cover my bases: Technically, I believe whatever just exploded will no longer have the same gravitational attraction to anything within the cloud of debris. Anything outside the cloud will still be attracted in roughly the direction of the center of the debris cloud's mass, but anything inside the debris cloud would experience a weaker pull, since the debris that was blown past them will attract in a direction opposite that of the remaining debris's center of mass. Of course, unless you're blowing up planetoids, the gravitational effects from the debris would be negligible; my explanation is just being pedantic. —ShadowRanger (talk|stalk) 22:57, 3 March 2010 (UTC)[reply]
Really technically ShadowRangerRIT you're assuming a spherically symmetric explosion which would be subject to Birkhoff's theorem (relativity), however if your explosion was large enough and wasn't spherically symmetric you would feel a gravitational effect outside of the radius of the explosion. ;)
But tbh your first answer was completely correct. Looked at in another way, if I throw a ball straight into the air, I'm not using anti-gravity, but giving the ball a momentum away from the centre of gravitational attraction. Likewise when I blow up a pack of C4, anti-gravity isn't manifesting, instead chemical (potential) energy is converting into kinetic energy which gives the particles a velocity great enough to overcome not just their gravitational attraction but also their electromagnetic attraction which in this case is stronger. To the OP check out Explosive material 82.132.248.99 (talk) 00:12, 4 March 2010 (UTC)[reply]
More or less why I used the term "...form of..." since the consequence appears to be more or less the same. In other words in an environment where gravity does not exist exterior to the explosion. Another situation where gravity is present exterior to the explosion for instance might be the center of a too massive Black Hole; the explosion resulting from an intolerable presence and resulting imbalance of gravity and its counterpart. 71.100.11.118 (talk) 00:56, 4 March 2010 (UTC)[reply]
As you wrote above, you're excited about this idea of an "opposite" of gravity, but there is no such force. See our Gravitation article. Comet Tuttle (talk) 01:30, 4 March 2010 (UTC)[reply]
LOLFDL... The chasm between the realm of what might be and the realm that is assumed to be based on Wikipedia article is profoundly demonstrated by your statement. 71.100.11.118 (talk) 01:41, 4 March 2010 (UTC)[reply]
What might be has nothing to do with fact or science or truth - if we wrote articles about what might be, this would be an incredibly useless web site. There might be pink piano playing Aardvarks hiding in caves on the dark side of the moon - but we're not going to write articles about that. SteveBaker (talk) 01:55, 4 March 2010 (UTC)[reply]
oooh, another bleak assumption that anything written about here is 100% certain and without err. 71.100.11.118 (talk) 01:58, 4 March 2010 (UTC)[reply]
No, but read scientific method. A hypothesis is only interesting if you have some evidence to show that it might be correct. Comet Tuttle (talk) 05:14, 4 March 2010 (UTC)[reply]
Is the aardvark really pink if it is not reflecting any light? Googlemeister (talk) 14:11, 4 March 2010 (UTC)[reply]
Yes. Definitely. It's axiomatic that pink aarvarks are pink. SteveBaker (talk) 15:42, 4 March 2010 (UTC)[reply]
I don't know why you'd consider a 100% correct Wikipedia to be a "bleak" outcome - perhaps you don't understand what an "encyclopedia" is all about. Certainly it's not the case that everything here is 100% correct - like any major piece of writing, it contains errors and important omissions. However, it most certainly is our goal that everything here be both 100% correct and 100% referenced back to reliable sources so that people can satisfy themselves that we are 100% correct - and so that they can spot things which might not be because they aren't properly referenced, and fix them. Hence, excluding things for which are false and those for which there are no reliable sources is important. That's not to say that we can't write about patently false things like Time Cube or Perpetual motion - so long as we provide reliable sources that explain that they do not exist. We can't write about pink piano-playing aardvarks because we have no reliable sources that say whether they do or do not exist. Whether they actually do exist is unimportant to an encyclopedia - if there is no evidence, we can't write about it. SteveBaker (talk) 15:42, 4 March 2010 (UTC)[reply]

What is leaching?

Please look at this. I put a link in the article which currently goes to a disambiguation page. None of the articles seem appropriate for the use since it's a process of nature rather than intentional.Vchimpanzee · talk · contributions · 22:48, 3 March 2010 (UTC)[reply]

I'd drop the link. It's a definition, not an encyclopedic topic per se. Maybe a link to wiktionary, but the term is relatively simple standard English and doesn't need to be linked. —ShadowRanger (talk|stalk) 22:50, 3 March 2010 (UTC)[reply]
Leachate is probably the closest article on the dab page, but it's about leaching from waste sites. The idea is the same, though. -- Flyguy649 talk 22:56, 3 March 2010 (UTC)[reply]
I see you fixed it. Thanks.Vchimpanzee · talk · contributions · 14:12, 4 March 2010 (UTC)[reply]

Reversing entropy

At the current level of technological advancement of human society in 21st century, is there the possibility that entropy could be reversed? —Preceding unsigned comment added by Crockadoc (talkcontribs) 22:52, 3 March 2010 (UTC)[reply]

With our current level of technological development? No. And in theory, never. It's the topic of a short story, The Last Question, but even the story is indulging in hand-waving speculation. —ShadowRanger (talk|stalk) 22:59, 3 March 2010 (UTC)[reply]

I added the title to the OP's question. Cuddlyable3 (talk) 01:42, 4 March 2010 (UTC)[reply]

Short answer: wait long enough. Long answer: it's equivalent to saying "is there a possibility that we could flip a fair coin x times and get heads every time?", where x is the amount of entropy you want to reverse (measured in the right units). If you try long enough, it will happen eventually, but there is no way of causing it. The concept of entropy really isn't part of science in the sense of the scientific method; it's a mathematical concept, with properties that can be proven by the H-theorem. This theorem basically says that lowering entropy is impossible unless whatever thing you want to lower the entropy of has been prepared in a very specific way beforehand (think of this as flipping a biased coin). Note that any process that prepares a system in such a way will inevitably increase the entropy of another system by at least as much as you are lowering the entropy of the first system. Also, if you're planning on "waiting long enough", you might want to see Boltzmann brain. 70.27.196.12 (talk) 03:40, 4 March 2010 (UTC)[reply]
This may be off the track a bit, but if you had a coin-tossing machine that would toss the coin in the identical way each time, wouldn't the outcome be predictable? Or are you assuming a human tossing it? ←Baseball Bugs What's up, Doc? carrots07:25, 4 March 2010 (UTC)[reply]
I would assume he is assuming a theoretical Laplace coin of infinitesimal thickness , thrown by a spherical cow in a vaccum. --Stephan Schulz (talk) 09:59, 4 March 2010 (UTC)[reply]
... and locked in a box with a cat that is both dead and alive. Gandalf61 (talk) 11:16, 4 March 2010 (UTC)[reply]
Coin tossing is a chaotic process - tiny changes in initial conditions can make big changes to the outcome. But it's almost certainly possible to control the situation enough to get a reliable answer. There was a documentary on TV a while back about people who have trained themselves to beat the game of craps by learning how to roll specific numbers on a pair of dice with much higher than usual reliability - it's entirely possible that someone could learn to toss a coin reliably too. However, in conversations of this sort, we're definitely talking about an idealized, utterly random binary outcome. Coin tossing is the spherical cow in a frictionless vacuum of the world of statistics. SteveBaker (talk) 15:17, 4 March 2010 (UTC)[reply]
It was a metaphor. I said fair coin for a reason. I guess I should have made it a link the first time, but a fair coin is no more a real coin than a cow is spherical and frictionless. 76.67.78.86 (talk) 02:52, 5 March 2010 (UTC)[reply]
You can easily reverse entropy locally. Take a box full of kiddies building blocks - carefully sort them by color and size and pack them neatly away in their box - and the entropy of the box has been reduced. However, it took energy to do that - or to state that scientifically: the low-entropy cornflakes that you ate for breakfast this morning got turned into higher entropy poop + high entropy heat. The overall entropy of the universe got a bit higher, the entropy of your breakfast got a lot higher and the entropy of the bricks in the box was lowered. We don't have any idea how to reduce the entropy of the entire universe - or any "closed system" for that matter (and it's very likely that this is totally impossible) - but we reverse entropy locally on a daily basis. SteveBaker (talk) 15:10, 4 March 2010 (UTC)[reply]
I once saw a video on youtube exemplifying laminar flow, but instead of linking that here, here's a description of it: http://www.fas.harvard.edu/~scidemos/NewtonianMechanics/ReversibleFluidMixing/ReversibleFluidMixing.html That isn't at all a counterexample of any law of thermodynamics, but it's a pretty neat and counterintuitive thing to see. 20.137.18.50 (talk) 20:08, 4 March 2010 (UTC)[reply]
There are several youtube videos showing this, this one is probably the best. It's truly an amazing thing. Strongly counter to daily experience! SteveBaker (talk) 04:14, 5 March 2010 (UTC)[reply]

Warp Drive

Is warp power in Star Trek based on actual technology or is it all made up? Does such warp drives theoretically exist? —Preceding unsigned comment added by Velderon4 (talkcontribs) 23:05, 3 March 2010 (UTC)[reply]

I'd suggest reading the article on the Warp drive. Short answer: No, there is no existing technology like that, but there are certain theories that might allow for something like it, but it's purely theoretical. —ShadowRanger (talk|stalk) 23:16, 3 March 2010 (UTC)[reply]
( After e/c with ShadowRanger)
Warp Drive is not really based on any real theory.
The closest I've ever heard of is the theoretical Alcubierre drive. Needless to say it has a variety of practical problems. APL (talk) 23:18, 3 March 2010 (UTC)[reply]
The warp drive is a very useful device for telling stories in outer space, and after having read the Alcubierre drive article, it sounds like that's about all it is. Vranak (talk) 23:33, 3 March 2010 (UTC)[reply]
Warp drive is as real as backwards time travel, using salt shakers as medical equipment, and experiencing deja vu. ←Baseball Bugs What's up, Doc? carrots04:59, 4 March 2010 (UTC)[reply]
Are you suggesting that deja vu doesn't exist? Because I've certainly experienced it, and the article accepts it as a real phenomenon (although one hard to induce under laboratory conditions) Buddy431 (talk) 05:40, 4 March 2010 (UTC)[reply]
That was a subtle hint that this question is similar to another one on the ref desk, asking if some fictional technology is for real. And all three things are "real", just not in the same way. ←Baseball Bugs What's up, Doc? carrots05:47, 4 March 2010 (UTC)[reply]
These[22][23] to name two. There's a discussion on the ref desk talk page about these questions from what could ba a series of socks or maybe just some kids fooling around. ←Baseball Bugs What's up, Doc? carrots05:51, 4 March 2010 (UTC)[reply]
... But it's not deja vu if you've literally read the same thing twice! Deja vu is when you have (Or feel that you have) two memories of an event that you know only happened once! And I'm sure that this question hasn't only happened once. APL (talk) 05:57, 4 March 2010 (UTC)[reply]
In any case, there's a group of users that are asking similar questions, basically whether Star Trek is "real". ←Baseball Bugs What's up, Doc? carrots06:02, 4 March 2010 (UTC)[reply]
An antimatter powerplant on the other hand would probably work great, the big problem is getting the fuel. Rather the biggest problem. Isolating your fuel would also be difficult. Googlemeister (talk) 14:09, 4 March 2010 (UTC)[reply]

March 4

In-text and Reference Citation for websites and textbooks

I'm aware of how to do in-text and reference citations for papers. But what is the format for websites and textbooks? —Preceding unsigned comment added by 142.58.132.87 (talk) 00:14, 4 March 2010 (UTC)[reply]

Usually questions about how to use Wikipedia belong at Wikipedia:Help desk rather than the Reference Desk, but I'm going to backstab us by answering: See Template:Cite web and Template:Cite book. In general the Template:Citation template is going to be flexible enough for anything at all, if you choose to read through the bazillion options. Comet Tuttle (talk) 00:28, 4 March 2010 (UTC)[reply]
If you are asking about how to cite things in your own papers (not on Wikipedia), it depends on the citation style you are using. (And for some styles, there simply is no official way to do it.) --Mr.98 (talk) 01:10, 4 March 2010 (UTC)[reply]

It's the same format as on paper. Why would it be any different just because the pages are digitized. —Preceding unsigned comment added by Telijelly (talkcontribs) 13:55, 4 March 2010 (UTC)[reply]

There are LOTS of reasons why it would be different in a digital world - for one thing you can have active links. You don't really need all that stuff about authorship, publishers, dates and page numbers when a single mouse click on the reference will magically teleport you to the actual document being cited. SteveBaker (talk) 14:45, 4 March 2010 (UTC)[reply]
That's fine until the target document moves or is deleted, which happens a lot online: then detailed bibliographical data becomes essential. --Normansmithy (talk) 14:51, 4 March 2010 (UTC)[reply]
That's why the DOI system was introduced - theoretically, as long as the document exists somewhere the DOI should resolve to the URL. —Preceding unsigned comment added by 131.111.185.68 (talk) 16:42, 4 March 2010 (UTC)[reply]

euthanasia

  • Though several people have commented in good faith on this discussion, it appears that the original poster is more interested in either starting a debate or promoting a particular belief. Neither is appropriate for the reference desks. I am proactively closing this discussion down. If anyone is interested in the topic, we have articles on Euthanasia and assisted suicide for further reading. Other discussion should happen at some other website. Just not here. --Jayron32 21:21, 4 March 2010 (UTC)[reply]
This discussion has been closed. Please do not modify it.
The following discussion has been closed. Please do not modify it.


If it were an animal suffering from a severe illness people would say to put it to sleep because it was the humane thing to do. However, why must a human suffer because people are unwilling to do the “humane” thing and let them die with dignity so their suffering may end. —Preceding unsigned comment added by Thekiller35789 (talkcontribs) 00:42, 4 March 2010 (UTC)[reply]

See our articles euthanasia and voluntary euthanasia. The latter says, "As of 2009, some forms of voluntary euthanasia are legal in Belgium, Luxembourg, the Netherlands, Switzerland, and the U.S. states of Oregon and Washington." The article has links to more specific articles, like Euthanasia in the Netherlands, Euthanasia in the United States, etc. Comet Tuttle (talk) 00:58, 4 March 2010 (UTC)[reply]
The real meat-and-potatoes can be found in Voluntary euthanasia#Reasons given against voluntary euthanasia. Vranak (talk) 01:15, 4 March 2010 (UTC)[reply]
was it by any chance a video of a blind dog that made you ask this? I thought exactly the same thing. All the comments were "do the right thing" and it struk me too how if that was a person, no one would say that. Vespine (talk) 05:05, 4 March 2010 (UTC)[reply]
There are many excellent arguments in favour of voluntary euthanasia and assisted suicide, but I don't think the claim that it would place humans on the same eithical footing as animals is going to be one of the most persuasive. After all, we eat animals, but that is hardly an good argument for cannibalism, is it ? Gandalf61 (talk) 09:37, 4 March 2010 (UTC)[reply]
Well, we don't generally claim that we're eating the animals for their own good. We do make that claim for euthanasia. -- Coneslayer (talk) 13:54, 4 March 2010 (UTC)[reply]
We are not really killing the blind dog for his own good, we are doing it for our own convenience, and just deluding ourselves so we feel better. Googlemeister (talk) 14:06, 4 March 2010 (UTC)[reply]
I'm not familiar with the "blind dog" video referenced above, and the OP didn't ask about blindness specifically. I was thinking of illnesses that cause pain, inability to eat, etc. -- Coneslayer (talk) 14:13, 4 March 2010 (UTC)[reply]
We recently had to euthanise ("kill") our dog because at 12 years old, she developed a lump and started to be very slow and lethargic. She turned out to be riddled with cancer. With no way to complain, she'd probably been in considerable pain for six months or more - but as our vet said "Dogs are amazingly tough - they soldier on gracefully under the most crippling pain - but when they give up, they give up entirely." A serious effort to treat her would have cost $10,000 which we simply don't have and couldn't borrow - and would probably have only maybe a one in ten chance of working anyway. Unlike human hospitals, vetenarians don't work for free - even in emergency, life-threatening cases - and the government doesn't provide ultimate fallback health care for dogs. Because neither us nor our vet speak fluent dog - it would have been completely impossible to know whether "end of life care" (painkillers) were sufficient to provide a comfortable (albeit lingering) end.
So after some debate, we paid the vet extra to come to our house, we carried the dog to her favorite spot in the back yard (she really couldn't walk anymore), settled her on her comfy dog-bed, gave her some of her favorite tit-bits (which she didn't eat) and then the vet shot her full of a strong sedative. When she fell asleep, an overdose of muscle relaxant killed her within maybe 30 seconds by stopping her heart and breathing. After we'd shed some tears, the vet took the body away for disposal. Not a bad way to go, all things considered. After a few months, we grabbed a 7 week old puppy from the animal rescue center. The old dog died after a good life and one less puppy had to be euthanised at the rescue center - a net win for dog-kind in general. It's amazing how much the act of repeatedly cleaning dog poop off your living room carpet and finding your best leather shoes reduced to 3cm x 3cm pieces all over the bedroom floor helps you get over the loss of an old friend.
However, for humans who are suffering like that we have the financial support to at least make an effort to cure serious diseases and failing that, we can generally interact with the patient to give them at least some measure of relief from pain in a hospice-like setting. There is still a case for euthanasia ("killing") people who have had enough and are able to clearly say so - but there are certainly legal and ethical hurdles - as our articles indicate. It is unlikely that the loss of an adult due to some terminal disease will result in a family adopting a child as a replacement...and children that are not adopted are not routinely executed by the government.
So there is zero comparison between the two circumstances.
SteveBaker (talk) 14:42, 4 March 2010 (UTC)[reply]


there certainly is comparison, its not all about what the family wants either. if a person lives alone and has no family. if they are blind, paralyzed , in severe pain and allergic to opiates. they should have the right to be killed. —Preceding unsigned comment added by Thekiller35789 (talkcontribs) 15:01, 4 March 2010 (UTC)[reply]

Euthanasia is less the right to take your own-life than it is the right to allow someone else to take your life for you. Be that the state, a relative, a doctor or whoever. Allowing it introduces risks. Risks of systematic abuse, risks of misunderstandings, risks of coercion. On the face of it the idea of allowing euthanasia feels reasonable, feels like a good idea. Thinking through the logical and potential reality is where I (just my opinion) find it gets much more 'grey' and difficult. The key obstacle I believe is whether any party involved will be able to make a fair judgement. Relatives might want to put someone out of their misery because from the outside looking it that sort of a 'non existence' looks miserable, but what if that person doesn't want it but cannot say? What if the person said 10 years ago "if i'm in this position kill me" but once they're in the position they realise they want to live but cannot say? It's really not anything like as simple as you (thekiller35789) are seemingly making it. It's an incredibly complex issue with (for me) compelling arguments on both sides. ny156uk (talk) 17:11, 4 March 2010 (UTC)[reply]
The OP persists in using this Ref. Desk to argue their support for euthanasia viz. "a person..should have the right to be killed". The question "Why must a human suffer?" finds varying answers within various belief systems and is therefore unanswerable neutrally. The OP uses a rhetoric device to fill the vacuum of that unavailable answer with their loaded contention "because people are unwilling to kill them". The underlined action is what is meant but the OP does not admit that. Instead the OP substitutes the weasel worded advocacy "do the "humane" thing and let them die with dignity". That prevarication assumes meanings of humane and dignity that only advocates of euthanasia apply in this context. "Let them die" is a transparent euphemism for kill which by definition is to prevent a life running its natural course. Claiming a justification for the killing to be "so their suffering may end" is direct advocacy of euthanasia. The OP's tendentious question is already handled by the Wikipedia reference that Vranak provided. This section should not be held open for the on-going debate that the OP seeks. Cuddlyable3 (talk) 21:07, 4 March 2010 (UTC)[reply]

Vectors

It's my understanding that, in order to prove that some quantity with direction is a vector, one must show that it is commutative and rotationally invariant. First question: are these the only two criteria? Second question: I've been able to show that the angular velocity vector is commutative, but how would I be able to show that it's rotationally invariant. Finally, how would one show that torque is a vector. Since torque is just a cross product of two other vectors, I quess this would reduce to proving that the cross product of two vectors produces a third vector, but I haven't been able to find a proof of this online. Thanks! —Preceding unsigned comment added by 173.179.59.66 (talk) 04:32, 4 March 2010 (UTC)[reply]

I'm sorry, but nothing that you wrote there makes any sense. Vectors are neither commutative (only operations can be commutative) nor rotationally invariant. The fact that the cross product of two vectors produces a vector is obvious from the definition of the cross product. I'm afraid that you're so far from understanding what you're doing that you might have to go back and relearn the basics of what a vector is. Looie496 (talk) 08:07, 4 March 2010 (UTC)[reply]
1) Sorry, I meant commutative under addition. 2) How can a vector not be rotationally invariant? That would mean an equation like F = ma would depend on the choice of axis. 3) To my knowledge, the cross product is defined as a determinant of some sort. The result of this computation produces something of the form ai + bj + ck, but that doesn't mean it's a vector.173.179.59.66 (talk) 08:35, 4 March 2010 (UTC)[reply]
Looie is right. What you are talking about is usually refered to by physicists as covariant under rotations. The Vectors are covariant under rotations and the vectorial equations are invariant under rotations. Dauto (talk) 19:42, 4 March 2010 (UTC)[reply]
That's funny, because this entire time I was mixing covariant and invariant, really sorry about that! —Preceding unsigned comment added by 173.179.59.66 (talk) 20:25, 4 March 2010 (UTC)[reply]

Have a look at http://en.wikipedia.org/wiki/Vector_%28geometry%29#Vectors.2C_pseudovectors.2C_and_transformations . The components of vectors should transform like coordinates under rotations. It is my understanding that a cross product is actually a pseudovector but don't quote me on that. —Preceding unsigned comment added by 157.193.173.205 (talk) 08:31, 4 March 2010 (UTC)[reply]

I think you should close this question and ask it again on the math reference desk. Those guys will nail this one very quickly. SteveBaker (talk) 14:12, 4 March 2010 (UTC)[reply]
Something doesn't need to have direction to be a vector. The definition of a linear vector space is http://en.wikipedia.org/wiki/Vector_space#Definition Basically the space must have closure, it must be distributive in the scalars, scalar multiplication must be associative, Addition must be commutative and associative, there is a null vector and an inverse under addition for each vector. For instance operators can be treated as vectors.
On another point vectors can be commutative, in a space of Real vectors all vectors are commutative because < A | B > = < B | A > * = < B | A >

82.132.248.82 (talk) 23:13, 4 March 2010 (UTC)[reply]

Who can make the definition of "Space" understandable to users with a high-school education?

I have chased down every (blue) referenced term in both the Space page and across numerous pages in a vain attempt to gain an understanding of the true understanding of the nature of 'space'. I accept the mathematical assumptions made in much the same way as a theoretically dimensionless point is defined for the purposes of function and equation. However, the original "Space" page left me with no descernable improvement in my understanding of both the existential nature of 'space' nor the efficacy of the many clumsy analogies that are proffered in support of meaning. I have read much of the arguements regarding the nature of 'expansion' with reference to space and the balloon analogy is a case in point: To declare that the balloon represents the expanding Universe and then stick an ant on the outside to argue that its corporal structure is unaffected by the expansion of the balloon universe is not helpful. The balloon analogy only works when assuming that the Universe is represented by the balloon; to add an ant (on the outside) is to break the very conventions of the analogy itself by introducing an ant (presumably) outside the Universe!

What would help users like me (and there are many - I assure you) is a comprehensive AND comprehensible explanation of the fundamental nature and function of space in both its physical and theoretical applications. For instance:

  1. If space is a vacuum, or a volume in which nothing exists; how does gravity bend it?
  2. If space is truely a state of non-existence, a vacuum within which nothing exists; surely this definition is based upon the attribute of 'existence' being the sole property of matter, as theories abound of zero-point quantum energy fields and some empirical evidence to support the idea that the Universe is filled with energy in all manner of modes and quantities, etc.
  3. If space is merely a mathematical species of frames or matrixes that provide quantative relevence to various behaviors and transformations at macro, micro and quantum scales - what really causes space lensing??
  4. The elevated circles within which the aether was argued and eventually discarded were not successful in transmitting a truely understandable analogy of this conclusion to the satisfaction of those of us that work with the real Universe. Such ideas may be erroneous but they are also intuitive - to existing generations as well as to our venerable ancestors. Wikipedia is at its best when it conveys to the masses a wholesome and digestible explanation of these endemic misunderstandings.

Finally, quantum physics has introduced us to the concepts of granularity and "forbidden states" that lie between manifest energy levels at sub-atomic scale; however, (in my clumsy attempts to consolidate their analogies and explanations) space appears to be devoid of any such constriction. We are left to ponder such (intuitive) conundrums as:

  1. Is space confined to a resolution that must obey quantum rules?
  2. Can string-theorists simply magnify the dimensions of particle physics by a billion and claim to have discovered a true and final base for the smallness of things?
  3. Does matter really move; or does it simply emerge, first in a quantum spacepoint (A), then sink back into the quantum sea only to manifest again in Spacepoint (B) - that just happens to be the adjacent location closest to (A) - thereby giving the illusion of motion yet simply being a series of manifest existences bounded by the granularity of space; time; and energy/matter?

Scientists have done a wonderful job of improving upon the once clumsy definitions provided for the concept of "Energy" in this Wikipedia; I hope that a comparable attempt will be made to de-mystify the nature of "Space".

GPCViriya (talk) 09:46, 4 March 2010 (UTC)[reply]

Wow, that's a lot to take on. Let me just answer one small part of it by saying that space is not a pure vacuum. It certainly contains small densities of ordinary matter, and quite possibly other forms of matter such as dark matter. StuRat (talk) 13:53, 4 March 2010 (UTC)[reply]
OK - well, let me try to answer some of these:
  1. If space is a vacuum, or a volume in which nothing exists; how does gravity bend it? - In general relativity, matter bends space and bent space produces the illusion of gravity. That's only one view, but it's the prevailing one. Why matter bends space is probably an unanswerable question. It just does.
  2. If space is truely a state of non-existence, a vacuum within which nothing exists; surely this definition is based upon the attribute of 'existence' being the sole property of matter, as theories abound of zero-point quantum energy fields and some empirical evidence to support the idea that the Universe is filled with energy in all manner of modes and quantities, etc. - Space (as described by physicists and cosmologists isn't the same thing as 'vacuum'. It's kinda like the sheet of paper on which a diagram is drawn. The diagram is matter and energy - the paper is "space". It's the underpinning of what we perceive as "distance" or "position". The physicists concept of "space" is present even inside solid bodies. Now, if we're talking about "space" as (say) an astronaut might talk about it (the gaps between solid things that's mostly vacuum) then that's really something different. The "vacuum" of deep space isn't really empty - there are a few atoms per cubic meter, also lots of photons and all sorts of other exotic things like virtual particles.
  3. If space is merely a mathematical species of frames or matrixes that provide quantative relevence to various behaviors and transformations at macro, micro and quantum scales - what really causes space lensing?? - Gravitational lensing can be thought of as the consequences of bent space (see question (1), above). Space is bent - light takes the shortest path through it and the shortest path across a bent surface isn't a straight line (kinda like aircraft flying long distances across the Earth travel along "great circle" routes because they are less distance than "straight lines"). To our senses - which can't directly comprehend the curvature of space, the light appears to bend.
  4. The elevated circles within which the aether was argued and eventually discarded were not successful in transmitting a truely understandable analogy of this conclusion to the satisfaction of those of us that work with the real Universe. Such ideas may be erroneous but they are also intuitive - to existing generations as well as to our venerable ancestors. Wikipedia is at its best when it conveys to the masses a wholesome and digestible explanation of these endemic misunderstandings. - The aether theory was just wrong. It's a discarded hypothesis because it simply doesn't fit the facts. It may be intuitive - but it's a useless explanation because it doesn't explain the behavior of light.
Quantum physics questions:
  1. Is space confined to a resolution that must obey quantum rules? - There is a minimum unit of distance which (I suppose) you could think of as the "resolution" of space.
  2. Can string-theorists simply magnify the dimensions of particle physics by a billion and claim to have discovered a true and final base for the smallness of things? - No. String theory is much more complicated and radically different from the 'normal' view of particle physics.
  3. Does matter really move; or does it simply emerge, first in a quantum spacepoint (A), then sink back into the quantum sea only to manifest again in Spacepoint (B) - that just happens to be the adjacent location closest to (A) - thereby giving the illusion of motion yet simply being a series of manifest existences bounded by the granularity of space; time; and energy/matter? - No, matter moves.
SteveBaker (talk) 14:08, 4 March 2010 (UTC)[reply]
Regarding the balloon analogy: the surface of the balloon is analogous to space. As the balloon inflates, the surface expands. The ant represents a point (or a set of points near each other) on the surface of the balloon. Zain Ebrahim (talk) 14:17, 4 March 2010 (UTC)[reply]
It may be easier if you don't think of matter as something that's "in" a vacuum, but rather think of it as part of the nature of the vacuum. If you look at general relativity by itself, without any matter, it's still a highly nontrivial theory—you have gravitational waves that propagate around and interact with each other, just like any other kind of wave might do, even though there's nothing "in" the space. In the Standard Model of particle physics, which is the best current theory of everything other than gravity, everything other than gravity is like this too. The "particles" are various kinds of oscillations of the vacuum. This is enshrined in the Standard Model in a fairly deep way, through the concept of spontaneous symmetry breaking. The nicely symmetric fundamental vacuum state of the Standard Model is unstable, so, at low enough energies, the vacuum ends up in a different, stable state at some "distance" (in the landscape of possibilities) from the center of symmetry. The particles of everyday life are small oscillations around that state, and they're only indirectly related to oscillations around the unstable center. When particle physicists say "vacuum", they mean a small neighborhood of an equilibrium state, or, in other words, the vacuum proper and all of the low-energy particles and their interactions. For example, when they talk about a "landscape of string vacua" and trying to find a vacuum that corresponds to our world, that's what they mean.
At least this means we don't have to understand space as a container for matter; we only have to understand it as a thing unto itself. Unfortunately, that's the best you're going to get right now. Space behaves like a multidimensional continuum that can be deformed in various ways, and nobody knows anything more than that. There have been many extremely speculative attempts to give it another structure, which go by the name of pregeometry. Probably the best developed of those ideas is spin foam, about which I know nothing beyond what's in the Wikipedia article. -- BenRG (talk) 02:37, 5 March 2010 (UTC)[reply]


Thank you,
(StuRat): You are saying that space is seldom (or ever) found in a pure state of vacuum; however, I understand that matter - ordinary, dark or exotic is not included in the definition of the nature of space, it simply exists at a certain point in space, manifesting its own family of characteristics and behaviors. Therefore, you are describing vacuum as a condition relating to the ratio of matter to the volume of space - not describing the fundamental nature of space (in itself).

Thank you, (SteveBaker):

  1. The unanswerable question, (as how matter bends space) does not eliminate the underlying arguement of my question. My premise was; presuming that space (in its own nature) has no attributes, how can any thing bend it? When one refers to the change in shape (from flat to bent), under the influence of gravity; it is assumed that space (itself) has been influenced by the proximity of matter, which possesses an attribute of mass, which manifests a force called gravity, and has deformed space to reflect the influence of this force upon the very fabric of space.
  2. Your comments (in 2) address my question in point 3. I assume that by paper, you mean physicists rely upon the paper to illustrate the positions and/or co-ordinates occupied by particles within a certain arrangement in space? This accords with my own understanding - that space is used as a foundation for the deliniation of such properties as; position, motion, etc. These properties are not ascribed to space, but to the particles, etc,. that are included in the diagram. Your extra comments regarding the condition of inter-stellar space returns to your earlier comments regarding the nature of a vacuum and do not directly address the nature of space.
  3. In 3, you return to the point, that space is bent and light takes "the shortest path through it" thereby attributing some characteristic to space that is both bent and exerts an influence upon light to respond to this condition by following a bent path. However, this characteristic of space is left undefined albeit your contention that, whatever it is - it is bendable.
  4. I presume that you are referring to the particle/wave characteristics of light when you mention that the aether hypothesis has been proved wrong. I have no arguement with this; however, it does help to illustrate my point regarding the unintended confusions created by using analogies that do not eliminate the underlying inference for just such a medium. By this, I mean; any and all mention of bent-space has the immediate effect of creating the idea of space as a thing, or medium within which the behavior of particles, under certain conditions, can only be explained by pertubations or deformations in the fabric of that space, medium or thing. Creating a vacumm to demonstrate the passage of light without the support of a medium does not eliminate the possibility that space, itself, has the characteristics of a medium. Simply removing all other particles from a region in space does not change the nature of the space itself. A vacuum is not devoid of space any more than the volume contained within the body of a sub-atomic particle is devoid of space, regardless of its mass.
  5. To my quantum physics questions, you peaked my interest when you mentioned the existence of a theoretical minimum distance; then you simply discounted any significance to this assertion when you stated that "matter moves". Clearly both cannot be right. If matter was required to move only a fraction of the "minimum distance", it could not - then we would have to deal with all the potential and/or kinetic energies that may still require transition. If the matter was to move only 1 unit of the minimum distance; it could only achieve this (not by an analogue re-positioning) but by a cessation of existence in one location and a subsequent manifestation at the next, or nearest location (as any smaller movement would be prohibited).
  6. String theory may appear to be more elaborate and complex than particle theory, but it still must address the fundamental characteristics that have been thus far observed. In essence, (to the ear of a layman like myself); string theory has replaced the elementary particle with a field of strings, at a greatly reduced magnitude in both scale, momentum and inertia, and used the more numerous unique characteristics of a string (as opposed to a particle) in their analogies - to ascribe the various configurations and resonant conditions in the stings to represent the observed momentums and behaviors of higher species of elementary units.

I am in no position to question the verasity of the science, experimentation, and theoretical conjecture that is contained in either the Standard Model nor the fledgling String Hypotheses; however, neither side has provided the general population with any working analogies that withstand even the most precursory analysis. Although such examples are not meant to explain the complete picture; they actually pollute the mind of the thinker when these analagies are pushed too far.

Zain Ebrahim's opening comment is a case in point: Given that the analogy of the balloon is representative of the fabric of space and its inflation indicative of the effects of expansion: the ant may represent either a point, or several points in the aforementioned space. Assumed to be a point - the ant experiences no change; as a point is considered to be dimensionless and therefore immune to any change in scale happening around it. Assumed to be a collection of points where the feet touch the surface of the balloon - the ant will find its feet becoming ever more widely spread the longer the expansion of the balloon continues. These are necessary considerations that require the layman to first accept that an ant, outside the balloon, is still in the Universe that has been defined as the surface of the balloon. In fact, we need to discount both the volume of air within the balloon AND the volume of air outside the balloon to focus upon the real efficacy of using the balloon surface as an analogy for an expanding Universe. Introducing the ant invalidates the analogy completely and all subsequent arguements regarding the state of the ant serve only to obfuscate the intended meaning of the exercise. I think the original arguement was concerned with applying a fixed, scalar reference inside an expanding universe. What changes are observed to the space, matter and the arrangement of matter within that space and the influence that the expansion of the universe was having upon both the nature of the matter AND the measurements of their locations and dimensions relative to each other. I presume that the balloon analogy will not be helpful in resolving that one.

Finally, BenRG brings in a veritable plethora of ideas that both illustrate and clarify the nature of my arguement and afford me the chance to zoom in on the specific point that bothers me and is keeping me from appreciating the work that theoretical physics is doing:

  1. The dicotomy between vacuum and matter is not the crux of my questions regarding the nature of space. Any and all descriptions of the interaction between particles (both mass-carrying and force-carrying) may be conceived of as occurring in space without the need to interact with the space in any way. For example; the number of particles, the ratio between their energy levels and their freedom to move within a defined space can be used to calculate their density and even their pressure within a confined space and this can be simply illustrated by analogy without referring to the nature of space - apart from the volume included in the observation. Such a description treats space as a mathematical matrix or yard-stick by which to measure and subsequently observe inherent characteristics of both quantity and quality or behavior. Such observations and calculation do not ascribe any qualitative characteristics to space, only volume is used to provide scale and quantity.
  2. BenRG goes on to provide a whole list of potential waves and particles with which to describe phenomena that do not rely upon the presence of space to contribute to their behavior at all. Regardless of our choice to refer to gravity as a wave, a particle, or a wavefunction; any choice is valid and is all that is required to describe the effect it will have upon another particle. The trajectory of a photon is effected by its interaction with a graviton, gravity wave, or the mass of an attendant atomic or sub-atomic particle, which is included in the observation, and can be predicted, tested and described without the use of space as anything other than a location in which to observe and measure the interation. The Standard Model contains sufficient species of particles to carry any and all of the fundamental material and force-characteristics that exist in the Universe bounded by space, without having to assign any of these behaviors to space itself.
  3. In essence, we do have to think of space as a "container"; with no unique characteristics nor interactions with matter or force, and useful only as a field of observation, manifestation and quantification of all other species of behavior and existence. Any distortions to this "multidimensional continuum" are by inference alone, they can be explained by interations between particles and/or wavefunctions that describe the inherent characteristics and behaviors of specific modes of energy, manifesting on any level, from the quantum sea of virtual-quarks up to the distribution of galaxies within the Universe.

In conclusion; I see no need to include space as a discreet entity of existence, replete with unique characteristics that are essential to the orderly interaction of all other entities contained in the Standard Model. It appears simply to act as an inert background for manifest existence and only makes an appearance (as the 4 dimensions) when observations and measurements are required. So far, science has failed to convince me (an uneducated but rational humanbeing) that space is a term that describes anything other than the mind's ability to differentiate both scale and distance in reference to an observation of discreet objects.

Respectfully,

GPCViriya (talk) 09:33, 5 March 2010 (UTC)[reply]

Two hearts

Has there ever been a recorded case of a human having two hearts? —Preceding unsigned comment added by Kandorko (talkcontribs) 10:38, 4 March 2010 (UTC)[reply]

This might be interesting [24] Also this paper if you can find it: [25] Aaadddaaammm (talk) 11:53, 4 March 2010 (UTC)[reply]
Given a single character typo I was initially torn between "two hearts", "two ears" and "two bears" - fortunately, Aadddaaammm read the title and has the answer covered! SteveBaker (talk) 13:44, 4 March 2010 (UTC)[reply]
Not sure if these count:
Zain Ebrahim (talk) 14:34, 4 March 2010 (UTC)[reply]
The Hensel twins have two hearts and one lower body, but that might not count as they are not "a" (single) human. ←Baseball Bugs What's up, Doc? carrots14:47, 4 March 2010 (UTC)[reply]

Inverted crater on Mars

I came across this image from the Mars Reconnaissance Orbiter. The image description calls it an inverted crater. What is an inverted crater? Is it just a round-shaped plateau that just happens to look like a crater? According to inverted relief, natural processes on Mars can cause features like river beds that were once depressions to end up above the surface. Is this another example of that? —Preceding unsigned comment added by 72.94.164.21 (talk) 12:33, 4 March 2010 (UTC)[reply]

[26] (the official NASA version of the Yahoo story with actual facts and stuff) says that the crater was originally a regular crater that filled up with sediment which compacted and became harder rock than the landscape from which the crater was formed - then erosion eroded away the softer crater walls and the surrounding land to leave the compacted sediment as this "inverted crater". A similar kind of thing happens on earth when a volcano erupts in soft material - then the lava hardens into a really solid rock like basalt - and then the soft ground around it erodes leaving a column of harder basalt. The name for those things escapes me for the moment - but I'm sure someone here will tell us. SteveBaker (talk) 13:33, 4 March 2010 (UTC)[reply]
Were you possibly seeking Intrusion or Volcanic plug? -- 124.157.247.225 (talk) 14:53, 4 March 2010 (UTC)[reply]

Inverted relief —Preceding unsigned comment added by Telijelly (talkcontribs) 13:52, 4 March 2010 (UTC)[reply]

American Cat Scans

Because of the sterotype that americans are fat, there was always this joke going around that amerians would get stuck on cat scan machines. Is it true than quite the contrary, the manufacturers made the machines large to accommodate fat people, but in doing so really thin and small people were unable to use the machines because it was scanning air around them and giving false resulte? —Preceding unsigned comment added by Gordanginton (talkcontribs) 13:26, 4 March 2010 (UTC)[reply]

No. First, I doubt the premise of your question. Second, the machines can scan whatever is within their circumference irrespective of whether that happens to be air or flesh. --Tagishsimon (talk) 13:32, 4 March 2010 (UTC)[reply]
My wife used to work as a nurse in the radiography department of a large hospital - she said that they had on occasion cat-scanned premature new-born babies - which are about the smallest thing you'd ever consider trying to scan - and that worked perfectly. So no - this is flat out not true. SteveBaker (talk) 13:36, 4 March 2010 (UTC)[reply]
does she still work as a nurse, just not in radiography anymore? What department is she working in now? I'm asking because I know someone in a similar situation and wonder what a good switch is from the radiography department. Thanks. —Preceding unsigned comment added by 84.153.250.71 (talk) 14:49, 4 March 2010 (UTC)[reply]
I work for a medical device company whose name doubles for a type of screwdriver, and in recent years our marketing materials have been completely written to emphasize the usefulness of our new products in handling "difficult patients" -- this is marketing-speak for fat/overweight/obese however you choose to describe it. Simply put, for something like an ultrasound it just takes more juice to get a good picture when there's more "material" between the sensor and the object you wish to image. 61.189.63.188 (talk) 14:37, 4 March 2010 (UTC)[reply]
You work for Flathead?? ike9898 (talk) 14:45, 4 March 2010 (UTC)[reply]
Or maybe Stanley ? StuRat (talk) 15:58, 4 March 2010 (UTC)[reply]
But there must be limitations on that method. Specifically, doesn't the near flesh get an increased exposure level ? StuRat (talk) 15:55, 4 March 2010 (UTC)[reply]

Patient size is a bigger factor for an MRI machine than for a CAT scanner, because the device benefits a lot more from having the sources and sensors close to the subject. MRI machines are notorious for causing claustrophobia. Looie496 (talk) 17:09, 4 March 2010 (UTC)[reply]

I was going to ask about that. When I was in grad school, the neuroscience/cognition folks had 3 sizes of MRI machines. I think they had one for humans, one for smaller primates, and one that was smaller yet (dogs? rodents?). I was wondering if the smaller machines worked better on the smaller animals, or if they were just less expensive. -- Coneslayer (talk) 17:13, 4 March 2010 (UTC)[reply]
I would think that 1 human sized MRI device would be cheaper then 3, regardless of the size of the others, but if there was a low % of machine downtime, it makes more sense. Googlemeister (talk) 17:30, 4 March 2010 (UTC)[reply]
A rodent-scale MRI can get resolution down to a tenth of a millimeter, as I recall, which is 10 times better than you get with an ordinary human MRI, so there are definite advantages to having a separate machine if there is money to buy it. Looie496 (talk) 17:52, 4 March 2010 (UTC)[reply]
The logical conclusion, then, is that to do a first rate job of analyzing a foot problem, they should saw it off, run it through a rodent MRI machine, then sew it back on. Among other benefits, this approach would likely reduce the number of people requesting scans for brain tumors. :-) StuRat (talk) 19:01, 4 March 2010 (UTC)[reply]

Side issue: Was that a correct use of the word steriotype? Americans have afterall really become on average quite overweight. Dauto (talk) 20:59, 4 March 2010 (UTC)[reply]

I've heard stereotype used to mean "any characteristic ascribed to a group of people, whether true or not", so it sounds correct here, to me. StuRat (talk) 22:40, 4 March 2010 (UTC)[reply]

The only limits are upper limits. Throughout the 1980s and 90s, most US CAT scan machines had an upper limit capacity of 300 lbs: many radiology depts or imaging centers would refuse to image someone who was heavier. Partly it was fitting in the cylinder and partly it was the warranty limit on the moving parts of the machine. There are no lower limits to size except resolution limits. alteripse (talk) 21:11, 4 March 2010 (UTC)[reply]

Compressing Water

Can water be compressed? How much of a space saving will be achieved from compressing water to it's maximum —Preceding unsigned comment added by Harodeuam (talkcontribs) 14:05, 4 March 2010 (UTC)[reply]

Yes, but not to any significant degree. See our article on the properties of water for details, but note that at a pressure of 400 atmospheres, water retains over 98% of its original volume. — Lomn 14:13, 4 March 2010 (UTC)[reply]
The article mentions oceans at 4 KM. I wonder if there is any info about the compression at around 11 KM, the approximate depth of the Mariana Trench? As I recall from physics class, the reason objects can be crushed at great depths is not so much that the water is "denser", but that there's a column of water several miles high crushing it. But I would think that weight would also impact the density of the water itself - even if only to a small extent? ←Baseball Bugs What's up, Doc? carrots14:40, 4 March 2010 (UTC)[reply]
The article also says that the compressibility decreases as the pressure increases. So if you get 1.8% compression at 4 km depth, the compression at 11 km must be more than that, but less than 5%. --Anonymous, 01:21 UTC, March 5, 2010.
Assuming that doesn't push you past a phase transition to one of water's many allotropes, I think you mean. Not that I've checked whether any of those are much denser, or whether the pressure at 11 km is even close to enough. --Trovatore (talk) 01:26, 5 March 2010 (UTC)[reply]

I got a related quesiton guys. If you don't store water, but only the hydrogen and the oxygen that you could combust together to produce it, then can you compress THOSE gasses in a way that is equivalent to compressing water? e.g. if you have an x cubic meter room, can you get x + y cubic meters of water to flow out of it, for an appreciable amount of y, not because you had tanks of compressed water in it (which would be very difficult, and the y would be tiny) but because you had tanks of compressed hydrogen and oxygen in it, and combined them to produce your water flow? 84.153.250.71 (talk) 14:44, 4 March 2010 (UTC)[reply]

Water has 2 grams of hydrogen per 18 grams of water, which is 18 millilitres, (See molecular formula and atomic weight). By Avogadro's Law, 2 grams of hydrogen under standard pressure and temperature would occupy 22.4 litres, which is 1,244 times the volume of the water it would form when burnt with atmospheric oxygen. The oxygen required would take up 11.2 litres, if it is not available locally. The burning of hydrogen will also produce very high temperature steam, which would then need to be cooled down. So in answer, no, it is not practicable to carry just the hydrogen. CS Miller (talk) 15:15, 4 March 2010 (UTC)[reply]
Let's see... at standard temp and pressure, it would take 33.6 liters to store 18 grams of water-equivalent. Assuming that we didn't vary temperature, then, you've got to crunch 33600 milliliters to 18 milliliters to get to water volume equivalence. That would be storing the gases at over 1860 atmospheres. Clearly, this is less practical than just storing water. — Lomn 15:04, 4 March 2010 (UTC)[reply]
I'm sorry, I don't get the "upshot". If I have an x cubic meter room underwater (no ambient oxygen source) and I want to store MORE than x cubic liters of water in it, is it better to try to compress water (maybe also cooling it as much as possible short having it freeze and expand) or to try to store it separated, as compressed, cooled gases hydrogen and oxygen? Assume you can just take the heat from combustion and boil the outside water with it... 84.153.250.71 (talk) 15:23, 4 March 2010 (UTC)[reply]
p.s. obviously there is no need to store water in a room underwater. I'm just askin'.. 84.153.250.71 (talk) 15:23, 4 March 2010 (UTC)[reply]
I doubt it. Reacting 29.5 litres (2kg) of liquid hydrogen with 14 litres (16kg) of liquid oxygen will produce 18 litres of water. I don't have the density of frozen hydrogen or oxygen handy. CS Miller (talk) 16:05, 4 March 2010 (UTC)[reply]
There could be a reason. If you want drinking water and you're under the ocean, you'd need to desalinate it and sterilize it before drinking. Whether the apparatus to do this takes up more space than the stored water depends on the duration of the stay as well as the technology used. This is a similar trade-off to space ships either carrying water or recycling water from breath, urine, and poo (when the water comes out brown, it's time to change the filter). StuRat (talk) 15:51, 4 March 2010 (UTC)[reply]
(ec)If you read Lomn's response, he is saying that in order to get the hydrogen content of water down to the same volume as the water (before even thinking about the oxygen), you'd need to produce an ungodly amount of pressure. So yes, if you can produce, say 3000-some atmospheres of pressure, you might be able to store it in a smaller volume as independent gasses (though the water itself would likely have compressed a bit more at that point). At a certain point, hydrogen would enter its metallic phase which is basically as a compressible as it gets without triggering fusion. Since you're not going to produce that amount of pressure just to store water, we can safely say you're better off just storing the water. —ShadowRanger (talk|stalk) 15:52, 4 March 2010 (UTC)[reply]
Oxygen condenses at room temperature under sufficient pressure, doesn't it? It will become essentially incompressible at that point too. I have no idea what that point is, though. --Tango (talk) 04:45, 5 March 2010 (UTC)[reply]
No. According to our article, the critical temperature of oxygen is about −118 Celsius. Above that temperature, no amount of pressure can liquefy it. Still, sure, you can compress it to the point where the atoms are more or less touching, and even though it's not a liquid, it will still be almost incompressible at that point. --Trovatore (talk) 06:25, 5 March 2010 (UTC)[reply]
If you put oxygen under great pressure, it would behave like a liquid, but you could not get a surface on it. So the resistance to pressure would climb. If you just compress water enough you will get a form of ice, at about 2000 atmospheres. Forming metal by compressing hydrogen or oxygen is much more difficult needing millions of atmospheres of pressure. Ice III has a density of 1.16. So 15% above water. Graeme Bartlett (talk) 09:02, 5 March 2010 (UTC)[reply]

What type of medical specialist does this?

This might seem like a request for medical advice, but really it isn't...

The type of medical insurance I have allows me go directly to a specialist without first going to a primary care physician to get a referral. I have an injury which I suspect is something along the lines of a injured upper back muscle (either a small tear or separation from the bone). So, what sort of physician specializes in this sort of problem?

ike9898 (talk) 14:43, 4 March 2010 (UTC)[reply]

Orthopedic surgery --Normansmithy (talk) 15:08, 4 March 2010 (UTC)[reply]
It's really impossible to decide yourself what the actual cause of a given sensation is. I had a similar shoulder problem, which felt like a joint or tendon malfunction. My doctor diagnosed it, however, as muscular adhesions, and sent me to the physiotherapist. The physio cleared it right up. Figuring out which specialist is appropriate is exactly the job of a primary care physician. -- Finlay McWalterTalk 21:00, 4 March 2010 (UTC)[reply]
No argument, but I'm sure you can see the appeal of possibly reducing the number of doctor appointments you have to go to. Of course, if I initially go to the wrong specialist, that won't save anybody any trouble! ike9898 (talk) 21:46, 4 March 2010 (UTC)[reply]

Plasma

How is Plasma made? —Preceding unsigned comment added by Harodeuam (talkcontribs) 14:45, 4 March 2010 (UTC)[reply]

Are you referring to blood plasma or Plasma (physics), a state of matter comprising partly ionized gas? Blood plasma is made up of lots of different components from different sources: proteins, hormones, glucose, water, etc. Plasma (physics)#Common artificial plasma has information on making the other sort. --Normansmithy (talk) 15:14, 4 March 2010 (UTC)[reply]
(EC)Plasma (gas) or Blood plasma (the non-cell part of blood)? (there are other plasmas on the diambiguation page, but these two are the most likely). I suggest looking at those articles. -- Flyguy649 talk 15:16, 4 March 2010 (UTC)[reply]

Sun size

Daft question i'm sure but...is the sun noticeably 'larger' with the naked eye when you're closer to the equator? The moon obviously varies in how far away it is and it's very noticeable when it's larger, but sitting here in sunny (for today) England fully aware that those near the equator are 'closer' to the sun I was wondering whether that effect is noticeable. I'd recommend some people try but staring at the sun isn't that great for your eyes. ny156uk (talk) 17:22, 4 March 2010 (UTC)[reply]

Well, let's think about the scales involved. The sun is somewhere between 90 and 100 million miles away. How much larger do you think it would look if you were 4,000 miles closer (.004%)? This would not be a large enough difference to be noticeable to your average human. Also, the distance from the sun varies by much more then that amount throughout the year because the orbit is not a perfect circle. The moon on the other hand is only about 250,000, so 4,000 miles is around 2%, and might be noticeable. Googlemeister (talk) 17:26, 4 March 2010 (UTC)[reply]
(EC) There would be no difference detectable. The earth's mean radius is 6371 km. The mean distance of the earth to the sun is 149.6 million kilometres. Ignoring the 23° tilt of the earth's axis, someone at the pole is one earth radius farther from the sun than someone at the equator. That's 0.004 percent. I doubt there is a noticeable difference in the size of the sun from perihelion to aphelion. -- Flyguy649 talk 17:32, 4 March 2010 (UTC)[reply]
Also, the width of the Earth may vary by 4000 miles from the poles to the equator, but would vary far less between the equator and England. I'd guess maybe 1500 miles (or 0.0015%), but invite others to do the math. Note that the variation isn't linear, meaning the width of the Earth varies far less from the equator to 45 degrees than from 45 degrees to the poles. StuRat (talk) 17:37, 4 March 2010 (UTC)[reply]
Note also that the eccentricity of the Moon's orbit (some 42 000 km variation) dwarfs the equator-to-pole variation and the horizon-to-overhead variation (6 300 km). Between that and the Moon illusion (note that said illusion also holds for the sun, which has no appreciable difference in observable size, so there really is a psychological component), I'm not sure the naked eye would actually observe the difference in an equatorial versus England-based viewing. — Lomn 17:47, 4 March 2010 (UTC)[reply]
I posted an gif file that shows the aparent movement of the moon throughout a month (libration) that shows clearly how large is the change in aparent size of the moon due to the eccenticity of its orbit. Dauto (talk) 18:23, 4 March 2010 (UTC)[reply]
Interesting animation. It appears to show the Moon rolling slightly upward over the course of the month, which confuses me since one side of the Moon always faces the Earth. Is this just part of a little wobble, or is this an illusion, too ? StuRat (talk) 18:57, 4 March 2010 (UTC)[reply]
Libration should have all the details, but it's quite real. The Moon rotates on its axis at an even rate, but it orbits the Earth at a varying rate (due to the orbital eccentricity). The net effect is that the Moon alternately lags and leads, showing nearly 60% of its surface over the course of a sidereal month, even though only the same 50% is visible at any given point in a sidereal month. — Lomn 19:03, 4 March 2010 (UTC)[reply]
Is that additional 10% entirely due to libration, or is a small portion also due to having a slightly different view of the Moon from different locations on Earth ? StuRat (talk) 19:26, 4 March 2010 (UTC)[reply]
Some very small fraction is gained by moving around the Earth (or by waiting for the Earth to revolve; same effect), but the vast majority is due to the libration. I'd guess it's a difference of three orders of magnitude, though I haven't tried to work the math. — Lomn 21:53, 4 March 2010 (UTC)[reply]

Great pace - 3 answers in less than 15 minutes! Yeah figured it wasn't likely but wasn't sure of the numbers. Thanks everyone for their input - just came to me on the way home and figured what the hell i'll ask. ny156uk (talk) 17:44, 4 March 2010 (UTC)[reply]

Follow-up question for any who might know: My intuition would be that the size wouldn't change at all due to relative distance (as noted, the delta in the distance is minuscule compared to the total distance), but that it might vary a bit based on changes in the atmospheric lensing. I'm not a physics guy, so I'm probably wrong, but wouldn't the appearance be distorted the further you are from the equator, depending on the season? Perhaps slightly larger in the Northern hemisphere's winter and with a slight disturbance to its shape? Probably not all that observable on a human scale (particularly given the problems with looking directly at the sun), but more measurable than the effect already addressed? Similarly, the shape would change somewhat over the course of the day, since at sunrise and sunset it would be passing through a greater volume of the atmosphere than it does at solar noon? Anyone know if that has an effect greater than, say, a couple millimeters in observed size? —ShadowRanger (talk|stalk) 18:14, 4 March 2010 (UTC)[reply]
Atmospheric effects ca definately deform the observed shape of the sun. Look at that picture from the green flash article. Dauto (talk) 19:37, 4 March 2010 (UTC)[reply]
So presumably for some seasons and latitudes you'll get a visible effect even at midday. After all, above the Arctic circle the sun spends some time effectively permanently rising (or setting, however you want to view it). Still curious if places like London or Manhattan see any such effects (outside of sunrise and sunset), but that at least partially satisfies my curiosity. —ShadowRanger (talk|stalk) 19:52, 4 March 2010 (UTC)[reply]


I've taken the liberty of thumbnailing the images posted in this thread; the rest of the reference desk should be have to be subjected to such large and distracting images. User:Curious Cactus 20:43, 4 March 2010 (UTC)[reply]

Would you believe "should not"? --Anonymous, 01:30 UTC, March 5, 2010.

One further point. Do you notice that the Sun looks smaller in July than in January when it is 3,000,000 miles (5,000,000 km) closer? Then why would you expect to notice when it is 4,000 miles closer due to being seen from a different place on the Earth? --Anonymous, 01:32 UTC, March 3, 2010.

Highway-side notches

There's these weird notches in the pavement that alert a driver to if they are drifting too far to the left or right side of the highway. I want to know how they work, and if we have an article about them. Mac Davis (talk) 17:47, 4 March 2010 (UTC)[reply]

Rumble strips. -- Coneslayer (talk) 17:48, 4 March 2010 (UTC)[reply]
Thanks. I've noticed that if you drive backwards on them no noise is made, while if you drive forward on them, noise is loud. How could they be designed that way? Mac Davis (talk) 17:57, 4 March 2010 (UTC)[reply]
I believe (not 100%) that this is due to the way in which some strips are cut at an angle. If you drive forward, the wheel slowly (relatively speaking) descends the slope, then hits the sharp cut at the far side with the full force of your forward momentum, which jolts both the wheel itself and the attached suspension, creating a louder sound. In reverse, the only impact is the fall off that same ledge onto the slope, so the primary impact is only as strong as the momentum produced by gravity in a fall of an inch or so; your reverse momentum continues with minimal impact.
Of course, when you're going forward, you're usually going faster, so you're going to get more noise in any event, since you rarely go fast in reverse. But for some rumble strips, the slope-and-wall design would account for your experience. —ShadowRanger (talk|stalk) 18:04, 4 March 2010 (UTC)[reply]
Ah, that makes perfect sense. I should have thought of that. It seems like the notches are not very wide compared to the tire circumference though. I don't know if the tire would really go "up" or "down" a slope. Mac Davis (talk) 18:29, 4 March 2010 (UTC)[reply]
Parts of the tire would. They're flexible. So no, usually the tire never reaches the bottom (unless you're really low on air), but it still descends enough to trigger the effect mentioned. —ShadowRanger (talk|stalk) 18:30, 4 March 2010 (UTC)[reply]

Popcorn

My bag of Boston's Lite Popcorn advertised that it was "whole grain" -- what does that even mean? That they don't remove that brown chaff, because it doesn't seem like anyone removed it (other than corn pops, but that's not popcorn). DRosenbach (Talk | Contribs) 18:27, 4 March 2010 (UTC)[reply]

Whole grain means the bran, endosperm and germ of a plant are found in the product. Mac Davis (talk) 18:31, 4 March 2010 (UTC)[reply]
I wasn't asking for the definition of "whole grain" in general, but for its application to corn. DRosenbach (Talk | Contribs) 18:34, 4 March 2010 (UTC)[reply]
It probably means someone in the marketing department had a clever idea. You need the kernel intact in order to trap the moisture to enable popping in the first place. Whole grain is big, so they decided to cash in by mentioning it (despite the fact that it was already the case). At best a "non-whole grain" popcorn might be able to trim a tiny part off the "point" of each kernel (where it connected to the cob) without compromising structural integrity, but I doubt any effort is made to keep or remove it, they just do whatever is most efficient to strip the kernels. —ShadowRanger (talk|stalk) 18:34, 4 March 2010 (UTC)[reply]
So it's a scam -- just as I thought. Thanx! DRosenbach (Talk | Contribs) 18:36, 4 March 2010 (UTC)[reply]
Well, it's a scam if they are trying to imply that their product is any healthier than other popcorn. On the other hand, it might be useful to remind people that popcorn is a whole grain snack, when comparing with other snacks, like pretzels, where some may be whole grain and others may not be. Of course, the good the extra fiber does you probably pales in comparison with the harm all that salt and grease does. StuRat (talk) 18:49, 4 March 2010 (UTC)[reply]
As a good friend of mine says: popcorn is merely a vehicle for butter and salt. Vespine (talk) 23:26, 4 March 2010 (UTC)[reply]
You can get hot-air popcorn poppers that add no fat, and you can refrain from adding salt, and then you have a healthy snack that's as tasty as Styrofoam packing peanuts. +Angr 23:49, 4 March 2010 (UTC)[reply]
Plus, it's cariostatic (and that was a nice take on it, StuRat...never considered that). DRosenbach (Talk | Contribs) 04:54, 5 March 2010 (UTC)[reply]

How intelligent are beavers

Moved from Talk -- Coneslayer (talk) 18:26, 4 March 2010 (UTC)[reply]

How intelligent are beavers? Has anyone done comparative tests? The article on Beaver dams suggests that their dam building is hardwired in but I guess that still makes them the second most intelligent species on the plnet when it comes to building dams, but how do they do in more general things? KTo288 (talk) 18:25, 4 March 2010 (UTC)[reply]

I think your premise is based on a skewed view of the definition of intelligence. Bees and other wasps are able to construct virtually perfect three dimensional hexagonal hive structures, but I don't think that makes them smart. Weaver birds construct ridiculously cool nests...but really, all bird nests are cool. It would probably take you a long time to make one, and if likely wouldn't be as neat and sturdy, and you are using two hands with 5 fingers each, while birds generally do it with a beak. Intelligence has nothing to do with this. DRosenbach (Talk | Contribs) 18:30, 4 March 2010 (UTC)[reply]
Intelligence has more to do with pattern recognition and creation based on learned patterns than following instinctive behavior. Mac Davis (talk) 18:32, 4 March 2010 (UTC)[reply]


It's not hard to see that a certain amount of very simple but 'logical' thought has to go into the logistics of harvesting trees and building a dam. But,what makes you think that other rodents don't have similar cognitive abilities? Just because squirrels don't have an instinctive ability and need to build damns doesn't mean that they couldn't solve very simple problems. Squirrels often come up with clever ways of defeating anti-squirrel bird feeders. Chipmunks store a surprising amount food for the winter.
A lot of that is trial and error, and not planning things out ahead of time, but it's not like beavers don't make mistakes.
Sometimes beavers fell trees and then later realize they can't get them to the dam site. (I wonder if they're capable of feeling frustration.) Sometimes they start a dam in one place and then abandon the site and build in a slightly different place on the same stream. Sometimes they use a rock or tree as a major structural piece that isn't really tied down well at all.
What makes beavers seem so outstandingly intelligent is their determination and hard work. They make lots of screw-ups, but they keep at it until they get it right. Then you come along in the morning and see their completed dam and imagine that they built it right the first time. APL (talk) 19:37, 4 March 2010 (UTC)[reply]
Our article North American Beaver unfortunately does not refer to the intelligence of the species, but does note that in one experiment, researchers placed a cassette tape player, playing the sound of running water, in a field, and the beavers ran over and covered up the tape player with mud and wood to try to make a dam. I believe the editor who introduced this information may have held the view that beavers are idiots. Comet Tuttle (talk) 18:34, 4 March 2010 (UTC)[reply]
It is sometimes difficult for us to distinguish between intelligence and instinct. Spider webs also seem amazing, but they are apparently made via a complex set of instinctual actions, as opposed to higher thought and problem solving ability. A more obvious case is birds that "talk". At first it might seem that they are intelligent and holding a conversation, but in short order you realize they just repeat sounds they hear without any knowledge of what they mean. StuRat (talk) 18:43, 4 March 2010 (UTC)[reply]

Intelligence is task-specific, so your inquiry cannot really produce any useful results. Suffice to say that beavers can respond to varying conditions fairly well. My dad works as a civil engineer, and there are often issues with beavers causing problems in his municipality. They can make do in a semi-urban landscape, until wildlife control steps in. Vranak (talk) 18:51, 4 March 2010 (UTC)[reply]

Thanks for the responses (sorry for asking in the wrong place). From the responses above I guess I'm concentrating too much on the results of what they do rather than how they do it.KTo288 (talk) 21:46, 4 March 2010 (UTC)[reply]

PAD vs PADI (peptidylarginine deiminase)

Reading some papers on peptidylarginine deimmanses has gotten me into a fluster! The enzyme citrullinates arginine to citrulline with some interesting consequences, in most the literature the enzymes are abbreviated to PAD, a few refer to them as PADI. However a few papers imply that the PADI is the gene sequence that encodes for PAD.

Some clarity on the matter would be appreciated as I don't want to be refering to genes as if they were proteins...

129.31.206.253 (talk) 19:22, 4 March 2010 (UTC)[reply]

I should add the wiki pages aren't helpful, titles refer them as PADIs whereas references refer to them as PAD MedicRoo (talk) 19:26, 4 March 2010 (UTC)[reply]

Usually gene names and protein names are the same, with the gene name in italics and the protein Capitalized. However, sometimes there are nomenclature differences between species, e.g. human and rodent. Searching at the Entrez database, shows Padi3 in rats and PADI3 in humans. Pad3/PAD3 are listed as synonyms. I suggest that the PADI vs PAD is probably due to either some minor disagreement amongst researchers or different names in different species. Also, HUGO is the human gene nomenclature database. Similar results can be found there. -- Flyguy649 talk 19:38, 4 March 2010 (UTC)[reply]
PAD doesn't seem like a good abbrev, to me, since it's also an abbreviation for so many other things, including at least one item in the medical field: Peripheral artery disease. So, if a medical journal has a title "New research on PAD", which do they mean ? StuRat (talk) 19:57, 4 March 2010 (UTC)[reply]
If you think that's bad you should check out APC, specifically the four distinct meanings in molecular biology/biochemistry.131.111.185.68 (talk) 23:32, 4 March 2010 (UTC)[reply]
StuRat, it is a minor problem since identifying which protein is the subject of the article is usually apparent from other words in the title. If that doesn't make it clear, the abstract will. Clicking on an article found in Pubmed goes to a page with the abstract and a link to the article. Also using an advanced search can help limit results. For example, I used to read papers discussing AP2. The one I was interested in is involved in vesicular trafficking; the other is a transcription factor. I could usually tell by the title which it was. (Both those articles are terrible, by the way). -- Flyguy649 talk 16:30, 5 March 2010 (UTC)[reply]

Unknown beetle that looks like a large ladybug

I found a beetle on my wall today, and am trying to determine what it is. Unfortunately I don't have a picture because, as I was watching it, it attempted to fly away but had a fail and fell down behind something. Now I can't get to it. It looked just like a ladybug, however, except larger. It appeared to be about 1 cm in diameter, shaped like a half-sphere(ie, not a flat insect). It had the second pair of flight wings like ladybugs and many other beetles do. I didn't get a good look at what its head looked like due to the angle it was at and how large the round "shell" wings were, and when I tried to walk closer it fell down, but I believe there may have been long antennae present. Does anybody know of any beetles that resemble this? —Preceding unsigned comment added by 69.243.51.81 (talk) 20:24, 4 March 2010 (UTC)[reply]

You'd have to give your physical location to get a useful answer. There are millions of insect species, most of which are confined to extremely limited ranges. —ShadowRanger (talk|stalk) 20:37, 4 March 2010 (UTC)[reply]
If it looked like a large ladybug, it probably was. It's size just means it's engorged on aphids, a normal part of the ladybugs life cycle at this time of year. User:Curious Cactus 20:40, 4 March 2010 (UTC)[reply]
Ladybugs -- Coccinellidae -- are beetles. 1 cm in size is at the upper end of their size range, but there are species that reach 1 cm size. There are also other beetles with approximately "half sphere" body shape; for example, some Chrysomelidae and a number of water beetle species/genera from several families. --Dr Dima (talk) 21:27, 4 March 2010 (UTC)[reply]
The OP never denied that ladybugs are beetles. In fact, he confirmed it when he wrote "ladybugs and many other beetles". +Angr 21:32, 4 March 2010 (UTC)[reply]
I'm from maryland, so about halfway up the eastern coast of the usa. When I looked up ladybugs, most of the sources I found said that they didn't usually grow past 5mm, so finding one twice that size was what made me think it must be a different species. 69.243.51.81 (talk) 22:31, 4 March 2010 (UTC)[reply]
There are very many different species of 'ladybugs' or Coccinellidae beetles (5,000+ worldwide, 450+ in North America), and some are indeed up to 10mm in diameter. It's probably impossible for us to be sure what this one was without a good picture, but Harmonia axyridis (aka the Asian lady beetle, Japanese ladybug or Harlequin Ladybird) seems like a good candidate. 87.81.230.195 (talk) 14:19, 5 March 2010 (UTC)[reply]
This is the conclusion my friend and I came to last night. She has those kind in her house, and she routinely sees them as large as I saw the one yesterday, so we think it may have come home in my bag from her house. Thanks for your help. 69.243.51.81 (talk) 16:55, 5 March 2010 (UTC)[reply]

Is there any life on the planet Earth?

If we were looking at Earth orbiting a star, is there anything about it which would indicate that it had life? How close would you have to get to detect life? 92.29.76.62 (talk) 20:52, 4 March 2010 (UTC)[reply]

You could detect radio signals from earth that would show there was people with that technology. User:Curious Cactus 20:58, 4 March 2010 (UTC)[reply]
There's also an argument that the mere presence of abundant atmospheric oxygen would indicate life. The lack of oxygen wouldn't make life impossible, but according to Neil deGrasse Tyson on the Colbert Report a few days ago, oxygen is too prone to reacting with other elements to remain stable without living things to produce it (e.g. cyanobacteria). I doubt this theory is universally accepted, but it's another opinion on the matter.
Your question does require some parameters though. How close are the observers? Martians could see the light from cities on our night side and deduce it was artificial. How technologically advanced are they? We're improving our ability to examine distant stars all the time; as recently as a few years ago we couldn't actually detect planets outside our solar system, now we can detect particularly large ones and even divine some of their properties (mass, distance from their star, certain facts about their atmosphere, etc.). —ShadowRanger (talk|stalk) 21:13, 4 March 2010 (UTC)[reply]
It's tough to guess how the Martians would think. If the Martians don't know about aerobic life, and instead regard oxygen as a deadly poison, it might not immediately occur to them that high oxygen levels were an indicator for life. Their scientists might spend years debating what sort of unusual geologic process is causing our high oxygen levels. APL (talk) 22:07, 4 March 2010 (UTC)[reply]
Martians could also see the greening of each hemisphere's temperate region when it's spring occurs. They might not immediately think of photosynthesis and chlorophyll, but maybe they would. StuRat (talk) 22:32, 4 March 2010 (UTC)[reply]
I won't say it is impossible, but I think it is unlikely one could see terrestrial city lights from Mars. The total luminosity of civilization, as viewed from Mars, would be less than 1/1000th of a bright star, and you'd have separate that from reflected sunlight and moonlight. That would be a technically challenging thing to resolve directly. Dragons flight (talk) 06:28, 5 March 2010 (UTC)[reply]
1/1000 corresponds to about 7.5 magnitudes. I'm not sure what you mean by "a bright star" exactly, but you must mean at least magnitude 1, I sure, so we're talking no dimmer than magnitude 8.5. That's easily visible with a human eye looking through binoculars. If you look when Mars is at opposition and it's a new moon, the Earth's disc should be completely dark (except near the limbs where some light will be refracted by the atmosphere) and seeing down to mag 8.5 through binoculars shouldn't be at all hard. With a decent telescope, you would have no difficulty at all. --Tango (talk) 06:39, 5 March 2010 (UTC)[reply]
In response to Cactus: Intelligent life is different from "life". Radio signals might give the game away, but the question of how to detect low complexity life is quite different. —ShadowRanger (talk|stalk) 21:24, 4 March 2010 (UTC)[reply]
Of course detection of intelligent life is one indication of "life". My understanding of the state of SETI technology is that it is unlikely that even the most advanced receivers we use today could pick up our own transmissions, with the possible exception of our highest powered military radars, were they being emitted from as far away as the next to nearest star. This was mentioned in a Google Tech Talk on SETI, but I would like to find a better reference and add it to the SETI article. The article currently says: Furthermore, the earth emits considerable radio radiation as a byproduct of communications such as TV and radio, and these radiations would be easy to recognize as artificial due to their repetitive nature and narrow bandwidths. If this is typical, one way of discovering an extraterrestrial civilization might be to detect non-natural radio emissions from a location outside our solar system. I think most people believe it would easy to pick up Earth's television emissions from anywhere within a 73.5 light year radius. 124.157.247.225 (talk) 00:42, 5 March 2010 (UTC)[reply]
With regards to people asking about the observers; the OP is specifying if we could detect that life. So, I think we can assume the observes are oxygen-breathing, and at a technology level equal to ours. The question of distance is still unanswered, though. Vimescarrot (talk) 22:29, 4 March 2010 (UTC)[reply]
If we were looking at a duplicate Earth, the basic answer is that we could identify intelligent transmitting life from within a radius of a few dozen light years(maybe even a few hundred, depending on the power and directionality of the broadcast) if we were looking in the right direction, but we probably couldn't detect the planet itself if it wasn't in our solar system. If the life isn't using electricity, we'd be blind to anything outside the solar system.
In the next few years (once our interstellar planet detection technology improves a bit so we can detect Earth-sized planets and determine the composition of their atmosphere accurately), we could probably identify the presence of an Earth-sized world with an oxygen atmosphere in a our immediate stellar neighborhood (a few dozen light years in any direction) which would indicate, but not prove the existence of life. —ShadowRanger (talk|stalk) 23:00, 4 March 2010 (UTC)[reply]
Life was detected on earth with the Galileo and LCROSS probes, but of course these probes were quite close (well within our own solar system). -- Coneslayer (talk) 12:40, 5 March 2010 (UTC)[reply]

Council house with air conditioning

A council house near me has what looks like air-conditioning. The house is still owned by the council. The climate in the UK does not require air-conditioning. What could explain this? Thanks 92.29.76.62 (talk) 20:56, 4 March 2010 (UTC

Isn't it obvious? The previous residents must have had it installed. Maybe they just had spare money and decided they wanted to try it. Also, the UK does have a climate that would warrant air conditioning in the summer. Especially since global warming it getting worse and worse. User:Curious Cactus 21:02, 4 March 2010 (UTC)[reply]

Methinks you do not know what a council house is. It was installed quite recently. 92.29.76.62 (talk) 22:04, 4 March 2010 (UTC)[reply]

What kind of air conditioning? Window units or central air? If window units, are they present in every single flat? If they're only present in some, then I'd guess the tenants had them installed at their own expense. I agree it's unlikely a city government would pay for air conditioning for all council house tenants in a climate like the UK's. On the other hand, you only said it looks like air conditioning. Is there anything else it could be? +Angr 22:15, 4 March 2010 (UTC)[reply]
A/C doesn't only lower temperatures, it also lowers humidity, and you do sometimes get high humidity there, as on a foggy day. (There are also dehumidifiers, which only lower humidity.) While I doubt if anyone in the UK would die from a lack of A/C, they could certainly be more comfortable at certain times of year with it. Now the question is whether they should pay, at taxpayer expense, to make poor people more comfortable. Some would say yes, others would say no. I would tend to answer differently depending on why they are poor. If they are a drug addict who can't hold a job, then hell no. If they are a paralyzed veteran who was wounded in combat, then yes. StuRat (talk) 22:21, 4 March 2010 (UTC)[reply]
Re people dying from lack of A/C: Newspapers do report old people dying of heat in un-air-conditioned homes during hot summers, actually. I don't know if they specified exactly what the cause of death was. Vimescarrot (talk) 22:26, 4 March 2010 (UTC)[reply]
Vimescarrot: Heatstroke. Ks0stm (TCG) 22:37, 4 March 2010 (UTC)[reply]
You only get fog on about two or three days in winter - it is clear that you are uniformed about the matter that you are taking about. 92.29.76.62 (talk) 23:21, 4 March 2010 (UTC)[reply]
Well, if you can't specify the location any more precisely than "UK", you can't expect us to know the weather very precisely, either, now can you ? StuRat (talk) 03:09, 5 March 2010 (UTC)[reply]
The above applies to all of England and Wales, and probably most or all of Scotland. 78.151.93.38 (talk) 14:26, 5 March 2010 (UTC)[reply]
So, no place in the UK gets fog more than 3 days a year, and always in winter ? I don't believe that. StuRat (talk) 15:01, 5 March 2010 (UTC)[reply]

Great! Now my question is completely ruined and I will never get a sensible answer but several paragraphs of inevitable bickering and worse-than-useless uninformed speculation! 92.29.76.62 (talk) 22:47, 4 March 2010 (UTC)[reply]

Welcome to every day on the Wikipedia Reference Desk. Enjoy your stay. User:Curious Cactus 22:52, 4 March 2010 (UTC)[reply]
In your question, you basically asked us to speculate on something we have no experience of. We haven't seen the house you're talking about, nor are we in charge of what appliances are installed in it. Unless you provide more information (like answering the questions I asked above, or taking a photograph of the building and posting it here), uninformed speculation is all you can hope for, because quite frankly, it's all your question in its current state can possibly be answered by. +Angr 23:27, 4 March 2010 (UTC)[reply]
It is customary and normal etiquette to only answer questions you have knowledge of, rather than just guessing from several thousand miles away. 84.13.22.99 (talk) 00:57, 5 March 2010 (UTC)[reply]
Yes but clearly most of the respondents did feel they have sufficient knowledge to be able to offer informed speculation. There's no requirement you actually live in the UK to answer a question concerning the UK. As it stands, the OP still hasn't provided any information to ascertain whether it's possible the tenant installed these airconditioning units which is surely a key point. It's customary and normal etiquette to accept it when people are trying to help regardless of whether you agree with the answer particularly when the question calls for extensive speculation because of the nature of the question and the failure of the questioner to provide any real information even when asked. I would add that I believe VimeCarrot lives somewhere in the UK & Angr lives in Germany, so not 'several thousand miles away'; and I have no idea where User:Curious Cactus lives, do you? Also for all the complaints of pointless bickering, part of the reason this question has gotten offtrack is because of the OP's complaint. Nil Einne (talk) 01:57, 5 March 2010 (UTC)[reply]
Are you talking about a window A/C, or a central A/C?. I'm not from the UK, but if it's a window unit, I don't see what's wrong with Cactus's answer. They're cheap and easily installed, someone could have done so on a whim, or because they're fussy or medically sensitive to temperatures or air quality, or whatever.
Central A/C might be more of a mystery, if you're sure that's what it is. In a commercial building I would just assume that there was some technology in the building, like a server-room, that needed A/C, but that doesn't work in a residence. APL (talk) 02:24, 5 March 2010 (UTC)[reply]
I was reluctant to say this given the OP's complaints but screw it. It's perhaps worth remembering that many A/Cs can function as heatpumps and would probably be the most efficient form of electrical heating. Here in NZ, they're extensively promoted as such given that many of our houses have extremely shit insulation and many places lack gas heating it probably makes sense. Of course many may use them in the summer as well and this is useful in quite a few parts of the country like Auckland. I appreciate things are different in the UK, but it seems easily possible that the same thing would be an added incentive for some people to have them installed. This ref [27] says they're starting to become standard in new urban developments for example. Nil Einne (talk) 02:41, 5 March 2010 (UTC)[reply]
How sure are you that it is a council house and not an ex-council house? The right to buy scheme has resulted in a very large number of council houses now being in private hands. --Tango (talk) 04:49, 5 March 2010 (UTC)[reply]

Its absurd that people from the other side of the Atlantic who may never even have set foot in the UK in their lives or even know what a council house is should believe that they can and should answer a question about a UK council house. 89.242.242.77 (talk) 10:04, 5 March 2010 (UTC)[reply]

I don't understand why you are so bemused by this. Occupants of council houses have great flexibility in making improvements to them, although the council can insist that it they are kept in good order, and that changes are reversed when the occupants leave. If the current occupants plan on staying for the long-term, they might well decide it is worth installing air conditioning (depending on the system, it may also be possible to take it with them if they move). While most occupants of council houses are on low incomes, it isn't essential; they can stay on if their income increases, and in the few areas where there is a surplus of council housing, they are readily available to anyone. Warofdreams talk 11:50, 5 March 2010 (UTC)[reply]
Oh no! Government-owned, subsidized housing is completely unknown out here in the Rest Of The World, How will we ever answer your question about why a residential unit might need a piece of equipment that looks like HVAC unit??!?!?!?
Ok, I think I've figured out the answer you're fishing for, Here you go : "Because the [Political Party] is pandering to the lazy and shiftless poor, and needlessly wasting your tax dollars providing unnecessary luxuries to the [ethnic group] poor!" Does that help? APL (talk) 15:25, 5 March 2010 (UTC)[reply]
A tangental query, but what areas have a surplus of council houses please? I thought there were long waiting lists for them. Thanks 78.151.93.38 (talk) 14:07, 5 March 2010 (UTC)[reply]
Certain deprived areas in the north and Scotland, which have a lack of demand for housing in general, have surplus council housing. Here, for instance, is a recent document from West Dunbartonshire Council noting that they have around 350 council properties with no demand, while here is something similar from Salford. Of course, it may be that these houses are run down, and with a little investment would be in demand - describing them as "surplus" might well be a way to justify selling them or demolishing them and selling the land, to make the council some quick cash. Warofdreams talk 15:00, 5 March 2010 (UTC)[reply]

The what-looks-like air conditioning is a large metal box on the ground within a few inches of the wall of the house containing a large fan, with pipes from it going into the house. It is painted exactly the same colour as the walls of the house, suggesting it was installed at the expense of the local government who own the house. I wonder if it might be required for something like a kidney dialysis machine, but in the past they required an extra room to be built on to the house to contain the machine, and there is no extension on that small house. Perhaps those machines have got smaller in recent years. Council houses are lived in by people who are poor, particularly in that part of the country, and I am sure they would not buy air conditioning on a whim as they are not needed here. The house is still definately owned by the local government. I think its practically impossible that its some sort of cooling equipement for computer servers, as the local council would not give permission, and the people are very likely to be poor, uneducated, and irresponsible. 78.151.93.38 (talk) 14:24, 5 March 2010 (UTC)[reply]

Wow, lots of stereotypes there. There are many legitimate reasons why responsible, educated people can find themselves to be poor, such as divorce (and thus needing to support two households on an income that formerly only supported one), illness (making work impossible), etc. And perhaps the people in that house have slightly improved their financial situation, enough to afford A/C, but perhaps not enough to afford better housing. If they needed to get permission from the council to install the A/C, one of the requirements might well have been that they make it blend in as well as possible, including matching the paint color.
Modern dialysis machines are completely self contained and require no external cooling. There could also be a medical necessity for A/C, such as if the person is morbidly obese and thus more prone to heatstroke. Or, they could have severe allergies which prevent them from cooling the house by opening windows. I suggest you contact the council directly, and ask them for the details. StuRat (talk) 15:18, 5 March 2010 (UTC)[reply]

energy from earth's rotation

i'm sure it must be unfeasable, but can't see exactly: why can't we derive power from the angular momentum of the earth? 109.246.247.147 (talk) 20:58, 4 March 2010 (UTC)[reply]

See? Do you mean Sea? User:Curious Cactus 20:59, 4 March 2010 (UTC)[reply]
corrected to we 109.246.247.147 (talk) 21:17, 4 March 2010 (UTC)[reply]
We can derive power from the rotation of the Earth, that's what tidal energy is. Tides are produced by the rotation of the Earth relative to the Moon and, to a lesser extent, the Sun. StuRat (talk) 21:01, 4 March 2010 (UTC)[reply]

this is correct, but doesn't quite answer the question i had in mind, which is why is it not possible to transfer momentum from the earth's rotation directly into torque or some other means of driving a earth-based engine? 109.246.247.147 (talk) 21:17, 4 March 2010 (UTC)[reply]

Virtually all power generation is based on energy level differentials. In order to derive power *directly* from the angular momentum of the Earth (as opposed to indirectly through the tides), you'd need to find a way to connect it to something not experiencing that momentum. And anything you attached it to would be giving up its kinetic energy to provide the power; deriving energy from the Moon for instance would cause a gradual orbital decay (and/or reduce the rotational speed of the Earth). The tides (and tidal power) are doing basically the same thing, it's just on a scale so low that it has minimal effect on our own rotation and the Moon's orbit. Of course, I'm not a physicist, so I'm sure I made some minor errors in my explanation, but I believe the basic point is sound. —ShadowRanger (talk|stalk) 21:20, 4 March 2010 (UTC)[reply]
The real reason is angular momentum conservation. BTW the moon's orbit is actually rising, not decaying. Dauto (talk) 21:30, 4 March 2010 (UTC)[reply]
I always thought that an orbit can "decay" either upwards, until the object leaves orbit, or downward, until it crashes. StuRat (talk) 22:24, 4 March 2010 (UTC)[reply]
Obviously, since the Earth is slowing down. The moon takes up much of that energy (another part is dissipated as heat, and yet another part should actually lift the Earth into a higher orbit around the sun...). --Stephan Schulz (talk) 21:34, 4 March 2010 (UTC)[reply]
I would say true, but not obvious.Dauto (talk) 21:38, 4 March 2010 (UTC)[reply]
Hmm. If the moon's orbit decays and the Earth slows down, you need a really big energy sink somewhere. --Stephan Schulz (talk) 21:42, 4 March 2010 (UTC)[reply]
Heat?Dauto (talk) 21:45, 4 March 2010 (UTC)[reply]
Yep. The tidal energy pretty much all turns into heat eventually, either in the water or in the Earth's core, mantle, and crust. StuRat (talk) 22:05, 4 March 2010 (UTC)[reply]
It was a rhetoric question but thanks for answering anyways.Dauto (talk) 03:10, 5 March 2010 (UTC)[reply]
We had pretty much the same question recently about attaching a gear to the earth Earth GearVespine (talk) 21:43, 4 March 2010 (UTC)[reply]
What you want is a 'Gyrogenerator' as seen here in the Museum Of Unworkable Devices. (Second one down on that page.)
Perhaps someone better at crunching the numbers could tell us if this would actually work. (Assuming sufficiently frictionless equipment, of course.) APL (talk) 22:28, 4 March 2010 (UTC)[reply]
I think you are confusing energy and momentum. Steve will be along soon to explain it. --Tango (talk) 05:04, 5 March 2010 (UTC)[reply]

acidosis

what is the treatment for urinary acidosis other than iv bicarbonate which is just a temporary fix. no other problems can be found to cause this. the person is not diabetic. (this is a hypothetical question.. im not asking for the cause but the treatment. —Preceding unsigned comment added by Thekiller35789 (talkcontribs) 21:36, 4 March 2010 (UTC)[reply]

Unfortunately, we cannot recommend treatment options, even in the hypothetical. Please read the Wikipedia:Medical disclaimer. Instead, you should seek, or advise your friend to seek, the advice of a qualified medical professional who can see and treat your friend in person. --Jayron32 22:18, 4 March 2010 (UTC)[reply]

contribs) 22:39, 4 March 2010 (UTC)[reply]

Also see the reference desk guidelines on the matter, which explains what we do and don't answer here specifically. Acidosis is a particularily good example of why we have these guidelines, because there are different possible causes, and the best treatment is often to eliminate the underlying cause. We cannot hope to diagnose the underlying cause of a specific instance in a specific person, and then proscribe the best form of treatment. If someone you know does have acidosis, they should discuss it with a medical professional who has the access and training to evaluate and treat the condition, rather than asking a random group of people on the internet.Buddy431 (talk) 22:28, 4 March 2010 (UTC)[reply]


no one i know has acidosis and neither do i . im just interested in it. it sounds to me like their is no real treatment if no underlying cause can be found ?

This looks like a homework question to me. The answer will be in your text book —Preceding unsigned comment added by 79.68.242.68 (talk) 00:46, 5 March 2010 (UTC)[reply]

There is no disease called urinary acidosis. You may be mixing up renal tubular acidosis, which comes in several varieties caused by many different underlying conditions. Treatments also vary according to the underlying condition. alteripse (talk) 02:03, 5 March 2010 (UTC)[reply]


im 45 years old and not in school, lol. the acidosis im taking about is when your urine has a low ph. —Preceding unsigned comment added by Thekiller35789 (talkcontribs) 02:39, 5 March 2010 (UTC)[reply]

There are uncountable reasons why a person's urine would have an abnormally low pH, and without a proper medical examination to determine the underlying cause, it would be absolutely impossible to recommend a course of treatment for it. That's why we don't answer questions like this; if its idle curiosity Urine#pH contains a very brief discussion of the issue, and articles like Acid-base homeostasis discusses the physiological pH levels in general. You could follow links from there to see what you find. If you have specific concerns about your own urine or the urine of people whom you know, seek a doctor. --Jayron32 06:27, 5 March 2010 (UTC)[reply]

zoonoses from dogs

Besides rabies, are there other zoonoses that humans can get from dogs and v v ? Googlemeister (talk) 21:58, 4 March 2010 (UTC)[reply]

Kennel cough User:Curious Cactus 22:53, 4 March 2010 (UTC)[reply]
The kennel cough article says nothing about transmission between dogs and people. Deor (talk) 23:05, 4 March 2010 (UTC)[reply]
Who said that Wikipedia articles are necessarily comprehensive? DRosenbach (Talk | Contribs) 05:01, 5 March 2010 (UTC)[reply]
Wikipedia did! It doesn't say anything about Wikipedia being accurate, though... --Tango (talk) 05:39, 5 March 2010 (UTC)[reply]
What about a barking cough? —Preceding unsigned comment added by 79.68.242.68 (talk) 00:48, 5 March 2010 (UTC)[reply]
A few come to mind:
I'm sure there are others. -- Scray (talk) 06:13, 5 March 2010 (UTC)[reply]
There is at least one other, mange. Dismas|(talk) 16:23, 5 March 2010 (UTC)[reply]

Queen bee naughty bits

I can't seem to find a good diagram of queen honey bee anatomy to answer this question for myself. Apparently the stinger in most stinging insects is a modified ovipositor. A queen bee has a stinger, and she posits ova (I don't know if she has an ovipositor as such), so how does that work? Thanks. --Sean 22:12, 4 March 2010 (UTC)[reply]

It does seem odd and I can't find an explanation. Our article, Bee sting says: "In worker bees, the sting is a modified ovipositor. The queen bee has a smooth sting and can, if need be, sting skin-bearing creatures multiple times, but the queen does not leave the hive under normal conditions." This seems to confirm your statement, but does not really explain it. --Tango (talk) 05:12, 5 March 2010 (UTC)[reply]
Why can't it do double duty ? That is, deliver poison, when needed, and eggs when needed. The urethra in mammal males does double duty and the cloaca of many birds, reptiles, etc., does triple duty or more. StuRat (talk) 12:41, 5 March 2010 (UTC)[reply]
Perhaps it does, but the sizes seem wrong. A honeybee egg looks like a 1/2 scale grain of rice, and all the stingers I've ever seen are tiny in diameter, like a stout dog hair. I'm going to try a more specialized forum and will report back here. --Sean 13:57, 5 March 2010 (UTC)[reply]
I bet a queen bee's stinger is significantly larger, and probably also stretches dramatically when producing young, as a vagina does. StuRat (talk) 14:57, 5 March 2010 (UTC)[reply]

March 5

Water for Hydrogen Fuel in Deserts (cont.)

I am continuing from my last question.

Who has proposed using an energy store in deserts? I have recently read a book called Solar Hydrogen Energy: The POWER to save the Earth which proposed that hydrogen be used to transport solar energy from deserts to other places. It also proposed that most production of solar energy be in deserts so they won't take up farmland.

The water in oceans is saltwater. Could we extract the hydrogen used to transport energy from saltwater?

Most combustion of hydrogen as fuel would be in places outside deserts, places all over the world. They would be very far away from deserts. Alot of the combustion of hydrogen would be in transports and not in producing electricity so they wouldn't be transported back to deserts.

An Unknown Person (talk) 04:44, 5 March 2010 (UTC)[reply]

There have been lots of proposals to use hydrogen as a storehouse for excess energy production from solar, wind, hydroelectric, etc. It will likely eventually become a common fuel source, however there is an utter lack of hydrogen-fuel infrastructure (basically, no fuel stations exist to sell the stuff; no distribution network exists to get it to the stations, etc.) the way there is with gasoline. So you get a catch-22 with hydrogen as a fuel: No one owns hydrogen-fueled cars because you can't get hydrogen anywhere to fill the tank with, and you can't find hydrogen stations anywhere because there are no customers with hydrogen cars to buy them from. This barrier is what has kept fuel cell vehicles off the road, despite being a feasible means of fueling a car for, oh, 50 years or so. --Jayron32 04:56, 5 March 2010 (UTC)[reply]
That doesn't mean it's impossible. I imagine you could start with solar hydrogen production facilities near desert cities, like Las Vegas and Phoenix, and provide refueling stations in those cities and maybe a fleet of rental cars that use hydrogen. You could eventually expand the network with hydrogen pipelines to cities near the desert, like Los Angeles. It might never be feasible to ship hydrogen all the way across the country to places like New York, however, depending on the prices of other fuels. StuRat (talk) 12:32, 5 March 2010 (UTC)[reply]

What does the integral of the position function represent?

That is, suppose we have velocity v'. If we integrate (with respect to time), we get position v + c. What do we get if we integrate again?--70.122.117.52 (talk) 05:06, 5 March 2010 (UTC)[reply]

The integral of the velocity is displacement, not position. That is, if we integrate a velocity function between any two arbitrary points in time, you get the distance traveled over that time. I'm not sure that integrating the displacement function gets you any meaningful physical quality. Mathematically (especially if you have a complicated velocity function) you could repeatedly integrate any function until you get only a constant left, but that doesn't mean that the operation produces a meaningful physical result. --Jayron32 05:18, 5 March 2010 (UTC)[reply]
(ec)The definite integral of velocity is displacement. The OP is talking about the indefinite integral, which would be position if you take the constant as being the starting position. Also, if you repeatedly integrate you don't end up with a constant - that's repeated differentiation. If you repeatedly integrate you end up with some function plus a polynomial (of ever increasing degree). --Tango (talk) 05:32, 5 March 2010 (UTC)[reply]
I don't think the integral of position wrt time has any physical significance. It would have units of length times time (eg. metre seconds), which doesn't correspond to anything useful that I can think of (an angular momentum divided by a force, I suppose, but I can't think why you would ever do that). --Tango (talk) 05:32, 5 March 2010 (UTC)[reply]


Why would you only end up with a constant through iterated integration? Wouldn't that be what happens through iterated differentiation? —Preceding unsigned comment added by 70.122.117.52 (talk) 05:31, 5 March 2010 (UTC)[reply]

Yeah yeah yeah. Stop piling on. --Jayron32 05:34, 5 March 2010 (UTC)[reply]
Two people edit conflicting does not a pile make! --Tango (talk) 05:38, 5 March 2010 (UTC)[reply]
That depends on the size of the people. I have met people who constitute a pile all by themselves. --Jayron32 06:19, 5 March 2010 (UTC)[reply]
Fair point. I don't know about the OP, but I'm pretty skinny - I think you would need at least 3 or 4 of me to constitute a pile. --Tango (talk) 06:24, 5 March 2010 (UTC)[reply]
Neither iterated integration, nor iterated differentiation guarantee that you'll end up with a constant. Even with iterated differentiation, that only happens for polynomials. —Preceding unsigned comment added by 157.193.173.205 (talk) 08:22, 5 March 2010 (UTC)[reply]

Sun

Could a star like the earths sun just collapse in on itself, like due to chaos theory or something? —Preceding unsigned comment added by Jetterindi (talkcontribs) 09:27, 5 March 2010 (UTC)[reply]

No. The sun is not massive enough to become a neutron star or a black hole. In about 5 or 6 billion years it will become a red giant, and then finally a white dwarf surrounded by a planetary nebula. See stellar evolution for more details. Gandalf61 (talk) 09:39, 5 March 2010 (UTC)[reply]
I heard this same thing in grade school about thirty years ago. Don't you mean "4,999,999,970 - 5,999,999,970 years"? Kingsfold (talk) 14:19, 5 March 2010 (UTC)[reply]
Also, "chaos theory" doesn't mean any crazy thing you can think of might happen. It just means that tiny variations in the beginning of an event can have large and unpredictable effects on the outcome. --Sean 14:19, 5 March 2010 (UTC)[reply]

Thinking brain?

The idea that our thinking activity is located inside the head seems very natural, but is it something we have learned, or do we know it because we feel the brain thinking? (like e.g. we feel our belly digesting the food). If we have learned it, when did men first have this intuition, and how did they came to it? --pma 10:07, 5 March 2010 (UTC)[reply]

I saw a really sad program once where a "scientist" chopped the head off two monkeys and put one of the heads onto the other monkey. He threw the other head in the bin without care. The monkey head on the new body was shown to be aware and think. This shows thinking is inside the brain and not the body. —Preceding unsigned comment added by EEEnacells (talkcontribs) 10:15, 5 March 2010 (UTC)[reply]
You obviously saw it in a fictional movie (if your memory is correct), since a head transplant is totally beyond the abilities of science men. 82.113.121.110 (talk) 11:25, 5 March 2010 (UTC)[reply]
According to the Head transplant article, the procedure has been Performed with limited success on dogs, monkeys and rats.... Mitch Ames (talk) 11:34, 5 March 2010 (UTC)[reply]
even worse, if that was the end of the "experiment", it only proved: a thinking activity was either in the first monkey's head, or in the second monkey's body was thinking, and nowhere in the "scientist". But my question is more on the psychological side: how is that we feel that our thinking activity happens inside our head: is it cultural, that is we learn it when we are kids, or is it physiological, that is, we know what part of our body is thinking much the same way that e.g. we know which part of our body is eating or defecating. --pma 11:45, 5 March 2010 (UTC)[reply]
Scientists probably first knew this from people with head injuries, causing brain damage, which led to abnormal thought processes (speech problems, etc.). But people might have thought this intuitively because our perception is associated with our senses, and 4 out of 5 are located on the head alone. This is a rare case where the intuition was right on. One exception seems to be emotion, which instinct told people was in the heart, since it beats faster or slower based on our emotional state. However, emotion, just like logic, actually comes from the brain. StuRat (talk) 12:18, 5 March 2010 (UTC)[reply]
History of neuroscience has some useful information. The ancient Egyptians believed intelligence was situated in the heart; Alcmaeon of Croton (c. 500 BC) is claimed to be the first person to suggest the brain was used for thinking. Galen (AD 129 – 199/217) also did work on brain structure. Although little is known about him, Alcmaeon probably used vivisection and dissection to make his discoveries, e.g. cutting the optic nerve of live animals. --Normansmithy (talk) 12:24, 5 March 2010 (UTC)[reply]

if thez thought intelligence was situated in the heart, what did they think the material in the skull was for? Also, presumably they knew what some of the other organs did because people injured in those organs all suffered in the same way as a result. Didn't a single person in ancient times suffer traumatic head injury that caused brain damage, so it became obvious to everyone that intelligence and cognitive functions are done there? I seriously can't imagine how they could have thought brain-stuff was for anything else than thinking... what did they think it was? 82.113.121.110 (talk) 13:06, 5 March 2010 (UTC)[reply]

The first article linked by Normansmithy above answers your questions directly. Remember that for much of the times under discussion, there were almost no means of communication other than face-to-face, few libraries or other centres of learning existed, and very few people were literate anyway: consequently, most people's opinions could only be based on their own direct experiences. While some did deduce that the brain was the seat of thinking, some thought instead that the brain was merely packing material, and some (following Aristotle's opinion) that its function was to cool the blood which, since the brain does indeed have a very generous blood supply and does indeed create and radiate a significant proportion of the body's heat was, though wrong, not at all unreasonable. 87.81.230.195 (talk) 13:52, 5 March 2010 (UTC)[reply]

The best discussion of this issue that I've seen is in The Mind's I, in two essays by Daniel Dennett called "Where am I?" and "Where was I?". I think the basic answer is that we feel ourselves to be located in our heads because that's where our eyes are -- we are vision-dominated creatures. By using remote-vision equipment it is possible to create a very strong sense of being located outside your body in various ways. Looie496 (talk) 17:11, 5 March 2010 (UTC)[reply]

Goedel and Wikipedia

Does the Goedel incompleteness theorem imply that insofar as Wikipedia is comprehensive, it cannot be accurate, and insofar as it is accurate, it cannot be comprehensive? 82.113.121.110 (talk) 11:30, 5 March 2010 (UTC)[reply]

Gödel's incompleteness theorems deals with questions about the fundamental definitions of maths. It is quite a stretch to apply it to wikipedia, without taking more poetic licence than is normally allocated to pure maths. I think it is nonsense to suppose that wikipedia will ever be complete (even if we knew what that meant in wikipedia terms) not completely accurate. We can duduce all of that without the need to appeal to Gödel. --Tagishsimon (talk) 11:43, 5 March 2010 (UTC)[reply]
Wikipedia will not be complete until it contains an article that is about itself. Looie496 (talk) 17:02, 5 March 2010 (UTC)[reply]

do we benefit in any way from c?

Is there any benefit for anyone from having a current universal speed limit of c? ie, if the Universe were a democracy, and its lawmaker obeyed the will of the people in it, is there any reason we would do well to have the lawmaker keep c? At least two good reasons for abolishing c from the laws of the universe would be: 1) easier intergalactic space travel, easier communication with probes on mars, etc, 2) smaller ping times to China, and faster processing within a single piece of electronics. In fact, in today's 3 GHz processors, I heard that electrons only have enough time between clock ticks to travel a few centimeters (you can verify this for yourself with a simple Google calculation). Now this means we can't possibly make processors much bigger than that, with logic that has electrons travelling far more than that distance, depending on what path they take. So, this might be a simplification, but abolishing c could also bring better computation power. But as I asked above, my question is now: is there ANY benefit at all from actually having c? 82.113.121.110 (talk) 12:41, 5 March 2010 (UTC)[reply]

Your question is nonsense. c is a fundamental constant. Okay, technically there are some limited theoretical circumstances where it bends (shortly after the Big Bang), but we have no control over it. Nature's laws are not up for a vote. —ShadowRanger (talk|stalk) 13:13, 5 March 2010 (UTC)[reply]
In English, when you say "if the Universe were", the were means that the speaker knows he or she is asking about an imaginary case that is contrary to actual reality. (Otherwise he or she would say "if the Universe turns out to be" or even "if the Universe really is"). So, your point is totally invalid, instead please answer my question: is there any benefit to c, and even though we can't, if we could petition to have this law abolished, would there be any practical benefit to us if we could and we did and it were? —Preceding unsigned comment added by 82.113.106.97 (talk) 13:22, 5 March 2010 (UTC)[reply]
There are obvious benefits, as you listed. The question is whether the universe could still exist (in any way still somewhat pleasant to us) if we raised c. After all, it's not a speed limit in isolation of the rest of physics. Fine-tuned universe addresses variations on the question. --Sean 14:32, 5 March 2010 (UTC)[reply]
Thanks. Unfortunately your benefit to c in that without it perhaps the Universe "couldn't exist in a way still pleasant to us" is so very broad. Could you, or anyone else, possibly give a more direct, narrow benefit to having c as a speed limit? Does this actually help in a practical, specific (rather than an overarching universal) way? Thanks. 82.113.121.103 (talk) 14:44, 5 March 2010 (UTC)[reply]
The slow speed of light has so far prevented the evil Zorn empire from invading from their galaxy and eating our brains. :-) StuRat (talk) 14:42, 5 March 2010 (UTC) [reply]
But, seriously, there may well be (or have been) other intelligent species out there which would, at some point, have colonized the Earth if they could only get here, meaning we may never have existed. We don't need to attribute evil motives to them, as they might not have found any intelligent life if they arrived long ago, but maybe they wondered what that slime was in those volcanically heated ponds. StuRat (talk) 14:48, 5 March 2010 (UTC)[reply]
This is exactly the type of answer I'm looking for. Do you know of any other answers of this specific nature? 82.113.121.103 (talk) 14:44, 5 March 2010 (UTC)[reply]
Perhaps next time, you could specify which type of answer you are looking in front, and save us and yourself some time? :-) DVdm (talk) 14:53, 5 March 2010 (UTC)[reply]
um, I didn't have any idea of this answer. I like it because it is specific, and OBVIOUSLY a benefit. I would easily pay any amount of my money not to have my brain eaten by Zorns. Are there any other, specific benefits like this you can list? 82.113.121.103 (talk) 14:58, 5 March 2010 (UTC)[reply]
George Gamow wrote a little book "Mr. Tompkins in Wonderlandwherein the Mr. Tompkins dreams about a world where the speed of light was 30 miles per hour. Relativistic effects are seen when someone rides a bicycle. Surely someone has written a similar work where c was orders of magnitude higher. Radio antennas would get bigger, at least for the same frequency. Optics might have to change their size, at least to focus the wavelengths used presently in vision and photography. I wonder if electron orbitals would have to change, along with the the size of atoms and molecules? Edison (talk) 15:03, 5 March 2010 (UTC)[reply]
Changing c would change the fine structure constant, and Fine-structure_constant#Anthropic_explanation suggests that that would result in a very different, and probably inhospitable, universe. Also, if you are talking about an infinite c (instead of just a larger c), I'm not sure you would have electromagnetic waves at all anymore. -- Coneslayer (talk) 15:11, 5 March 2010 (UTC)[reply]

Okay, the above is a real benefit: smaller radio antennas than if C were larger. Then Cones Layer says the same thing others have been saying, that it's a general requirement for our whole universe. Guys, I got this part. Are there any other specific benefits to the current c, as the bit about smaller antennas is? Thanks. 82.113.121.103 (talk) 15:38, 5 March 2010 (UTC)[reply]


I can think of a couple of Japanese cities that would not have suffered as much if the value of c in E=mc2 were not so large. TimBuck2 (talk) 17:07, 5 March 2010 (UTC)[reply]

new Best answer (so far) as chosen by OP:

As you increase c, I think it would get harder to implement the Global Positioning System with the same accuracy. Obviously it wouldn't work at all with infinite c. If you must split my name in two, it should be Cone Slayer, not Cones Layer. -- Coneslayer (talk) 15:43, 5 March 2010 (UTC)[reply]
Along the same lines, it would be more difficult for us to precisely measure the distance to the moon. —Bkell (talk) 15:54, 5 March 2010 (UTC)[reply]
the GPS is the best practical answer anyone here has given so far, and it is spot on, since it relies on specific timing of distances that are travelled at or nearly at c. Therefore, with an infinite or much larger c, this would become difficult. Can anyone come up with other practical aspects of our life that could not work but for c on the scale it currently is? Thank you. 82.113.121.103 (talk) 16:52, 5 March 2010 (UTC)[reply]

Entropy

Is the Moon a higher or lower entropy environment then the Earth? What caused it? TheFutureAwaits (talk) 12:59, 5 March 2010 (UTC)[reply]

Our article on entropy suggests the definition "entropy is as such a function of a system's tendency towards spontaneous change." As such, the Earth is a lower entropy system than the Moon, as it is more prone to spontaneous change, and it is such primarily because it is larger -- large enough to retain an atmosphere and an active volcanic system. — Lomn 14:06, 5 March 2010 (UTC)[reply]

I think it's the opposite way : low entropy means highly ordered, for example instead of a moon, a perfect sphere made entirely of a single element and uniformly a single temperature would be highly ordered: you could basically describe it entirely in half a sentence, giving the diameter, the element it is made of, and the temperature (maybe I'm leaving out one or two things). Because you can describe it in very few words, it therefore has a very low entropy. Now the moon has much higher entropy than a perfect sphere made of a single element. You would need far more space to describe it fully. But the earth has a higher entropy still: it is much more complex. So, I would say that the Earth is a higher-entropy environment. To put it another way, as a percentage, you increase entropy far more when you put an American flag into the low-entropy conditions on the moon than when you place on in the high-entropy conditions on an Earthly mountain. Can someone better versed in math and science confirm my interpretation? Thanks. 82.113.121.103 (talk) 14:30, 5 March 2010 (UTC)[reply]

That's completely wrong. Entropy is not a measure of how many words it takes to give a macroscopic description of an object. It is measure of how many microscopic states are consistent with the macroscopic description. To labor on your example, if you were to melt the moon, mix if thoroughly and find a way to cool it fast enough to keep the mix uniform the final sphere would have a higher entropy. Read entropy of mixing. Dauto (talk) 15:42, 5 March 2010 (UTC)[reply]
I find this extremely hard to believe. You are telling me if we took the universe, melted it all together, and made a black hole out of it, with precisely 0 information in the black hole other than maybe it's total mass (a single real number, in grams) and MAYBE one or two more variables such as it's spin and charge (maybe) then there would be MORE entropy in Universe (even though you can just describe it as "1 black hole, in the "center" (ha ha) of nothing else, having mass x, charge y, and angular momentum z". Even if you give all of these to an obscenely unrealistic level of exactness, you still will use maybe a paragraph of digits. A paragraph, even using the best theoretically possible compression, is not enough to accurately describe (ie represent a compressed version of) even a single book (say, a collection of Shakespeare plays). So it seems to me that a SINGLE book would have more entropy than all of the universe, if you reduced the universe to a black hole. Likewise, it seems to me that a SINGLE city on Earth would have more entropy than the Moon, if the moon were a uniform substance you can perfectly describe in a few words. If I really am wrong, maybe it's because I'm conflating physical entropy with information entropy? For me, the fewer words you can use to give a second God in a different Universe enough information to fully reproduce an exact copy of something, the lower entropy it has. Our God would need to give a LOT of information to a second God in a different Universe to reproduce the Earth, but considerably less if the Earth were a uniform ball that is an exact geometrical sphere, of fixed temperature, density, etc. Don't you think? Can someone confirm whether I'm right, or whether Dauto above is right? Thank you. 82.113.121.103 (talk) 17:13, 5 March 2010 (UTC)[reply]
These answers are just leading to more confusion. Dauto, is the Moon higher or lower entropy than the Earth? Why? TheFutureAwaits (talk) 15:56, 5 March 2010 (UTC)[reply]
If you take the planet as a whole I would venture that earth's mean specific entropy (entropy per unit volume if you will) will be higher simply because earth's core temperature is higher. Dauto (talk) 16:05, 5 March 2010 (UTC)[reply]

My wording may be off, what I'm getting at is which way is the energy exchange moving? So for example in the Sun-Earth system the Sun is increasing in entropy while the Earth is decreasing. How does this work in the Earth-Moon system? TheFutureAwaits (talk) 16:20, 5 March 2010 (UTC)[reply]

I doubt there's any meaningful give-and-take between the two. Entropy increases. — Lomn 16:36, 5 March 2010 (UTC)[reply]
I don't think the sun's entropy is increasing since the most important factor here is likely the fact that it is losing a massive amount of heat through radiation so its entropy is actually decreasing. The radiation exchange between the earth and the moon is not a very important factor since they are a similar temperatures which means radiation is moving in both directions. Dauto (talk) 16:40, 5 March 2010 (UTC)[reply]

Probably the most important point is that theoretical thermodynamics shows that entropy per se is not very important -- what matters for understanding interactions is the dependence of entropy on energy, which is measured by a quantity we call the temperature. Looie496 (talk) 16:58, 5 March 2010 (UTC)[reply]

panda

Where can you find a giant panda in the United States? —Preceding unsigned comment added by Yelopiclle (talkcontribs) 14:02, 5 March 2010 (UTC)[reply]

From the Giant Panda article...

As of 2007, five major North American zoos have Giant Pandas:

   * Chapultepec Zoo, Mexico City – home of Xi Hua, born on June 25, 1985, Shuan Shuan, born on June 15, 1987, and Xin Xin, born on July 1, 1990 from Tohui (Tohui born on Chapultepec Zoo on July 21, 1981 and died on November 16, 1993), all females
   * San Diego Zoo, San Diego, California – home of Bai Yun (F), Gao Gao (M), Su Lin (F), Zhen Zhen (F), and Yun Zi (M).
   * US National Zoo, Washington, D.C. – home of Mei Xiang (F) and Tian Tian (M).
   * Zoo Atlanta, Atlanta, Georgia – home of Lun Lun (F), Yang Yang (M) and Xi Lan (M)
   * Memphis Zoo, Memphis, Tennessee – home of Ya Ya (F) and Le Le (M)

Googlemeister (talk) 14:18, 5 March 2010 (UTC)[reply]

Who's the best when nature calls?

Hi

1.I've often wondered when you have to go, (pee) but sometimes in a situation where you can't (whatever the reason might be). Who's better at holding it, males or females? -However I don't think females can hold it that long.

2. What are the complications of holding a number 2 for too long?

3. I'm sure most guys have had this happen to them somewhere along their lives. However if you've still somehow managed to elude this experience you're in for a treat. I've been hit a couple of times in the groin, but on one or two occations it hurt so bad that I felt I was going to need a new pair of undergarments.

3.1 Is this normal and what are the complications when the injury is serious?


Thanks, NirocFX

41.193.16.234 (talk) 14:05, 5 March 2010 (UTC)[reply]

Women can hold for slightly longer in tests, but it's negligible. There are no adverse affects to health of holing a poo in too long, you'll simply lose control of the bowls and crap your pants. And it's very normal to experience massive pain for both men and women when impacted on the genitalia area. They risks include hemorrhaging and sterility. R12IIIeloip (talk) —Preceding undated comment added 14:47, 5 March 2010 (UTC).[reply]
I disagree on "holding in poo". The large intestine removes water, and poo held in too long (several days) will thus be dried out and cause constipation. StuRat (talk) 15:37, 5 March 2010 (UTC)[reply]
Cecil Adams covered the potential hazards of holding in "Number 1" for too long here. Doesn't mention the other, though. APL (talk) 15:43, 5 March 2010 (UTC)[reply]

Microbial locomotion time

If I am trimming the fat from some chicken thighs with a 5" chef's knife, how long could I expect it to take for the potentially harmful bacteria from the blade to make their way to the handle such that my right hand (holding the knife) should be reasonably assumed to be contaminated? I ask because I usually tend to perceive my right hand to be totally clean and use it without much discretion in terms of touching other things in the kitchen while preparing raw meat, such getting a pot from the cabinet, taking things from the fridge, etc. DRosenbach (Talk | Contribs) 16:40, 5 March 2010 (UTC)[reply]

Our Swarming motility article describes this as "rapid (2–10 μm/s) and coordinated translocation of a bacterial population across solid or semi-solid surfaces". If we take the upper end of that range (10 μm/s), then even if the bacteria were to move directly toward your hand, it would take more than 40 minutes for them to move an inch (or a couple of hours for a few inches). Add to that the less-directed nature of bacterial movement, the less-than-ideal culture conditions of a knife surface, and the fact that I assumed the upper end of the range of rates. For most users (perhaps not a trained health professional like yourself), it seems extremely unlikely that this mechanism would account for more contamination of your kitchen than a slip in technique (such as setting the knife down in a "clean" versus "dirty" area). -- Scray (talk) 17:13, 5 March 2010 (UTC)[reply]

120 volt equipment

Hello, Q-what happens to 120 volt 60 hz equipment when it is plugged in to 120 volt 400hz power source? my theory is that it will run for awhile, but eventually it would overheat, example such as an electric motor that's 120 volt hz? —Preceding unsigned comment added by 205.200.77.222 (talk) 16:48, 5 March 2010 (UTC)[reply]

Matter

I know matter can be converted to energy, but can it be converted to anything else? Dark matter perhaps?

Ant

Which species of any has a stinger AND bites to inject formic acid into the wounds? —Preceding unsigned comment added by 518c&e (talkcontribs) 17:04, 5 March 2010 (UTC)[reply]

Species upper limit?

Have scientists discovered almost all species that exist? —Preceding unsigned comment added by SpiderLighting (talkcontribs) 17:09, 5 March 2010 (UTC)[reply]